数学の質問スレ【大学受験板】part35

このエントリーをはてなブックマークに追加
1大学への名無しさん
数学の問題に関する質問をどうぞ。参考書・勉強の仕方等は各専用スレで。

質問をする際の注意
・その問題をどこまで解いたのか、どの部分が分からないのか、具体的に書く。
・必要と思われる場合は、自分がどこまで履修済みか書く。(例:1A2Bまで)
・数式を書くときは、極力誤解のない書き方をする。
 例えば、1/2aより、(1/2)a あるいは 1/(2a) のように書いた方が分かりやすい。

数学記号の書き方
http://members.at.infoseek.co.jp/mathmathmath/
図・グラフ掲示板
http://www6.tok2.com/home2/wi2003/cgi-bin/bbs3/bbsnote.cgi

前スレ
part34
http://school4.2ch.net/test/read.cgi/kouri/1093350731/

それ以前は>>2-10あたり
2大学への名無しさん:04/09/25 18:10:16 ID:Z7IXhVew
3大学への名無しさん:04/09/25 18:11:19 ID:Z7IXhVew
4大学への名無しさん:04/09/25 18:11:48 ID:Z7IXhVew
【掲示板での数学記号の書き方例】
■数の表記
 ●スカラー:a,b,c,...,z, A,B,C,...,Z, α,β,γ,...,ω, Α,Β,Γ,...,Ω, ... (← ギリシャ文字はその読み方で変換可.)
 ●ベクトル:V=[V[1],V[2],...], |V>, V↑, vector(V) (← 混同しない場合はスカラーと同じ記号でいい.通常は縦ベクトルとして扱う.)
 ●行列(1成分表示):M[i,j], I[i,j]=δ_[i,j]
 ●行列(全成分表示):M=[[M[1,1],M[2,1],...],[M[1,2],M[2,2],...],...], I=[[1,0,0,...]',[0,1,0,...],...] (← 行(または列ごと)に表示する.)

■演算・符号の表記
 ●足し算:a+b
 ●引き算:a-b
 ●掛け算:a*b, ab (← 通常"*"を使い,"x"は使わない.)
 ●割り算・分数:a/b, a/(b+c), a/(bc) (← 通常"/"を使い,"÷"は使わない.)
 ●複号:a±b=a士b, a干b (← "±"は「きごう」で変換可.他に漢字の"士""干"なども利用できる.)
 ●内積・外積・3重積:a・b, axb, a・(bxc)=(axb)・c=det([a,b,c]), ax(bxc)

■関数・数列の表記
 ●関数:f(x), f[x]
 ●数列:a(n), a[n], a_n
 ●平方根:√(a+b)=(a+b)^(1/2) (← "√"は「るーと」で変換可.)
 ●指数・指数関数:a^b, x^(n+1), exp(x+y)=e^(x+y) (← "^"を使う."exp"はeの指数.)
 ●対数・対数関数:log_{a}(b), log(x/2)=log_{10}(x/2), ln(x/2)=log_{e}(x/2) (← 底を省略する場合,"log"は常用対数,"ln"は自然対数.)
 ●三角比・三角関数:sin(a), cos(x+y), tan(x/2)
 ●行列式・トレース:|A|=det(A), tr(A)
 ●絶対値:|x|
 ●ガウス記号:[x] (← 関数の変数表示などと混同しないように注意.)
 ●共役複素数:z~
 ●転置行列・随伴行列:M', M† (← "†"は「きごう」で変換可.)
 ●階乗:n!=n*(n-1)*(n-2)*...*2*1, n!!=n*(n-2)*(n-4)*...
 ●順列・組合せ:P[n,k]=nPk, C[n.k]=nCk, Π[n,k]=nΠk, H[n,k]=nHk (← "Π"は「ぱい」で変換可.)
5大学への名無しさん:04/09/25 18:12:12 ID:Z7IXhVew
■微積分・極限の表記
 ●微分・偏微分:dy/dx=y', ∂y/∂x=y,x (← "∂"は「きごう」で変換可.)
 ●ベクトル微分:∇f=grad(f), ∇・A=div(A),∇xA=rot(A), (∇^2)f=Δf (← "∇"は「きごう」,"Δ"は「でるた」で変換可.)
 ●積分:∫[0,1]f(x)dx=F(x)|_[x=0,1], ∫[y=0,x]f(x,y)dy, ∬_[D]f(x,y)dxdy, 点[C]f(r)dl (← "∫"は「いんてぐらる」,"∬"は「きごう」で変換可.)
 ●数列和・数列積:Σ_[k=1,n]a(k), Π_[k=1,n]a(k) (← "Σ"は「しぐま」,"Π"は「ぱい」で変換可.)
 ●極限:lim_[x→∞]f(x) (← "∞"は「むげんだい」で変換可.)

■その他
 ●図形:"△"は「さんかく」,"∠"は「かく」,"⊥"は「すいちょく」,"≡"は「ごうどう」,"∽"は「きごう」で変換可.
 ●論理・集合:"⇔⇒∀∃∧∨¬∈∋⊆⊇⊂⊃∪∩"は「きごう」で変換可.
 ●等号・不等号:"≠≒≦≧≪≫"は「きごう」で変換可.

※ ここで挙げた表記法は1例であり,標準的な表記法からそうでないものまで含まれているので,後者の場合使う時にあらかじめことわっておいたほうがいい.
※ 関数等の変数表示や式の括弧は,括弧()だけでなく[]{}を適当に組み合わせると見やすい場合がある.
※ 上記のほとんどの数学記号や上記以外の数学記号は大体「きごう」で順次変換できる.
6大学への名無しさん:04/09/25 21:15:21 ID:oii3fBhW
前スレの997でa/√3・(1,√2)の部分がいまいちわからないのですが
教えてください。ちなみに数学板行ってないのでコピペではないんですが
同じ質問があったらすいません
7大学への名無しさん:04/09/25 21:23:53 ID:/s0icVj2
数学の漸化式の問題です。お願いしますm(__)m

a(1)=2
           
a(n+1)−a(n)=−1/n(n+2)

一般項を求めよ。
8大学への名無しさん:04/09/25 21:27:02 ID:zlzJ9w3B
998 名前:大学への名無しさん[] 投稿日:04/09/25 17:07:06 ID:/FdbEWJu
>>997
傾きルート2かつ長さa
9大学への名無しさん:04/09/25 21:30:31 ID:zlzJ9w3B
-1/{n(n+1)}=1/(n+1) - 1/n
a_(n+1) - 1/(n+1)= a_n - 1/n
あとは分かるだろ。
10大学への名無しさん:04/09/25 21:38:34 ID:xYYt9LLR
他スレでも聞いているのですが、質問させてください。

現在二浪目の文系のものなんですが、
センター数学科目の中に「情報」という強化が入っていたんですが、
これは今から始めても間に合いますか?
また、受験用の参考書や問題集は出版されていますか?
11大学への名無しさん:04/09/25 21:44:50 ID:/s0icVj2
>>9
すみません。そこから先が特にわからないので、教えてくださいm(__)m
12大学への名無しさん:04/09/25 21:45:43 ID:zlzJ9w3B
>>10
取り扱ってるのも有った気がする。
てか、あんまりそれやってる人いないと思うから、
解答は期待しないほうが良いよ。
13大学への名無しさん:04/09/25 21:48:06 ID:zlzJ9w3B
a_(n+1) - 1/(n+1)= a_n - 1/n=b_nとおく。
b_(n+1)=b_n
b_n=b_(n-1)

よって、
b_n=b_1=a_1-1=1
a_n - 1/n=1
a_n=1/n + 1
こんなもの考えれば思いつくはずだ。
思考力をもっとつけよう。
14大学への名無しさん:04/09/25 22:00:40 ID:g11gncZl
揃いも揃って問題読み間違えるなよ。
15大学への名無しさん:04/09/25 22:05:59 ID:/s0icVj2
>>13
ありがとうございます。
でも、問題集の答えは

5/4+【2n+1/2n(n+1)】

になるんですが・・・
16大学への名無しさん:04/09/25 22:11:57 ID:zlzJ9w3B
なに
17大学への名無しさん:04/09/25 22:11:58 ID:EBQsL9z+
a_n
=2+(1/2)*{1/(n+1)+(1/n)-(1/2)-(1/1)}
=(5/4)+(2n+1)/{2n(n+1)}
=(5n^2+9n+2)/{4n(n+1)}
18大学への名無しさん:04/09/25 22:12:38 ID:zlzJ9w3B
-1/{n(n+1)}=1/(n+1) - 1/n
ここで間違えてるジャンw
19大学への名無しさん:04/09/25 22:13:30 ID:zlzJ9w3B
a_n-a_(n-1)
a_(n+1)-a_(n)
をならべて足せば出るな。多分。
20大学への名無しさん:04/09/25 22:17:04 ID:/s0icVj2
>>17
ありがとうございました(>_<)!!
ずっと悩んでたので助かりましたm(__)m
21大学への名無しさん:04/09/25 22:20:02 ID:H0i+QCDT
>>10
情報関係基礎は過去問しかないから、素直に数2Bとったほうがいいよ
22大学への名無しさん:04/09/25 22:25:41 ID:xYYt9LLR
>>12>>21
レスサンクスです。
>>21
一応さっき過去問を覗いてみたんですが、やはり参考書無しでは解けそうもありませんでした。
表計算の問題もあったんですが、エクセルとは少し違うようでした・・・
23大学への名無しさん:04/09/25 22:43:54 ID:4/3kzJLq
>>22
問題キボン
24大学への名無しさん:04/09/25 23:02:32 ID:Fbz1yv7T
この問題の2−(2)を分かりやすく解説してください。

ttp://www.yozemi.ac.jp/nyushi/sokuho01/hokkaido/zenki/index.html
25大学への名無しさん:04/09/25 23:03:17 ID:Fbz1yv7T

文系のほうの2−(2)です
26大学への名無しさん:04/09/25 23:30:37 ID:Z7IXhVew
>>25
(1)が何のためにあるのか考えてみよう
実験させておいて、一般化してるだけだよ
27大学への名無しさん:04/09/25 23:37:59 ID:MMtYBgcM
>>6
PQ↑は傾きが√2だから(1,√2)に平行で、この向きの単位ベクトルは(1/√3)(1,√2)
PQの長さがaだから
PQ↑=(a/√3)(1,√2)

>>22
情報関係基礎は大学や学部によっては選択できないところもあるから注意。

>>24
答えがある。それでわからないならどこがわからないのかわかりやすくかいて。
28大学への名無しさん:04/09/26 00:17:33 ID:TVq6T7eW
>>8>>27
ありがとうございます
2924:04/09/26 00:25:44 ID:qypJi++i
>>26-27

(1)の作業を繰り返してみたら、(2)が分かりました
解答例とは違う解法になったけど、
要は>>26さんの言う通りでした
ありがとうございました
30大学への名無しさん:04/09/26 01:44:20 ID:qm3UY8Ju
同じ形の4個のサイコロを振って出る目の組み合わせは何通りあるか。
という問題が分かりません。同じ形のって言う事は(1,2,3,4)と
(2,1,3,4)などは区別しないってことだから重複組み合わせかな?と
思うのですがどうでしょうか。
31大学への名無しさん:04/09/26 01:48:23 ID:j4qr8oTi
>>30
重複組み合わせでいいよ。
C[9,4]になるかな。
32大学への名無しさん:04/09/26 02:41:36 ID:qm3UY8Ju
>>31
それだと126通りになりますよね?だけど手元の問題集には71通りと
書いてあるんです。どうして??と思うのですが答えしか載ってないんで
どういう考え方で71通りになるのか全く分かりません。僕は>>31さんと
同じように異なる六個の目から重複を許して4個とる組み合わせかな?と
思ったのですが…解答が間違ってるのかな?
33大学への名無しさん:04/09/26 03:01:42 ID:j4qr8oTi
>>32
そりゃ明らかに解答がおかしいよ。違う問題の答えじゃないの?
34大学への名無しさん:04/09/26 09:53:45 ID:T7E23aHg
任意なn角形上に二点A,Bをとる。このとき
 ・AからBまでの右回りの周に沿った長さと、左回りの周に沿った長さは等しい。
 ・直線ABはn角形の面積を二等分する。
を共に満たすA,Bが存在することを示せ。

シャーペンが動きません。どなたかお願いします。
35大学への名無しさん:04/09/26 10:05:22 ID:KgmbukLb
logの2乗ってどう計算するんですか?
36大学への名無しさん:04/09/26 10:12:41 ID:/QccHh9a
>>35
logを2乗するんじゃないか?
37大学への名無しさん:04/09/26 10:16:03 ID:KgmbukLb
例えばlog2なんかは2乗するといくつになる?
38大学への名無しさん:04/09/26 10:17:15 ID:/QccHh9a
(log2)^2
39大学への名無しさん:04/09/26 10:30:28 ID:KgmbukLb
そうか。分かりました。
40大学への名無しさん:04/09/26 10:34:42 ID:QrMoVeLi
>>34
多角形(閉曲線ならなんでもいい)が凸ならこんな感じ。

周の長さをL、面積をSとしよう。
今、適当に周上に点Oをとって、周上の位置をOから右回りに進んだ長さxで表す。(0≦x≦L)
点Aが位置xにいるとき、1番目の条件をみたすBはx+1/2の位置にいる。
このとき、Aの右側の周と直線ABで囲まれた部分の面積をf(x)とすると、f(x)は明らかにxについて連続。
f(0)=S/2なら問題ないので、今f(0)<S/2とする。
すると、f(0)+f(L/2)=Sだから、f(L/2)>S/2。
よって、中間値の定理より、0<x<L/2で、f(x)=S/2となるxが存在する。

凸じゃない場合も含めるともう少し工夫がいりそう。
41大学への名無しさん:04/09/26 10:45:38 ID:dKnU5sPm
(log[a,x])'=(logx/loga)・・・A
ここで(logx)'=1/xだから
(1/loga)・(1/x)=1/xloga
とあるのですが、
(u/v)'=(u'v-uv')/v^2の公式をAで使う事は出来ないのでしょうか?
(loga/x-logx/a)/(loga)^2
=(aloga-xlogx)/ax(loga)^2
ここからどうすれば1/xlogaにたどり着けるのか分からない・・・。
42大学への名無しさん:04/09/26 10:48:46 ID:QrMoVeLi
>>41
logaは定数だから、xで微分したら0だよ。
43大学への名無しさん:04/09/26 10:52:12 ID:dKnU5sPm
あー!
logx/aとしているところが0になって(loga/x)/(loga)^2=1/xlogaですね。
レスありがとうございました。
44大学への名無しさん:04/09/26 10:56:42 ID:Bdw88Fb3
>>41
そこでその公式は意味ないんだけど・・・

v=loga=定数
v'=0

(u/v)'
=(u'v-uv')/v^2
=(u'v)/v^2
=u'/v

定数vはそのまま係数として外に出ただけ
45大学への名無しさん:04/09/26 12:08:56 ID:1st1aWfn
ヘボい問題失礼します。一年です。
「箱の中に1から10までの10枚の番号札が入っている。この箱の中から3枚の番号札を一度に取り出す。
最大の番号が8以上で、最小の番号が2以下である確立を求めよ。」

お願いします
46大学への名無しさん:04/09/26 12:19:49 ID:NoOXRlpe
ここは宿題を代わりにやるスレではないのだが
47大学への名無しさん:04/09/26 12:30:33 ID:1st1aWfn
一応受験意識してやってるんですが
48大学への名無しさん:04/09/26 12:35:38 ID:QrMoVeLi
>>45
全部で選び方が何通りか?
最大が8以上で、最小が2以下の選び方が何通りか?

それぞれ求めて割り算。

下の方は最大10、最小1なら間は2〜9まで何でもいいから8通り。最大9、最小1の場合は…って考えていって足せばいい。
49大学への名無しさん:04/09/26 13:54:17 ID:Mhc56If6
ジョーカーを除いたトランプ52枚の中から1枚のカードを抜き出し、
表を見ないで箱の中にしまった。
そして、残りのカードをよく切ってから3枚抜き出したところ、
3枚ともダイアであった。
このとき、箱の中のカードがダイヤである確率はいくらか。

答えが1/4になる理由がわかりません
50大学への名無しさん:04/09/26 14:04:21 ID:Rr1F51pA
>>49
よくわからん問題だが
>そして、残りのカードをよく切ってから3枚抜き出したところ、
3枚ともダイアであった。

この部分を考えなければ単純に52枚のなかから1枚を抜き出したことになる
よって答えは1/4 ってことじゃないのかな?
引用した部分には既に箱に入れた1枚とは関係ない独立試行だし
51大学への名無しさん:04/09/26 14:41:52 ID:BJXAmlU6
再来年のセンター数学は

数TA、数UBとも、内容(三角比、数列、幾何など)の変更はありますか?
複素数平面がなくなることは知っているんですが、他の内容が知りたいです。
現役3年生ですが、浪人も視野に入れています。

宜しくお願いします。
52大学への名無しさん:04/09/26 14:58:52 ID:NoOXRlpe
>>49
>そして、残りのカードをよく切ってから3枚抜き出したところ、
"よく切って"なら確率は1/4じゃなくないか?

「よくきって(ランダムに)12枚抜き出したら全部ダイヤだった」って言ったとき箱の中のカードがダイヤである確率は低いよね。
それに対し「第3者が表をみながらダイヤを12枚選んだ」ってなら1/4のまま。

もちろん13枚ダイヤを抜き出したら確率はどちらの場合も0だわな。
53大学への名無しさん:04/09/26 15:29:27 ID:aNXJN7Ln
なんで後の動作によって、それより前に行う動作の確率が変わるんだ。
54大学への名無しさん:04/09/26 15:37:19 ID:OXPyl2Xz
>>53
後から13枚抜き出しても1/4ですか?マリックさんならわかるけど…
55大学への名無しさん:04/09/26 15:47:21 ID:aNXJN7Ln
後に何しようが、52枚から1枚引いてダイヤである確率は1/4だよ。
56大学への名無しさん:04/09/26 15:50:57 ID:aNXJN7Ln
あぁ、後からダイヤ13枚取り出せたら話は変わるだろうけど、>>52の問題では少なくともね。
57大学への名無しさん:04/09/26 15:53:57 ID:OXPyl2Xz
>>55
問題読んだ?1枚だけでなく4枚引いてるんだよ?
箱に入れないで4回戻さずにカードを引くって考えればわかりやすいかも。
「4回連続でダイヤを引く確率」と「1回目がダイヤ以外で2〜4回目がダイヤである確率」
どちらが高いか考えてみるべし!
58大学への名無しさん:04/09/26 16:13:12 ID:/QccHh9a
>>57
最初の1枚がダイヤの確率は1/4だろ。。。
59大学への名無しさん:04/09/26 16:44:55 ID:OXPyl2Xz
>>58
"戻さず"にって言ってるのに_| ̄|○

じゃもっと単純に考えよう。
ハート、ダイヤ、クローバー、スペードが1枚ずつ計4枚あります。
この4枚を戻さずに3回ひいた。1枚目は裏のままで2枚目はハート、3枚目はスペードでした。

1枚目がダイヤである確率は?1枚目がクローバーである確率は?1枚目がハートである確率は?1枚目がスペードである確率は?
それぞれの確率を全て足したらいくつ?
60大学への名無しさん:04/09/26 16:46:30 ID:7HikUUhI
>>49
こちらのスレなどで昔から散々ガイシュツの問題
昔の某大学の入試問題で
http://news13.2ch.net/test/read.cgi/news/1096099922/l50

条件付確率の問題として解くと答えは1/4ではないのだが
どう考えても1/4だろって意見もあり不毛な議論が続いてる。
多分2ちゃんではどこで聞いても両方の意見が出て収拾が付かなくなると思うから
ちゃんとした答えが欲しければガッコの先生に聞くのをお勧めする。
61大学への名無しさん:04/09/26 16:54:00 ID:/QccHh9a
>>59
結局1/4だよ。他の取ったカードが何であろうが「1毎目がダイヤである確率」は1/4。

つまり、2枚目3枚目で何を取っていようが、それを考慮する「1枚目がダイヤ2枚目が…」という問題でない限り、1/4。

この問題は後にカードを取り、そのカードを見ているが、そのカードの確率が考慮されない限り1/4。

カードがダイヤであるとか無いとか断定出来る状況は覗いている。
62大学への名無しさん:04/09/26 17:00:31 ID:/QccHh9a
あ違う
63大学への名無しさん:04/09/26 17:04:47 ID:/QccHh9a
後に抜いたカードで無いことが分かるから
52枚のカードに対し、ダイヤでない2枚を抜くと

13/50

m枚取ってダイヤがn枚出てたら

(13-n)/(52-m)

これで13枚ダイヤが出た時に確率が0になるし、ダイヤ以外の39枚を取った時に確率が1になるし、整合性が出るな。

よしよし。
64大学への名無しさん:04/09/26 17:08:58 ID:/QccHh9a
52→51
65大学への名無しさん:04/09/26 18:12:13 ID:aYoFSJmc
1のn乗根の問題って数Bの範囲でしたっけ?
66大学への名無しさん:04/09/26 18:15:04 ID:roV+RbVZ
>>65
そう
複素数の範囲
67大学への名無しさん:04/09/26 18:25:56 ID:OXPyl2Xz
>>65
のう
指数関数の範囲
68大学への名無しさん:04/09/26 19:53:59 ID:EhoAfBx2
eの定義についてなんですが、

lim_[h→±∞](1+1/h)^h=e

どうしてマイナス無限大でもOKなのか、がわからないのでお願いします。
69大学への名無しさん:04/09/26 20:16:26 ID:Mb5jr31q
>>68
1/h=t とおいてみよう。
70大学への名無しさん:04/09/26 20:39:34 ID:50DWnElY
>>65
1のn乗根という言葉を習うのは指数関数の範囲。
その値を複素数の範囲で求めろってのは複素数平面の範囲。
71大学への名無しさん:04/09/26 20:44:43 ID:Bt1m8UZP
1のn乗根って入試にででるっけ?
72大学への名無しさん:04/09/26 21:27:49 ID:Bdw88Fb3
>>68
(1+1/h)^h→e (h→∞)を許すと

(1+1/h)^h
={1-1/(h+1)}*{1-1/(h+1)}^{-(h+1)}
→1*e (h→-∞)
73大学への名無しさん:04/09/26 23:42:33 ID:HaWWSwn2
A(1,1,1)、B(3,4,2)、C(2,3,2)の時
ABベクトルのACベクトルへの正射影ベクトルを求める場合どのようにして求めればいいのでしょうか?
74大学への名無しさん:04/09/26 23:43:54 ID:/QccHh9a
>>73
内積
75大学への名無しさん:04/09/26 23:45:14 ID:c7sLcVHc
>>73
(|AB↑|/|AC↑|)AC↑
=(√14/√6)AC↑
76大学への名無しさん:04/09/26 23:48:46 ID:c7sLcVHc
>>73
ごめん>>75は間違い
AB↑・AC↑)/(|AB↑||AC↑|)AC↑
77大学への名無しさん:04/09/26 23:53:26 ID:HaWWSwn2
>>76
2ベクトルのなす角のcosにAC↑をかけるってことなんでしょうか?
答えは(3/2,3,3/2)なんですが合わないんです
78大学への名無しさん:04/09/27 00:03:54 ID:RClZSZNB
>>73
AB↑・AC↑/AC^2*AC↑
=9/6 * AC↑
=答え
79大学への名無しさん:04/09/27 00:04:24 ID:iA36dNcC
>>76も違ってる
ACベクトルと同じ向きで大きさが|AB↑|cosθ
(AB↑・AC↑)/(|AC↑|^2)AC↑
=(3/2)AC↑
が正解
80大学への名無しさん:04/09/27 00:06:09 ID:RClZSZNB
幾何的な意味としては成す角のcosに長さABを掛け、投射した時の長さを出し、投射する対象のベクトルの単位ベクトル(向きが同じで長さ1)に掛けるといったこと。
81大学への名無しさん:04/09/27 00:07:35 ID:RClZSZNB
このぐらいなら初頭幾何で出せそうだな。
82大学への名無しさん:04/09/27 00:25:55 ID:FNMFDM2y
大体理解できました。単位ベクトルをかけるのはどうしてなんでしょうか?
83大学への名無しさん:04/09/27 00:38:35 ID:12kuatra
>>82
長さを合わせるため。
84大学への名無しさん:04/09/27 00:48:24 ID:d6/BwMWF
a(1)=4、a(n+1)={a(n)+9/a(n)}/2 (n=1,2,3,4,5,...)によって定義されている数列{a(n)}がある。

(1)   a(n)>3 (n=1,2,3,..)を証明せよ。
(2)   a(n+1)-3<{a(n)-3}/2 (n=1,2,3,4...)を証明せよ。
という問題で(1),(2)はとけたんですが、
(3) a(n)<3+(1/2)^(n-1) (n=2,3,4,5,...)

が解けませんどなたか助けてください!!よろしくお願いします!!
85大学への名無しさん:04/09/27 00:59:50 ID:12kuatra
>>84
(2)よりn≧2のとき
a(n)-3
<{a(n-1)-3}/2
<{a(n-2)-3}/2^2

<{a(1)-3}/2^(n-1)
=(1/2)^(n-1)
よって
a(n)<3+(1/2)^(n-1)
86大学への名無しさん:04/09/27 01:48:58 ID:JlQz7EIL
xはaで割ると余りが1になる数とします。

xを何乗してもaで割ると余りが1になるのはなぜですか?

おねがいしまつ
87大学への名無しさん:04/09/27 01:51:33 ID:57c8Da0G
<{a(n-1)-3}/2
<{a(n-2)-3}/2^2

すみません
ここの変化かよくわかりません。
88大学への名無しさん:04/09/27 01:52:56 ID:x+KqszfJ
単純計算が一番ダメです。ヘボイ質問ですがどうかお願いします

x^2+y^2=(x-63)^2+y^2=(x-15)^2+(y-20)^2
の簡潔な解きかたを教えてください。
解は x=63/2 y=(-8)
なんですが、どうにもそうなりません。
89大学への名無しさん:04/09/27 01:56:05 ID:RClZSZNB
>>86
x≡1 (mod a)
x*x≡1*1 (mod a)
つまり、
x=ka+1とおくと
x^2=a(k^2a+2k)+1
90大学への名無しさん:04/09/27 01:57:24 ID:RClZSZNB
>>88
君がやってる計算手順を書いて
91大学への名無しさん:04/09/27 01:58:53 ID:57c8Da0G
>>85

<{a(n-1)-3}/2
<{a(n-2)-3}/2^2

すみません
ここの変化かよくわかりません。
92大学への名無しさん:04/09/27 02:02:34 ID:RClZSZNB
a_n-3=b_nとおけば
∀n∈N b_(n+1)<b_n
だから
93大学への名無しさん:04/09/27 02:03:06 ID:RClZSZNB
訂正
a_n-3=b_nとおけば(2)より
∀n∈N b_(n+1)<(b_n)/2
だから
94大学への名無しさん:04/09/27 02:08:27 ID:pDMzCVNQ
>>91
a(n+1)-3<{a(n)-3}/2
だから
a(n-1)-3<{a(n-2)-3}/2
でしょ。
これを
{a(n-1)-3}/2
に代入すれば不等式が成り立つ。
95大学への名無しさん:04/09/27 02:09:10 ID:x+KqszfJ
>90
展開して代入して…をやってる内にごちゃごちゃになってしまうんです…
9686:04/09/27 02:10:22 ID:JlQz7EIL
>>89
x≡1 (mod a)
は分かるのですが、それの
x*x≡1*1 (mod a)
への変形がさっぱり分かりません。

しつこくてごめんなさい。
97大学への名無しさん:04/09/27 02:14:01 ID:57c8Da0G
>>93、94
おかげ様でわかりました!!これで安心して眠れます!!
ありがとうございました!!!!
98大学への名無しさん:04/09/27 02:16:02 ID:pDMzCVNQ
>>96
p≡q,r≡s(mod a)のときpr≡qsが成り立つ。
p=r=x,q=s=1とすればOK
99大学への名無しさん:04/09/27 02:16:20 ID:RClZSZNB
左2辺を展開整理して
2*63x=63^2
x=63/2
最右辺と最左辺を展開整理して
30x+40y=225+400
xの値を代入して、
40y=625-63*15=625-945=-320
よって
y=-8
100大学への名無しさん:04/09/27 02:18:19 ID:RClZSZNB
>>96

>>98なんだけど、それで分からなければ>>89の後半みたいなことが起こってると思えば良いよ。
101大学への名無しさん:04/09/27 02:29:14 ID:JlQz7EIL
>>98,>>100
分かりました!modの式の変形(modの定理?)が納得できない思いがありましたが、今は納得できました。
どうもありがとうございます。
102大学への名無しさん:04/09/27 02:29:27 ID:x+KqszfJ
>99
ありがとうございます
やはり途中の単純計算で間違っていたようです・・
103大学への名無しさん:04/09/27 08:48:43 ID:d6+qcDkY
上で話題になっている確率の問題について。
大数'92.11臨時増刊からの抜粋

…あるいは、もっとアッサリと、52本中に13本のアタリがあるクジビキで、
2〜4番の人がアタリを引いたとして、1番目の人がアタリを引く確率を求め
てもよい。クジビキは神様の順列で、2〜4番の人が引いたアタリ3本を除い
た残りの49本のどれを1番の人が引いたかは、同じ程度に確からしい。そし
て、49本中のアタリは13-3=10本である。
 よって、求める確率は、明らかに10/49である。…

幾つか解答が載っているけど、一番面白そうな解答を選んでみた。
ちなみに、1976年の早稲田の問題だそうだ。
104大学への名無しさん:04/09/27 10:00:52 ID:8nR6pJkp
y=log|2x-5|を微分する。
u=2x-5とおけばy=log|u|
dy/dx=(dy/du)・(du/dx)=(1/u)・2
とあるのですが、(1/|u|)・2ではないのでしょうか?
105大学への名無しさん:04/09/27 10:41:39 ID:S6zGHKvi
>>104
一般に log|x| を x で微分すると 1/x ですが何か?
不安なら自分でちゃんと証明すれば良いだろ。
x>0 のとき
(logx)'=1/x
x<0 のとき
(log(-x))'={1/(-x)}*(-x)'
=(-1/x)*(-1)
=1/x
106大学への名無しさん:04/09/27 18:33:40 ID:Ij6ToAbb
UBの青チャートを買いに行って間違えて新課程を買ってしまったのですが、新課程はどこが新しくなっているのですか?そしてこれをこのまま使っていても大丈夫でしょうか?
107大学への名無しさん:04/09/27 18:35:09 ID:RClZSZNB
返品してこれば?
108大学への名無しさん:04/09/27 20:43:30 ID:x/cjsPSR
xyz空間で点と平面じゃなくて、点と直線の距離を求める公式ってありますか?
109大学への名無しさん:04/09/27 21:16:58 ID:zpi4OFQ8
あれを公式というのかどうかわからんけど
点と平面の距離と同じじゃないの?
110大学への名無しさん:04/09/27 21:37:01 ID:+LugTO9f
>>108
外積を使えばある。
111大学への名無しさん:04/09/27 21:37:43 ID:x/cjsPSR
でも、空間での直線の方程式は=が二ついるじゃない?
どうやって代入するの?
112大学への名無しさん:04/09/27 21:39:49 ID:x/cjsPSR
>110どういう奴ですか?
113大学への名無しさん:04/09/27 22:13:42 ID:+LugTO9f
空間内の直線が点A(a↓)を通り、方向ベクトルがd↓とすると
点と直線の距離は

|(a↓)X(d↓)|/|d↓|
114大学への名無しさん:04/09/27 22:47:26 ID:x/cjsPSR
>>113
イメージ的には

距離=(平行四辺形の面積)/(底辺)

みたいな感じですか?
115大学への名無しさん:04/09/27 22:51:01 ID:+LugTO9f
>>114
そうです。
さっきの式は原点との距離でした。
原点以外の点との場合は適当に修正してください。
116大学への名無しさん:04/09/27 23:15:23 ID:x/cjsPSR
>>115

ありがとうございました。
117大学への名無しさん:04/09/28 07:04:16 ID:tgOE1q6x
や、めっちゃどうでもいい事なんだけど、ド忘れしちまったんで。

判別式作るときにさ、ax^2-(b+1)x+cって、xにかけてある数がマイナスだったら、

D=-(b-1)^2-4acってなるんだっけ? 最初にマイナス付くか付かないか微妙…。orz
118大学への名無しさん:04/09/28 07:35:45 ID:W+U8N8Br
それはど忘れじゃなくて、理解していないんじゃないか?
ax^2+bx+c=0
だったら、D=b^2-4acだ。
bが-(b+1)に変わったらどうなるんだ?
119大学への名無しさん:04/09/28 10:17:53 ID:hBkX0ccS
>>118
>>117は釣りだろうさすがに。
120大学への名無しさん:04/09/28 11:14:54 ID:98D+bHF/
y=x√(x^2+1)+log(x+√(x^2+1))
これを微分する問題なのですが、どうしても答えと合いません。
x√(x^2+1)=x(x^2+1)^(1/2)
この式を微分すると(2x^2+1)/√(x^2+1)
log(x+√(x^2+1))を微分すると
(1+1/2(x^2+1)^(-1/2)・2x)/(x+√(x^2+1))
=(1+x/√(x^2+1))/(x+√(x^2+1))
となって、この二つを足しても解答の2√(x^2+1)になりません。
どこで間違えているのでしょうか。
121大学への名無しさん:04/09/28 11:30:49 ID:jPlcL3Na
>>117
a が a , b が -(b-1) , c が c なので
D={-(b-1)}^2-4ac です。

>>120
>この二つを足しても解答の2√(x^2+1)になりません。
ここで間違えています。
122大学への名無しさん:04/09/28 12:28:07 ID:98D+bHF/
解けません・・・計算手順を教えて下さい。
123大学への名無しさん:04/09/28 12:35:13 ID:a3iEMaAx
y=x√(x^2+1)+log(x+√(x^2+1))
y'=√(x^2+1)+x*x/√(x^2+1)+{1+x/√(x^2+1)}/(x+√(x^2+1))
=(2x^2+1)/√(x^2+1)+(x+√(x^2+1))/{(x+√(x^2+1))√(x^2+1)}
=(2x^2+2)/√(x^2+1)
=2√(x^2+1)
124大学への名無しさん:04/09/28 12:35:56 ID:a3iEMaAx
>>122
自分で信頼できるだけの自分の計算力をつけよ。
125大学への名無しさん:04/09/28 12:56:01 ID:98D+bHF/
>>123
ありがとうございました。
3行目までは計算できたんですが、
そのあと
(x+√(x^2+1))/{(x+√(x^2+1))√(x^2+1)}

1/√(x^2+1)
になるのがわかりませんでした_no
126大学への名無しさん:04/09/28 13:55:10 ID:e9DK0fDf
教えてください。。。
正方形を縦横それぞれ8等分した東西9条、南北9条の街路がある。
南西の端から東北の端へ遠回りをしないで行く道筋の中で、
点対称なもの、線対称なものは、それぞれ何通りあるか。
お願いします。
127大学への名無しさん:04/09/28 14:06:07 ID:jPlcL3Na
>>126
点対称:ド真ん中の点に関して点対称ってことね。正方形を縦横4分割して考える。(田の字型に)
左上の部分を通るとしたら点対称な右下の部分も通らなければならないが、
最短距離を通るとしたら左上と右下を両方通るということはありえない。
したがってド真ん中の点を通ることになる。
また左下の部分の通り方を1つ決めればそれに対応して点対称に右上の部分の通り方も1つに決まるから
左下の5×5の道の部分の通り方だけを数えればよい。
128126:04/09/28 14:13:21 ID:e9DK0fDf
>>127
すばやいレス有り難うございます!
理解できました。
線対称の場合はどうなるのでしょうか。。。
129大学への名無しさん:04/09/28 14:24:10 ID:jPlcL3Na
>>126
線対称:まず正方形の対称の軸は縦横十字の2本と斜めの対角線2本の計4本である。
縦の軸を対称の軸とすると、南西の頂点に対応するのは南東の頂点、北東の頂点に対応するのは北西の頂点だが
最短距離で行くためには南東の頂点と北西の頂点を同時には通れない。
したがって縦の軸は対称の軸にはならない。横の軸も同様。
斜めの2本の軸のうち南西と北東の頂点を結んだ線が対称の軸のときについては
南西の点から1歩右に進んだとすると南西の点より1つ上の点も通らなければならないが、これはまずい。
最初1歩上に進んだとしても今度は南西の点の1つ右の点も通らなければならなくなり、まずい。
したがって、対称の軸としてありえるのは北西と南東を結んだ対角線のみである。

このとき対称の軸上に9つの点があるが、この対角線上の点に到達するまでの通り方を1つ決めれば残りの通り方も対称に1つに決まるので
この9つの点それぞれについて到達のしかたを何通りか計算して、全部加えればよい
(9つのうちどの点も同時に通ることはないので普通に足せる)

ちなみに、東北じゃなくて北東だと思うぞ。
130126:04/09/28 14:35:12 ID:e9DK0fDf
>>129
有り難うございます!
丁寧に書いてくれたので私にも理解できました。
>ちなみに、東北じゃなくて北東だと思うぞ。
そうですね。。。(^^;
131大学への名無しさん:04/09/29 00:32:55 ID:EbdkIvBL
私が出そうとした問題がそのまんま載ってた
ありがとう >>126 & >>129
132大学への名無しさん:04/09/29 18:02:40 ID:KOpgsqI3
問題に関する質問というか、答えに関する質問です。

問題を解き終わった後、(証明終)や(答)なんかを最後に書き忘れてしまうんですが、
この、最後の記述は絶対書かなければいけないのでしょうか?
133大学への名無しさん:04/09/29 18:07:10 ID:m8iMfXkH
絶対ってことはないけど、
どの部分が最終結果なのかを採点者に伝えるのは大事。
答のところには下線をひく、というのはその手段のひとつ。
134大学への名無しさん:04/09/29 18:09:33 ID:qbGn7OBP
>>132
分かるように示していれば○
証明終わりにはQ.E.Dという書き方もある。
Quod Erat Demonstrundum(書く示された)
の略で、オレは多用する。
135大学への名無しさん:04/09/29 18:14:21 ID:qoSftb2k
((1+h)^n)/n の極限を求めろって問題です。

解説に、(1+h)^n > 1+nh+(n(n+1)h^2)/2 より・・・・


と、あります。
おそらく二項定理からでしょう。
この場合、 (1+h)^n > nh だけでも、問題解けるんですが、
(1+h)^n > nh だけじゃあダメなんですか? 弱いんですかね?
136大学への名無しさん:04/09/29 18:15:33 ID:KOpgsqI3
>>133-134
なるほど、確かにちゃんと確実に分かるように示さないといけませんねー、、
ありがとうございました。
137大学への名無しさん:04/09/29 19:14:49 ID:qbGn7OBP
>>135
その部分が2項定理だとわかるように書いたのかな。
138大学への名無しさん:04/09/29 20:11:29 ID:OtwU+9yx
>>135
(1+h)^n>nh だけでも解けるのならばかまわない。
それでダメなのか、弱いのかを判断するためには問題を書いてもらう必要がある。
「((1+h)^n)/n の極限を求めろ」では問題として不充分であるのは間違いない。
たとえば、h>0 , n は自然数で、n→∞ での極限を求めるのであればその条件では不充分に思われる。
139大学への名無しさん:04/09/29 20:13:34 ID:qbGn7OBP
>>138
あぁ、そうだね。。。
140大学への名無しさん:04/09/29 20:37:37 ID:j1e/ILrG
三角形OABの辺OAを1:2の比に内分する点をM、
辺OBを2:5の比に内分する点をNとし、
線分AN、BMの交点をCとする。また、点PはOP↑=s(OA↑)+t(OB↑)
(s,tは実数)と表される点とする。
(1)OP↑=3s(OM↑)+t(OB↑)だから、点Pが直線BM上にあるための条件は
  □s+t=□である。
(2)点Pが線分AN上にあるための条件は、□s+□t=2 かつ □≦s≦□である。
(3)点Pが点Cと一致するとき s=□/□□,t=□/□□ であり、三角形ABC の面積
は三角形OABの面積の□□/□□倍である。

最初のOP↑=3s(OM↑)+t(OB↑)はわかりましたが、
あとがぜんぜんわかりませんでした・・・。
宜しくお願いします。
141大学への名無しさん:04/09/29 21:16:34 ID:OI3BoR70
st平面上において、(1,0)にAを、(0,1)にBをとると
Mは(1/3,0)、Nは(0,2/7)となる。
直線ANの式は2S+7t=2,AN上にPがあるためには0≦s≦1
(3)の前半は直線ANと直線BMの交点を求めるだけ
142大学への名無しさん:04/09/29 21:29:05 ID:j1e/ILrG
OP↑=□s(OM↑)+t(OB↑)

OP↑=3s(OM↑)+t(OB↑)
この部分が分かっただけです。
説明不足すいませんでした・・・。
宜しくお願いします。
143大学への名無しさん:04/09/29 21:39:54 ID:OI3BoR70
>>142
一応>>141>>140へのレスなんだが
144大学への名無しさん:04/09/29 22:11:40 ID:fG2CNBNr
>>140
2点A,Bを通る直線:
OP↑=sOA↑+tOB↑ t+s=1

ちゃんと教科書のベクトル方程式の所に書いてある。
これを使うだけで解ける問題。
最後の面積の問題は、直線OCとABの交点をDとすると、DC:DOが高さの比になる。
145大学への名無しさん:04/09/29 22:16:30 ID:j1e/ILrG
>>141
st平面上ってことは、座標において考えるということですか??
もう少し詳しく教えていただけないでしょうか?
宜しくお願いします。
146大学への名無しさん:04/09/29 23:46:06 ID:OI3BoR70
>>145
教科書に載っているようにやってもいいし、
OA↑、OB↑は1次独立なのでOA↑、OB↑を基底にして
st平面を考えると簡単な直線の式を求める問題になる。
147大学への名無しさん:04/09/29 23:50:12 ID:OtwU+9yx
>>145
受験用には、>>144をお勧めします。てか教科書読もうな。
148大学への名無しさん:04/09/30 01:49:16 ID:/RjuINg7
問「実数aに対して、関数y=-x^2+aのグラフと円x^2+y^2=2を考える。このとき、2つの曲線が接するときのaの値を求めよ。」
というもので解答が
解「y=-x^2+a…@ x^2+y^2=2・・・A
@とAを連立した y^2-y+a-2=0・・・B
1)@が重解をもつとき y-a=0
これをBに代入 a=√2,-√2
2)Bが重解をもつとき a=7/4」
となっていました。僕は図で場合わけをして解いたのですが、解答の「1)@が重解をもつとき y-a=0」というところの意味がさっぱりわかりません。何のためにしてるのでしょう?教えてください。
149大学への名無しさん:04/09/30 02:25:03 ID:ItuamNBk
判別式
150大学への名無しさん:04/09/30 02:28:50 ID:ZZ8l1JOJ
>>148
その解答って板書写したりしたやつ?
それとも書籍か何かの解答?
明らかに解答が間違ってると思う。

y^2-y+a-2=0・・・B
この式のyが重解を持てばいいから
9-4a=0
∴a=9/4
これが解答だと思うけど。。
151大学への名無しさん:04/09/30 02:36:50 ID:/RjuINg7
>>150
それは違うってか、確かに解答は答えは合ってるよ。一般に曲線と曲線では接することと重解をもつことは同じとは限らないから。
152大学への名無しさん:04/09/30 02:37:35 ID:/RjuINg7
>>149
何のための判別式??
153150:04/09/30 02:37:45 ID:ZZ8l1JOJ
ああ。もう一つの場合分けもしないといけないな。。
150の解等に付け加えて
a=+√2、-√2
うっかりしてた
154大学への名無しさん:04/09/30 02:39:22 ID:/RjuINg7
>>151
確かに148は計算ミスはしてるが。
155大学への名無しさん:04/09/30 02:41:08 ID:/RjuINg7
図書くと簡単だけど、この解答はさっぱり。ちなみに、出典は塾テキストの解答。どっかの業者作成。
156大学への名無しさん:04/09/30 02:42:39 ID:/RjuINg7
>>153
そのa=√2,-√2の解答の導き方がなぞ。
157バカでスマソ:04/09/30 02:43:07 ID:IvlHb6MZ
x^2−2px+1=0の1つの解が3より大きく
他の解が3より小さくなるときの定数pの値の範囲はp>□である

って問題なんすけど答えは5/3になるのですが
途中の過程が分からないので教えてください。。
158大学への名無しさん:04/09/30 02:46:13 ID:/RjuINg7
>>157
解の公式で-のほうを3以下として、大きいほうが3以上って式をとけばでないの?
159大学への名無しさん:04/09/30 02:49:03 ID:/RjuINg7
>>157
もっと簡単だった。x=3代入して負になればいいじゃん
160150:04/09/30 02:52:12 ID:ZZ8l1JOJ
>>156
a=+√2、-√2
これは放物線が円の一番上と、一番下と
それぞれ接するときの条件。
導き方というか>>151の言うとおり曲線同士が接する条件は単に解の個数では議論できないから、
幾何的に場合分けするしかないと思う。
この場合は円にかぶさるようにか、円の一番上、一番下にそれぞれ接する場合があるから、
それぞれに的を絞って計算するしかないんじゃない?
だれか補足お願い。。
161バカでスマソ:04/09/30 02:52:36 ID:IvlHb6MZ
>>159
サンクス、あんた天才だ!!
162大学への名無しさん:04/09/30 03:05:46 ID:/RjuINg7
>>156
Bはxを消去してるから、円を関数として考えるためにy方向に半分にして考えてることになるから
端点は別の方法じゃないと出ない。で、多分1)は、yを定数(横線)と考えて、一度直線に重解条件を移して、改めてBの式で考えてるのかな。
どのみち、わかりづらい解法だ。
163大学への名無しさん:04/09/30 03:23:00 ID:ZZ8l1JOJ
>>148
円の一番上、一番下の部分と放物線が接する場合分けをした上で、
このときx=0は明らかだからこれを@に代入してy-a=0
っていう式が出たんじゃない?
164大学への名無しさん:04/09/30 03:25:04 ID:/RjuINg7
でもそれだと重解って書いてる意味がなくない??
165大学への名無しさん:04/09/30 03:30:07 ID:ZZ8l1JOJ
というかそもそも@は関数であって方程式ではないから重解云々の議論が出来ないはずだけど。。
166大学への名無しさん:04/09/30 03:31:57 ID:/RjuINg7
だからY=yみたいな定数関数との重解みたいに捕らえてるんじゃないのかな。>>162みたいな。
167大学への名無しさん:04/09/30 03:51:28 ID:ZZ8l1JOJ
やっぱり定数関数としてみたとして、
仮に判別式から重解を持つ条件としてy-a=0
が出たとしても、式としてまったく意味のないような。。
それをBに当てはめたところで意味不明な式でしかない。。

>>162
xを消去したところで円を半分にしてることにはならないんじゃない?
端点って言うのは何を指してるの?
やっぱりyを定数とは考えられないと思うんだけど。。
168大学への名無しさん:04/09/30 03:59:31 ID:ItuamNBk
暇だなこんなのどう?
Y=-y+aとおく。
Y=x^2
よって、
Y^2+Y(1-2a)+a^2-2=0かつ
Y≧0
これが重解を持つので、
4a=9
∴a=9/4
また、Y=0の時、1-2a>0の下で、
a^2=2
∴a=-√2
169大学への名無しさん:04/09/30 04:01:47 ID:/RjuINg7
xを消したってことはx=±√(y^2-2)を代入してるから、半分にしてるのと同じだと思う。端点は円を縦割りしたときの上と下。
定数関数としてみたとして、判別式から重解を持つ条件としてy-a=0とすると、・・・の意味は確かにいまいち。
170大学への名無しさん:04/09/30 04:12:26 ID:ItuamNBk
てかこの解答もともとおかしくない?明らかに3点接触してない?a=√2の時。
171大学への名無しさん:04/09/30 04:12:54 ID:ZZ8l1JOJ
x=±√(y^2-2)を代入したんじゃなくて
x^2=a-yだよ
式Bは@、Aの連立式と同値だからBは@、Aの連立式の定義域で成立してるよ。
>>端点は円を縦割りしたときの上と下。
これは具体的に言うと(0,-√2)、(0,√2)の2点のこと?
172大学への名無しさん:04/09/30 04:27:02 ID:ZZ8l1JOJ
>>170
√2のとき確かに3点で交点を持ってはいますがそのうちの一点は接しているので
命題の接するという条件を満たしてるので適当だと思います。
173大学への名無しさん:04/09/30 04:27:21 ID:SpTrnaP7
質問です。
鋭角三角形ABCにおいて、AB上に点D、BC上に点E、CA上に点Fをとる。
DE+EF+FDが最小値を取るとき、AB⊥DC、BC⊥AE、CA⊥BFを示せ。

という問題が分かりません。
174大学への名無しさん:04/09/30 13:58:14 ID:ngi1C4Dn
>>173
座標設定して計算で押していけばできそう
もっと簡単な幾何的な解き方があるんだろうけど
175大学への名無しさん:04/09/30 17:19:43 ID:RcEh7vk+
>>173
例えば、Dを固定したとき、DとBCに対して対称な点をG、DとCAに対して対称な点をHとしたとき、GHとBC,CAとの交点をそれぞれE,FとすればDE+EF+FDが最小になる。

ということを使わないと計算が厳しいかも。
176大学への名無しさん:04/09/30 18:11:17 ID:ibFV7WVM
x軸に平行な直線を微分すると結果は0ですか?
177大学への名無しさん:04/09/30 18:16:29 ID:9SX6jDP6
もちろん
178大学への名無しさん:04/09/30 18:36:12 ID:nHm5Jte7
f(θ)=cos2θ+2sinθ+1の最大値と最小値、そのときθの値をそれぞれ教えてください!
179大学への名無しさん:04/09/30 18:43:44 ID:9SX6jDP6
極大値はθ=π/4のとき1+√2
極小値はθ=3π/2のとき-2
180大学への名無しさん:04/09/30 19:43:06 ID:2bep7PcW
>>176
x 軸に平行な直線によって表される関数は x に関する定数関数となります
x に関する定数関数を x について微分すると 0 になります。
微分の質問をするのであれば、どの関数をどの変数について微分するのかを書きましょう。

>>178
f(θ)=-2sin^2θ+2sinθ+2
t=sinθ とおく (-1≦t≦1)
-2{t-(1/2)}^2+(5/2)
t=1/2 すなわち θ=(1/6)π+2nπ , (5/6)π+2nπ のとき最大値 5/2
t=-1 すなわち θ=(3/2)π+2nπ のとき最小値 -2
181大学への名無しさん:04/09/30 20:04:28 ID:ibFV7WVM
二次曲線上の点(x_1,y_1)における楕円の接線の方程式の証明で
x^2/a^2+y^2/b^2=1
微分して
2x/a^2+(2y/b^2)(dy/dx)=0
dy/dx=(-2x/a^2)・(b^2/2y)=-(xb^2/ya^2)
y=-(x_1b^2/y_1a^2)(x-x_1)+y_1
a^2(yy_1-(y_1)^2)+b^2(xx_1-(x_1)^2)=0
などとやってみましたが解けませんでした。
計算手順を教えて下さい。
182大学への名無しさん:04/09/30 20:07:56 ID:RcEh7vk+
>>181
その計算でできてるよ。
あとは、(x_1,y_1)が楕円上の点であることを使えばいい。
183大学への名無しさん:04/09/30 20:14:07 ID:ibFV7WVM
>>182
解けました!ありがとうございました。
184大学への名無しさん:04/09/30 20:49:41 ID:nHm5Jte7
>>180
ありがとうございました!
185ウィー:04/09/30 21:51:49 ID:vQmfwJcd
△ABCは円Oに内接し,AB=AC=4,BC=3とする。頂点Aを通らない弧BC上を点Pが動く。
(1)BPとCPはxについての2次関数x^2-2xAPcosB+AP^2-16=0の解であることを示しなさい。
(2)BP+CPの最大値を求めなさい。

全然わかんないのでおねがいします。
186大学への名無しさん:04/09/30 22:17:03 ID:1JoWdT2f
>>185
(1)△PBCに余弦定理を適用
(2)解と係数の関係
187大学への名無しさん:04/09/30 22:26:52 ID:eIOqDR/c
細野真宏さんので「〜が面白いほどわかる本」と「〜が本当にわかる本」の二冊がありますが
違いってなんなんですか?どちらか買うつもりなので教えてください!
188聖徳大学:04/09/30 22:27:44 ID:TOgB2vcX
先日、東京聖徳学園の職員採用試験を受けました。
筆記試験のとき、副理事長とかいう奴が出てきて、
自分の話を聞かない志願者に一喝!
でもよく考えてみてよ。まだお宅の職員じゃないよ。お客さんだよ。
それに驚いたのはその職員採用試験に聖徳出身者がかなり多い!
しかも制服着ているということは体育会かな?
とにかく聖徳大学に行っても就職できないってことだよ。
一番驚いたのは面接試験のエントリーシート。
志願者の父親・母親の出身学校を記載させる欄がある!
中途採用なら25歳は過ぎてる人も多いのに…。無意味だし、失礼だ!
(四流大学が志願者の親の学歴を気にしてどうするの?)
形だけ採用試験をやって
本当は就職できない哀れな聖徳大生を救ってるんでしょう。
バカバカしい。
そのエントリーシートには聖徳に知り合いがいるかどうか
詳しく書かせる欄もある。

受験生の皆さん!入試名前を書いたら合格できますが、
馬鹿が運営する四流大学なんかに行ってもしょううがないですよ。
聖徳グッズをつくって学習塾に配り歩いている学校です。
その広告費はどこから出ているのかな?
189大学への名無しさん:04/09/30 22:49:02 ID:vQmfwJcd
>>186
(1)の方詳しくお願いします
190大学への名無しさん:04/09/30 23:03:53 ID:2bep7PcW
>>185
(1)△ABPに余弦定理を適用。
191大学への名無しさん:04/09/30 23:06:44 ID:bk7TXkPV
31+9d<100<31+10dから
69/10<d<69/9
にはどうすればなるんでしょうか
192大学への名無しさん:04/09/30 23:11:54 ID:2bep7PcW
>>191
31+9d<100<31+10d
⇔ 31+9d<100 かつ 100<31+10d
⇔ 9d<69 かつ 69<10d
⇔ d<69/9 かつ 69/10<d
⇔ 69/10<d<69/9
193大学への名無しさん:04/09/30 23:12:56 ID:4fCYNerx
>>191
31+9d<100 かつ
100<31+10d
194大学への名無しさん:04/09/30 23:27:17 ID:TjZ/a6k8
>>185
数学板にマルチしまっくてるな。
195大学への名無しさん:04/09/30 23:30:16 ID:2bep7PcW
>>194
マルチは1ヶ所しか見つけられなかったんだが、そんなにしまくってるのか?
ってかマルチ予備軍相手にまともなレスなんかしてしまって正直スマンカッタ。
196大学への名無しさん:04/10/01 13:57:33 ID:9MVE4HHZ
複利計算についての色々な解説を読んだのですが、さっぱり意味が分かりません。

借入額の完済時における価値=毎回の返済額の完済時における価値の和

という概念が理解できないのです。特に右辺の意味するところがまったく
見当も付かない状態です。イメージできないのです。なぜそんな突拍子の
ないものを出してくるんだという感じです。
197長助:04/10/01 15:54:49 ID:n0u7Hk2M
>>196

例えば、年利が3%である社会においては、現在の100万円は10年後には、
100万円*(1.03)^10≒134万円の価値があると考えます。

一般にお金の価値は時刻の関数なので、2つを比べるときには時刻をそろえる必要があります。
>>196の場合には左辺が完済時なので、右辺も完済時の価値に変換して計算します。


あるいは、右辺は借り入れと同率の利子で積み立てをした合計額と解釈し、それが(利息を含んだ)
借入額と等しくなったときが返済完了である、と説明することもあります。
198大学への名無しさん:04/10/01 16:59:08 ID:Wn+VgG29
微積分基礎の極意の第3章の10問目についての質問なんですが、

問題 半径1の円Cの内部に円Cと接する半径の等しいn個の円A_1、
A_2…、A_nを順に並べる。ただし、隣り合うどの円も互いに外接するものとする。
 このとき、n個の円A_1,A_2…A_nの面積の総和をS_nとして、lim_[n→∞]nS_nを求めよ。


という問題があり、解答にはA_1〜A_nの半径をr_nとすると
2(1-2r_n)<2nr_n<2πが成り立つとあり、ここまでは分かるのですが、
その次に

従ってπ/(n+2π)<r_n<π/n

となっており、何故この式の左の不等号が成り立つのか分かりません。
解法の大まかな流れは分かるのですがこの部分だけどうしても分かりません。
どなたか教えてください。お願いします。
199長助:04/10/01 17:09:44 ID:n0u7Hk2M
>>198
2(1-2r_n)<2nr_n<2π ではなくて、2π(1-2r_n)<2nr_n<2π ではないですか?
200大学への名無しさん:04/10/01 17:11:41 ID:6rq+F/jg
>>198
左辺の不等式を解いただけ
>>1989の式であってる
201大学への名無しさん:04/10/01 17:12:37 ID:Wn+VgG29
>>199
すいません、間違えました。そうです。
202大学への名無しさん:04/10/01 17:15:43 ID:Wn+VgG29
>>200
本当だ。ありがとうございました。
アホな質問してすいません。
203185じゃないが:04/10/01 22:18:47 ID:pG+5PBi+
>>186
(2)は解の係数からどう解くの??
204大学への名無しさん:04/10/01 22:35:29 ID:+K9H9Woj
次の複素数をa+biの形に直せ
(1)z=1+iの時z+z^-1
(2)z=2-iの時z^2

(2)はただ二乗すればいいだけだとは分かるのですが、(1)がよく解りません…
よろしくおねがいします
205大学への名無しさん:04/10/01 22:43:42 ID:br5Sh+0O
1+i=√2(cos45゚+i*sin45゚)
206大学への名無しさん:04/10/02 00:11:55 ID:zR4fuyTB
>>203
解と係数の関係より
BP+CP=2APcosB
APが直径となるとき右辺は最大。
207大学への名無しさん:04/10/02 00:12:53 ID:zR4fuyTB
>>204
z^(-1)=1/z
208大学への名無しさん:04/10/02 00:34:47 ID:wBo3GST8
>>204
1/(1+i)は分母分子に(1−i)をかけて分母を実数化する。
もしくはz*z~=|z|^2=2を用いて
1/z=z~/2
209大学への名無しさん:04/10/02 01:39:26 ID:B/dwGmiq
フーリエ解析の問題集で良いのがあったら紹介してください。
210○○社 ◆XhYsRJwDD2 :04/10/02 01:40:59 ID:Po58b7PU
板違いププ
211大学への名無しさん:04/10/02 01:41:52 ID:xIjolpTA
>>209
線型代数演習
東大出版

線形か線型か。赤と黒の本。
212大学への名無しさん:04/10/02 01:44:09 ID:xIjolpTA
やり方だけなら物理数学なんかの本に載ってる。マクローラン展開は微積や物理数学。
213○○社 ◆XhYsRJwDD2 :04/10/02 01:44:38 ID:Po58b7PU
フリーエ解析って線形代数だったのか?
そういや大学の授業で使ってる線形代数学サイエンス社は激しくムズイし糞プ
214大学への名無しさん:04/10/02 01:51:21 ID:xIjolpTA
あぁ、ごめん。演習の方には載ってなかった。

>>213
結局はベクトルの投影だからな。フーリエ級数における内積の定義と単位ベクトルの考え方が分かればフーリエ級数は理解出来る…
215大学への名無しさん:04/10/02 03:53:54 ID:imfg1+7y
>>208
よく解りました、有難うございました
216大学への名無しさん:04/10/02 14:16:20 ID:uvITT25u
>>197 やっと理解できました。私にとってはこれ以上ないくらい
分かりやすい説明でした。ありがとうございます。
217大学への名無しさん:04/10/02 14:16:50 ID:5ex7XO2v
テンソル積の普遍性について教えてください
218長助:04/10/02 16:23:01 ID:G4Cfws4K
>>217

代数系として何をとるかによって、ステイトメントが多少異なりますが、単純な場合として、

可換環R上の加群L, M, N、双線形写像 F : L×M → N について、
L, Mのテンソル積をM(×)N、標準的な単射をι:L×M → L(×)M とすると、
準同型f :L(×)M → N が一意に存在し、F=f*ι をみたす。

↑の命題を指して普遍性と呼ぶことが多いと思います。
つまり、直積をテンソル積に置き換えることで、双線型を線型に置き換えられますが、
このとき情報は失われない、ということです。
219大学への名無しさん:04/10/02 18:58:58 ID:/7oqbn/1
受験に一〜三次のテーラー展開って必須ですか?計算が楽になるといわれましたが本当ですか?
220大学への名無しさん:04/10/02 21:23:25 ID:xIjolpTA
>>219
化学の計算で1次は使った。
221大学への名無しさん:04/10/02 23:26:16 ID:/7oqbn/1
化学って、使うんだ。因みにどの分野ですか?
VCまでなら三次まではいりますか?
222大学への名無しさん:04/10/02 23:33:49 ID:cHFn6F1Q
というか、知ってると便利だけど、
必要ではない。

数学では3次までの近似から作った問題とか見るけど、
マクローランとかテイラーとか解答用紙でつかっていいかどうかもわからないし、
知ってると、役には立つことは確かだが、
深入りはやめとき。

使った例
√(101)=10√(1+0.01)≒10(1+0.005)
223大学への名無しさん:04/10/02 23:42:26 ID:yRZ6T75w
っていうよりテイラー展開は記述式の答案では大学入試では使っちゃだめだよ。

ところで、微分の公式より、
lim[h→0](f(x+h)-f(x))/h=f'(x)
だから、hが0に近ければ、
f(x+h)≒f(x)+hf'(x)
っていうのは最近の高校の教科書にはないのかな?

これがテイラー展開の1次までの近似。
224大学への名無しさん:04/10/03 00:12:23 ID:5R4Ch/pz
すみませむ
a(n)=(n−1){a(n−1)−a(n−2)}
の漸化式はどうやって解きますか?
225大学への名無しさん:04/10/03 00:19:05 ID:O0AwPNqo
>>223
旧課程なら数Vの教科書にある。
新課程は知らんけど。
226大学への名無しさん:04/10/03 00:30:49 ID:aAPON0gq
>>224
その漸化式を解けって問題なの?
227大学への名無しさん:04/10/03 00:41:00 ID:hJ7QtQFD
187に詳しい人よろしく
228大学への名無しさん:04/10/03 01:11:13 ID:bkPDZ0R1
>>226
並べかたを求める問題です。
229○○社 ◆XhYsRJwDD2 :04/10/03 01:17:55 ID:maQSXWf4
>>224
とりあえず、初項第二項くらいかけ
230大学への名無しさん:04/10/03 01:24:42 ID:rtJ3pOYL
223 確かに教科書には乗ってますが、それを=で結ぶためにはどうしたらいいのですか?因みにe^(i・π)=1はテイラー展開で表せますか?
テイラーがダメならヘロンやロピタル等もダメなのですか?
231○○社 ◆XhYsRJwDD2 :04/10/03 01:30:09 ID:maQSXWf4
e^ix=cosx+isinx

はマクローリン展開じゃね?
ま、マクローリンとテイラーの違いがよくワカラン漏れには関係ない話だが。
232大学への名無しさん:04/10/03 02:08:18 ID:rtJ3pOYL
xではなくπですが、、、。まあ教科書以外は書かないほうがいいみたいかな、、、
233○○社 ◆XhYsRJwDD2 :04/10/03 02:11:34 ID:maQSXWf4
xにπ代入したのがオイラーの公式だろ?
ちなみにe^iπ=-1ね
234大学への名無しさん:04/10/03 02:21:30 ID:rtJ3pOYL
わざわざすいません、、、富豪間違えてた、、、
235大学への名無しさん:04/10/03 02:25:54 ID:aAPON0gq
オイラーの公式はマクローラン展開で出す。ちなみにマクローラン展開∈テイラー展開。
ヘロンの公式は知らないが、極限値を求める問題でロピタルを使うのは基本的に反則。

a<a_n<b
a_n<a_(n+1)
∴a_n→a

というのも無し。
236大学への名無しさん:04/10/03 02:32:06 ID:rtJ3pOYL
ヘロンは三角の面積で三辺がa、b、cのとき面積S=√{s(s-a)(s-b)(s-c)}ただしs=(a+b+c)/2 というやつです。
ということは検算しか使えないということですね
237大学への名無しさん:04/10/03 02:37:20 ID:aAPON0gq
>>236
あぁ、それか。それは確か使えた。基本的に大学レベルのテクニックは検算か時間が無い時の点数もぎ取り作戦にしか使えないと思った方が良い。
238大学への名無しさん:04/10/03 02:47:12 ID:rtJ3pOYL
わかりました。つまり苦肉の策で使うくらいしか役に立たないということですか。ヘロンが使えるなら円に内接する四角形の公式も使えますかね?
239大学への名無しさん:04/10/03 06:57:17 ID:EjV+kyAE
なんか、マクローラン展開って言われると
ローラン展開とマクローリン展開が頭の中で交差するんだが……
240大学への名無しさん:04/10/03 07:01:45 ID:XNYFjTJ7
>>238
トレミーの定理か?
241大学への名無しさん:04/10/03 07:18:30 ID:BizhlErv
>>240たぶんブラマグプタの公式だと思います

>>238
センターなどのマークならいいと思いますがブラマグプタをそのまま記述しては駄目だと思います。
242大学への名無しさん:04/10/03 11:55:07 ID:0HvwvGuD
y=x~3-3x~2+9|x-a|+1

が常に増加aの範囲ってどうやって解いたらいいです?一回微分したらaがなくなってしまうのでわかりません
243大学への名無しさん:04/10/03 12:49:37 ID:kJCr1rNU
>>242 絶対値ついてるね。場合わけ。
x<a のとき
y'=3(x+1)(x-3)>0 となる x の範囲は x<-1 , 3<x
これが x<a を満たすすべての x について成り立つための a の範囲は a≦-1
x≧a のとき
y'=3(x^2-2x+3). 判別式 D/4=-2<0 だからy'のグラフより常に y'>0
これが x≧a を満たすすべての x について成り立つための a の範囲は実数全体
(a がどんな値であっても y'>0 が成り立つんだからね)

求めるのはすべての実数 x に対して常に y'>0 となる a の範囲だから
x<a のときにも y'>0 となり、かつ x≧a のときにも y'>0 となる a の範囲なので
「a≦-1」かつ「a は実数全体」で
a≦-1
244大学への名無しさん:04/10/03 14:03:58 ID:SloUAMU8
>>235
それってa_nが収束することは言えるけど極限値まではでないでしょ?
245大学への名無しさん:04/10/03 14:33:55 ID:sYiPB0gL
関数を微分して0になる代入値を求めて、
その前後の代入値における微分結果の符合から極大極小を判断しますよね?
この時に書く増減表の書き方が分かりません。
たとえば0になる代入値より±1の代入値において微分して増減表を書いても、
その間でどう変化してるのか示せませんよね。
2で- 3で0 4で+ となっても
実は2.5から+ 3で0 3.5で- とかかもしれない。
増減表ってどう書けば良いんでしょうか?
246大学への名無しさん:04/10/03 14:45:35 ID:4Zkle2EF
>>245
その微分した関数が0になる代入値しか極値にならないので
代入値と代入値の間は単調増加か単調減少のみ
247大学への名無しさん:04/10/03 14:52:34 ID:sYiPB0gL
>>246
なるほど。
分かりました。
ありがとうございました。
248大学への名無しさん:04/10/03 15:32:46 ID:2DYAPTME
>>244
aとbがそれぞれ上限と下限なら
249242:04/10/03 15:50:10 ID:IFMXPLFS
>>243

よくわかりました。ありがとうございます。
250大学への名無しさん:04/10/03 17:16:57 ID:7wsTLwt6
Vのグラフかくときに必要な変曲点って
調べないときありますよね?
いったい、どういうときに必要で、どういうときはいらないのでしょうか?
Uなら3次関数変曲点なしにかいてましたよね。
251大学への名無しさん:04/10/03 17:19:02 ID:K6LquHiz
>>250
関数〜のグラフを書け→変曲点必要
微分して最大、最小値を求める場合→変曲点の必要なし
252大学への名無しさん:04/10/03 17:22:36 ID:7wsTLwt6
>>251
ありがとうございます。
でも、陰関数のグラフで調べてない問題があるんです・・・
253大学への名無しさん:04/10/03 17:24:21 ID:7wsTLwt6
d^2y/dx^2の意味教えてください。。。
254大学への名無しさん:04/10/03 17:45:17 ID:7wsTLwt6
模様みたいな対称性のあるグラフをかくときに
第2次導関数を調べてないのはどうしてですか?
255大学への名無しさん:04/10/03 17:48:36 ID:4Zkle2EF
調べてないことに意味なんかあるのか?
調べた方がより正確に書ける そういうものじゃねーの?

第二次導関数って言葉知ってるならd^2y/dx^2の意味普通分かるべ
関数yを変数xで二回微分した関数
256大学への名無しさん:04/10/03 17:58:18 ID:M+bqBUJg
すみませぬ

三角形ABCにおいて、AB=5、BC=3、cosC=5/9であるとき AC=□である。

っていうすごい単純な問題で答えは 6 になるはずなのですが
どうしてもならないので途中の過程を教えて下さい。。
257大学への名無しさん:04/10/03 18:02:03 ID:4Zkle2EF
正弦定理で角度Bを求めて余弦定理。あとは適当に計算すれば出ます

って解答用紙に書けば満点もらえるよ
258大学への名無しさん:04/10/03 18:04:07 ID:4Zkle2EF
うそついたw
いきなり角度Cに関する余弦定理で十分だね

5/9 = (x^2+9−25)/6x
259大学への名無しさん:04/10/03 18:04:12 ID:7wsTLwt6
>>255
ありがとうございます
260大学への名無しさん:04/10/03 18:10:11 ID:rwkKLOSW
内接四角形の面積Sは各辺をa,b,c,dとするとS=√{(s-a)(s-b)(s-c)(s-d)}ただしs=(a+b+c+d)/4 というやつです。ただ定理かどうかはわかりません
261大学への名無しさん:04/10/03 18:15:26 ID:M+bqBUJg
>>258
ホント申し訳ない、ありがとうございますた。。。
262大学への名無しさん:04/10/03 18:15:36 ID:hGPP4C3W
それがブラマグプタだろ
263大学への名無しさん:04/10/03 18:19:44 ID:rwkKLOSW
ブラマグプタというのですか?高3なのに知らなかったことに恥ずかしい、
264大学への名無しさん:04/10/03 18:50:11 ID:SW3R0CbC
しかも260微妙に間違えてるような
265大学への名無しさん:04/10/03 22:00:35 ID:f3b3cpGt
>>260
s=(a+b+c+d)/2だね
266大学への名無しさん:04/10/03 23:42:01 ID:0se1ah9K
図形の△ABCとか∠ABCとかのABCの順番がどうやって決まるのかよくわからないのですが…
適当でいいんですか?
267大学への名無しさん:04/10/04 00:48:37 ID:naao+J6C
>>266
三角形ならどうでも構わない。(普通は反時計回り)
角の場合は、真ん中に書いた点の部分にある角を指すことになる。
四角形より大きいのは、順番につないでねじれなければOK。
268大学への名無しさん:04/10/04 01:52:32 ID:q0+IlHBQ
 直交座標において、点A(√3,0)と準線x=4/√3 からの距離の比が
√3:2である点Pの軌跡を求めよ。
 またAを極,x軸正の部分の半直線AXなす角θを偏角とする極座標を定める。
このとき点Pの軌跡をr=f(θ)の形の極方程式で求めよ。

お願いします。
269大学への名無しさん:04/10/04 04:56:11 ID:X9yDP7YD
>>268
P(p,q)とする。点A(√3,0)と準線x=4/√3 からの距離の比が√3:2より
(p-√3)^2+q^2:(p-4/√3)^2=3:4
4{(p-√3)^2+q^2}=3(p-4/√3)^2
これを整理して(p^2)/4+q^2=1
Aを原点に置き直すと
(p-√3)^2/4+q^2=1
よってcosθ=(p-√3)/2,sinθ=qとおけるので
p=2cosθ+√3,q=sinθ
r^2=p^2+q^2
=4(cosθ)^2+4√3cosθ+3+(sinθ)^2
=3(cosθ)^2+4√3cosθ+4
=(√3cosθ+2)^2
r=√3cosθ+2
270おやじ:04/10/04 08:31:32 ID:uqhDPssr
よろしくお願いします。 長方形ABCDがあり、AB=CD=397、AD=BC=239とする。(単位はo)
 このとき、対角線ACの長さはいくつか。また、そのときできる直角三角形の、内角をそれぞれ求めよ。 これが問題です。よろしくお願いします。
271大学への名無しさん:04/10/04 08:41:41 ID:FkSPOAfr
>>270
写し間違い?
272大学への名無しさん:04/10/04 08:58:16 ID:suiJjfCJ
工作かなんかでもとめるのがめんどかったんだろ。
273大学への名無しさん:04/10/04 09:40:36 ID:2mc/v9WZ
>>270
三平方の定理と三角関数表と電卓を使いな。
274大学への名無しさん:04/10/04 14:42:37 ID:BwQS4gOt
関数f(x)=xlog(x/a)+(1-x)log{(1-x)/(1-a)}-2x^2+4ax(0<x<1)
についてf(x)の最小値を求めよ。ただし、aは0<a<1を満たす定数とする。

この問題については以下の様な考え方で解けば良いのでしょうか?(独学なので勘違いが無いか心配です)
まず微分して0になるxの値をもとめて、第二次導関数を使ってそれが極大か極小かを判断して、
後は定義域の両端と値を比較すれば・・・と思ったのですがこういう場合定義域の両端はlim[x→0]とかで良いんでしょうか?
第一次導関数ではx=aで0になり、これが極限の候補で
第二次導関数を求めると
{(2x-1)^2}/{x(1-x)}
となり、0<x<1の範囲では常に0以上、ということは第一次導関数がこの範囲では常に増加
第一次導関数は増加を続けてx=aで0だから、aより前ではマイナス、aより後ではプラスになる。
だからx=aが極小で最小。f(a)=2a^2でこれが最小値。
275大学への名無しさん:04/10/04 15:16:05 ID:Sxu0PTvN
>>274
>こういう場合定義域の両端はlim[x→0]とかで良いんでしょうか?
考え方としてはかまいませんが、あまり意味はないです。
両端の値を求めて大小を比べて… って解き方は楽に解くためのテクニックなので
両端の値が楽には求まらない、または両端自体が存在していないときに無理にやることでもないです。
微分して、増減調べて、最大・最小となる x の値を求めて、ってのが本来の解き方です。これだけでも十分。

>第一次導関数ではx=aで0になり、これが極限の候補で
違います。極値の候補です。

考え方はOKです。あとは、単調増加・単調減少という言葉を使いこなせれば望ましいと感じます。
276大学への名無しさん:04/10/04 17:04:50 ID:4x4eSk11
C:y=x^2+(6a+2)x+3a+4
の頂点をPとする
Cが異なる二点A、Bでx軸と交わるとき三角形APBの外接円の中心の座標を求めよ
お願いします
277大学への名無しさん:04/10/04 17:50:44 ID:b8oDmn1i
青チャート数Bの例題58(A)についてなんですが、解答の3行目
|→a|=1だから
→OH=(cosθ)→a=kaになるのかが、わかりません。一時間近く考えてます。それでもわからないんです。よろしくお願いします。
278大学への名無しさん:04/10/04 17:56:19 ID:sLeEC6xW
>>277
正射影でぐぐれ
279大学への名無しさん:04/10/04 18:30:49 ID:JN/LkLfx
>>276
aのことはひとまず置いといて
放物線C1:y=(x-p)^2+q
として問題を解く

対称性から外接円は
C2:(x-p)^2+(y-(q+r))^2=r^2
と書ける

C1とC2が点A(s,0)を通ることから
0=(s-p)^2+q
(s-p)^2+(q+r)^2=r^2
この2式より
r=(1-q)/2

外接円の中心は
(p,q+r)=(p,(1+q)/2)
pとqをaの式に直して終了
280276:04/10/04 18:57:52 ID:4x4eSk11
>>279 dクスです。ちなみにこの問題センターの本試験です。
捨て問だな。こりゃ
281大学への名無しさん:04/10/04 19:06:49 ID:JN/LkLfx
>>280
文系なんですね
理系ならまた来年まで頑張ってください
282大学への名無しさん:04/10/04 19:07:49 ID:JN/LkLfx
来年度に訂正
283大学への名無しさん:04/10/04 21:25:27 ID:mhTpcYcV
複素数 z,w があり、
  |z-i|=1 , w=(-i/2)z^2
を満たしている。ただし、z≠0,z≠2i とする。

(z-w)/w は純虚数である事を示せ。

とりあえず、共役な複素数との和が0になる事を示せばいいと思うのですが、
wの共役な複素数の出し方が分かりません。

|w|=|z^2|/2
|w|^2=(|z^2|^2)/4
ww~=)(z^2)(z^2)~)4
w~=)(z^2)(z^2)~)4w

これでいいんですかね?
全く以っていい気がしません。。。
284大学への名無しさん:04/10/04 21:26:22 ID:mhTpcYcV
|w|=|z^2|/2
|w|^2=(|z^2|^2)/4
ww~=((z^2)(z^2)~)/4
w~=((z^2)(z^2)~)/4w

に訂正します、すいません。
285大学への名無しさん:04/10/04 22:33:51 ID:p2nvrrMt
>>284-285
|z-i|=1 ⇔ zz~=i(z~-z)

(z-w)/w=-2/(iz)-1
286大学への名無しさん:04/10/04 22:36:16 ID:Sxu0PTvN
>>283
計算間違いはしてないように見える。

複素数 a,b に対して
(a+b)~=(a~)+(b~) , (ab)~=(a~)(b~)
が成り立つことを踏まえて
w の共役の求め方なら
w~={(-i/2)z^2}~
=(-i/2)~(z^2)~
=(i/2)(z~)^2
287大学への名無しさん:04/10/04 22:37:54 ID:bT9L5q6n
((z-w)/w)~=(2/(-iz)-1)~=(-2i)/(z~)-1

(z-w)/w+((z-w)/w)~=(2i)/(z)-1+(-2i)/(z~)-1
=(-2/|z|^2)(|z|^2+iz-iz~)

ここで、|z-i|=1より、
|z-i|^2=(z-i)(z-i)~=(z-i)(z~+i)=|z|^2+iz-iz~+1=1 ゆえ
|z|^2+iz-iz~=0
288大学への名無しさん:04/10/05 00:24:46 ID:ZvLybXWh
f(x)=Σ_[n=0,∞]x^2/(1+x^2)^n
でy=f(x)を求める問題なんですが、

x=0のときf(x)=0 は分かるのですが
x≠0のときは無限等比級数の和の公式を使って解けばよいのですか?
289大学への名無しさん:04/10/05 04:28:41 ID:kGi94OFY
一般に0が無限に足されると0だとは言えないが、方針は正しい。
290大学への名無しさん:04/10/05 04:32:41 ID:QwlHPkU9
291大学への名無しさん:04/10/05 06:58:31 ID:E7fH2eN1
>>289
0はいくら足しても0だよ.
292大学への名無しさん:04/10/05 08:55:19 ID:Lbu+yITj
>>280
それはまずいぞ
もっと勉強しる
293大学への名無しさん:04/10/05 10:45:42 ID:kGi94OFY
>>291
0*∞は不定形だろ?
294大学への名無しさん:04/10/05 10:56:52 ID:HTeDV1wF
>>293
この場合、0は極限じゃないだろ。
a_n→0, b_n→∞ なら、a_n*b_nは不定形だが、
a_n=0なら、b_n→∞でも、任意のnについて、0*b_n=0。

>>289は連続濃度の和(もはや和とは言いがたいが)みたいなことをイメージしてるんじゃないの?よく知らんが。
295大学への名無しさん:04/10/05 11:54:20 ID:DufbzUY5
>>269
ありがとうございました
296大学への名無しさん:04/10/05 12:30:33 ID:W/TYSdys
lim_[k=1→n]1=
この答えがnになるのですが何故ですか?
数列はなく定数1なのでkに1からnまで代入して和にできないのだから答えは0じゃないのですか
297大学への名無しさん:04/10/05 12:46:37 ID:HTeDV1wF
>>296
limじゃなくてΣ[k=1→n]1じゃないの?
Σ[k=1→n]1=1+1+…+1 (n個)=1だよ。

定数1だから、k(は実際には無いけど)に何をいれても1。

Σ[k=1→n](k+1)だったら、(1+1)+(2+1)+…+(n+1)って考えるでしょ?
これでkが0だったら…、って考えたら納得できないかな。
298大学への名無しさん:04/10/05 12:51:41 ID:TCnfKod6
>>296
>数列はなく定数1なので
違います。数列はあります。
1 , 1 , 1 , 1 , 1 , …
という数列です。第何番目の項であるかにかかわらず定数1であるような数列です。
一般項(すなわち第 n 項)は a_n=1 となります。a_1=1 , a_2=1 , a_3=1 , a_4=1 , …
299大学への名無しさん:04/10/05 16:49:37 ID:g+3njC+R
1+2+2^2+2^3+……+2^(n-1)=2^n-1
なんでこうなるのかわかりません
教えてくれませんか?
300大学への名無しさん:04/10/05 17:15:47 ID:022FzwQf
初項a=1、公比r=2の等比数列の一般項は、a[n]=a*r^(n-1) = 1*2^(n-1) で表せる。
よって第n項までの和は公式から、S = a{(r^n) - 1}/(r-1) = 1*(2^n-1)/(2-1) = (2^n)-1
301大学への名無しさん:04/10/05 17:32:43 ID:g+3njC+R
>>300
ありがとうござましゅ
等比数列の和はまったく頭に無く、ほかのことばっかり考えてて
まったく気付きませんでした…
302大学への名無しさん:04/10/05 17:59:16 ID:UdjdtYKq
順列の問題です。お願いします。

立方体の各面に1から6の数字が書いてあり区別が付くとする。
各面を赤か青かのいずれかに塗るとき、色の塗り方は何通りあるか。
303大学への名無しさん:04/10/05 18:24:47 ID:HTeDV1wF
>>302
2^6
304大学への名無しさん:04/10/05 18:24:56 ID:W/TYSdys
>>297-298
1,1,1,1,1,1,1…
これが数列だったとはorz
ありがとうございました。
305大学への名無しさん:04/10/05 20:27:00 ID:ltthBhaA
ガッコで平面図形に入ったんですが、定理ってうじゃうじゃあるじゃないですか。
あれはきちんと全部証明できた方がいいんですか?
私的に定理だけ覚えておけば良いかと思うんですが。
イケンください。
306大学への名無しさん:04/10/05 21:00:07 ID:UdjdtYKq
>>303
ありがとうございます。
分かりました
307大学への名無しさん:04/10/05 21:23:56 ID:TCnfKod6
>>305
きちんと全部証明できた方がいい。
308大学への名無しさん:04/10/05 21:36:49 ID:+6o3cemV
aは0<a<πを満たす定数とする。
n=0,1,2,3,4,5・・・・に対し、nπ<x<(n+1)πの範囲にsin(x+a)=xsinxを満たすxがただ1つ存在するので、
このxの値をx_nとする。
(1)極限値lim[n→∞](x_n-nπ)を求めよ。

この問題でまず、nπ<x<(n+1)π から、xは弧度法の単位円で言えば0とπを含まない全てに成り得るのだと考えたのですが、
ところがこれの解答で、
sinx_n=(1/x_n)sin(x_n+a) ・・・ 1
lim[n→∞]x_n=+∞
また|sin(x_n+a)|≦1
よって、1から、lim[n→∞]sinx_n=0
と書いてあったのですが、理解出来ません。
x_nが0とπにならないとすればsinx_n=0となる事は無いはず。
でも1の右辺が0になるのは分かります。
何処が間違っているのでしょうか?
309大学への名無しさん:04/10/05 21:56:46 ID:HTeDV1wF
>>306
sinx_n=0となることはないけど、0に近づく、という状況だよ。
この問題の場合は、x_n=nπ+a_nとでも書けば、0<a_n<πなんだけど、a_n→0という状況になっている。
310大学への名無しさん:04/10/05 22:32:18 ID:XA0cR6Wu
>>308
α=0とα→0を混同してるから
311305:04/10/05 23:03:26 ID:ltthBhaA
何故きちんと証明できた方がいいのでしょう?根拠があればぜひ聞かせて欲しいものです。
定理を覚えれば自然と証明できそうな気がするんですけどね。。。
312大学への名無しさん:04/10/05 23:21:19 ID:+KIA3Jqy
>>311
自然と証明は難しいでしょう。定理の証明をできるようにしとけば、証明の途中の知識が武器になる気がします。
先人の知恵的証明も図形ものにはおおいですし。
313大学への名無しさん:04/10/06 00:21:29 ID:qEj2xx15
>>311
証明できないより証明できたほうが良いにきまってるでしょう?
定理を覚えていることによって自然と証明ができているのであれば、それは証明ができるということです。
ちなみにきちんとしていなければ証明とは言いませんよね?
「証明ができる」と「証明の方針を知識として丸暗記している」というのは異なります。
314大学への名無しさん:04/10/06 00:34:19 ID:HdKIbxF/
「証明ができる」ことが無理でも「証明の方針を知識として丸暗記している」状態にしとくだけでもずいぶん効果はありますね。
証明が必要ないというのはもったいないと思います。
315大学への名無しさん:04/10/06 00:53:37 ID:qEj2xx15
必要か必要でないかと聞かれたら、どの程度の学力を目指すのかなどの目的によって変わってくるでしょうが
良いか悪いかと聞かれたらできるほうが良いに決まってるじゃない。
316大学への名無しさん:04/10/06 01:21:41 ID:lr0xc9L7
x^2+y^2=2の円において、点Aが円上を動くとき
B(x+y,xy)は
y=(1/2)x^2-1
の放物線上の(  )≦x≦(  )
また、Aが円状の内部の領域を動くとすると
Bが動く領域内には、x-y座標の格子点は(  )個ある。

3つの括弧を求める解法を教えてください
317大学への名無しさん:04/10/06 01:23:06 ID:lr0xc9L7
>>316
5行目訂正
円状の→円の
318大学への名無しさん:04/10/06 01:31:53 ID:CXBkwHyE
例えば極座標に直す
319大学への名無しさん:04/10/06 01:47:32 ID:j28JiMDK
普通にやれば解けるよ。
x+y=X
xy=Y
とおく。
但しX^2-4Y≧0
また、
x^2+y^2=2
X^2-2Y=2
2Y=X^2-2
先の条件より、
-2≦X≦2

また、
x^2+y^2≦2
X^2-2Y≦2
2Y≧X^2-2かつ
X^2-4Y≧0
X=0→2
X=1→1
X=2→1
よって対称性より
6個
320大学への名無しさん:04/10/06 01:48:47 ID:j28JiMDK
単なる同値変形の問題+おまけ
321大学への名無しさん:04/10/06 03:25:41 ID:lr0xc9L7
5行目の「但し」はどのようにしてその条件が出てくるんですか?
322大学への名無しさん:04/10/06 04:00:32 ID:L7wY6cGI
>>321
 頻出。
 x+y=X
 xy=Y
 なる変換を施した場合、XとYが存在することが必要で、
 実数X、Yが存在するとは、「ある二次方程式t^2−(x+y)t+xy=0」が実数解を持つっつーこと。
 この二次方程式の判別式Dが(ry
323大学への名無しさん:04/10/06 04:06:51 ID:lr0xc9L7
たびたび失礼
何故、XとYが存在する条件が「その二次方程式が実数解を持つとき」なんですか?
324大学への名無しさん:04/10/06 04:25:15 ID:L7wY6cGI
>>323
 xとyを、この二次方程式の実数解と見てるんだよ。
 「x+y、xyが共に実数」→「t^2−(x+y)t+xy=0が2つの実数解」
 解と係数の関係 みたいな。
325大学への名無しさん:04/10/06 04:29:40 ID:j28JiMDK
実数xとyが存在する⇔t^2-(x+y)t+xyの解が存在する⇔判別式(x+y)^2-4xy≧0
つまり
X^2≧4Y
326大学への名無しさん:04/10/06 10:48:24 ID:G/yUvXWm
青茶の例題98、対数の問題です。

log{2}(3)の値を小数第2位以下を切り捨てて、小数第1位まで求めよ。

指針
1<log{2}(3)<2であるから、次の条件を満たすaの値を求める。

1+a/10<log{2}(3)<1+(a+1)/10 aは整数、0≦a≦9

ここまでがすでにわかりません。お願いします。
327大学への名無しさん:04/10/06 12:50:42 ID:qEj2xx15
>>326
実数の十進小数展開についての理解が必要かもしれませんね。

まず実数 x>0 について、n≦x<n+1 となる整数 n が必ずただひとつだけ存在します。この n が x の整数部分と呼ばれるものです。
次に、n〜n+1 の区間を幅 1/10 の小区間10個に10等分します。数直線でイメージしてください。
この10個の小区間に左から順に 0〜9 までの番号を振っておくと、この番号はその小区間に含まれる数の小数第1位を表すことになります。
このとき n≦x<n+1 なので、x はこの10個の小区間のいずれかに含まれます。x の含まれる小区間が第 a 番目(0≦a≦9)の区間だとすると
n+a/10≦x<n+(a+1)/10 と書けます。この a が x の小数第1位です。
次に、n+a/10〜n+(a+1)/10 の区間をさらに10等分します。10等分したものをさらに10等分するのだから区間幅は1/100になってます。
同様に小区間 0〜9 までの番号を振ると、これはその小区間に含まれる数の小数第2位を表します。
このとき n+a/10≦x<n+(a+1)/10 なので、x はこの10個の小区間のいずれかに含まれます。第 b 番目(0≦b≦9)に含まれるとすると
n+a/10+b/100≦x<n+a/10+(b+1)/100 と書けます。この b が x の小数第2位です。
以下同様にして小数第3位、第4位… と求めていって x の十進小数展開
x=1.abcd… すなわち
x=1+a/10+b/10^2+c/10^3+d/10^4+…
が得られます。どんな実数もこの方法で無限小数として表すことができるのです。

これを踏まえて、考えてみてください。
328302:04/10/06 17:27:11 ID:CyN95uOL
すみません。お願いします。

(1)立方体の各面に数字がなく区別が付かないものとする。
  各面を赤か青かのいずれかに塗るとき、色の位置関係だけに注目すると
  色の塗り方は何通りあるか。
(2)(1)と同じ条件で、赤と青と黄のうちのちょうど二色を用いて
  各面を塗り分ける場合、色の塗り方は何通りあるか。


329大学への名無しさん:04/10/06 17:31:11 ID:MhEuWxnG
ここは宿題を代わりにやるスレではないよ。
330大学への名無しさん:04/10/06 18:13:24 ID:5MvsCqDM
1対1対応の演習3C(旧課程)から、
微分演習11で、
(2) f´(x)=(-α)/x+(1-α)/(1-x) (0<x<1) (0<α<1)
これが増加関数

(3) g´(α)=log(1-α)/α (0<α<1)
これが減少関数
らしいんですが、どうやったら分かるんですか?
331大学への名無しさん:04/10/06 19:48:02 ID:/dHCK5zS
>>328
(1)10通り
(2)30通り
332大学への名無しさん:04/10/06 20:13:44 ID:/dHCK5zS
>>330
y=(-p)/x  と  y=p/(1-x)  (p>0)のグラフを書いてみろ。
どちらもxについての増加関数であることから、それらを合成した関数
f´(x)=(-α)/x+(1-α)/(1-x) がxについて増加関数であることが言える。

(3)もy=(1-α)/α のグラフを書いてみたら分かる。
333大学への名無しさん:04/10/06 20:16:45 ID:/dHCK5zS
書き方が悪かった。
合成するのは (-α)/x と (1-α)/(1-x) ね。
334大学への名無しさん:04/10/06 20:59:53 ID:dOnTtWbH
1<α≦2に対して、ある自然数m、nが存在して、
[mα]=[n+n/(α−1)]が成り立つならば、αは有理数であることを示せ。

[ ]はガウス記号です。全然わかりません、ご教授お願いします
335大学への名無しさん:04/10/06 21:11:31 ID:5MvsCqDM
>>332
分かりました。ありがとう
336大学への名無しさん:04/10/06 21:59:56 ID:/dHCK5zS
>>334
β= 1 + 1/(α-1)と置くと、(1/α)+(1/β)=1
[mα]=[nβ]=k とすると、ガウス記号の定義より

[mα]≦mα<[mα]+1 ⇔ [mα]/α≦m<{[mα]+1}/α
[nβ]≦mβ<[nβ]+1 ⇔ [nβ]/β≦n<{[nβ]+1}/β

辺々加えると、

 (k/α)+(k/β)≦m+n<(k+1)/α +(k+1)/β
∴k≦m+n<k+1 ………★
m、n、k は整数であるから、
m+n=k
∴(k/α)=m、 (k/β)=n
∴α=k/m、β=k/n 
したがって、αは有理数である。■
337大学への名無しさん:04/10/06 22:02:13 ID:dOnTtWbH
>>336
天才やね。君、理III受かるよ。
338河合生:04/10/06 22:03:09 ID:Im3mSDnh
教えてくださいm(__)mXY平面上の曲線y=x^2上の3点をx座標のちいさいものから順にA、B、Cとする。AとBのx座標の差はa(a>0の定数)BとCのx座標の差は1の関係を保ちながら3点が動くとき、∠BACが最大になるときの点Aのx座標をaで表せ。
339大学への名無しさん:04/10/06 22:08:14 ID:dOnTtWbH
>>336
ごめん、もうちょっと教えてください。

>∴(k/α)=m、 (k/β)=n
になるのはなんでですか?
340大学への名無しさん:04/10/06 22:13:46 ID:/dHCK5zS
>339
さかのぼっていくと、不等式のすべてが等号成立でなければならないことが分かるyo
341大学への名無しさん:04/10/06 22:21:57 ID:dOnTtWbH
>>340
k/α≦m<(k+1)/α⇒k/α=mになるんですか??全然わからんですばい・・・
342大学への名無しさん:04/10/06 22:25:13 ID:+8W4rM79
∴k≦m+n<k+1
m、n、k は整数であるから、
m+n=k
からすべての等号成立がうまれるよ
343大学への名無しさん:04/10/06 22:33:06 ID:dOnTtWbH
>>342
ああああ、やっとわかりました。
こういうことですよね?
k≦m+n<k+1⇒k=m+nだったので、これが成り立つためには、逆をたどり、
(k/α)+(k/β)=m+nで、またたどり、[mα]/α=m、[nβ]/β=nである。
てことですよね?
344大学への名無しさん:04/10/06 23:22:09 ID:W9PW2sJL
すいません。お願いします。

AC=CB=BD=DA=3  OC=OD=5のとき
OC・ODが一定値であることを示せ。
345302:04/10/06 23:40:07 ID:CyN95uOL
>>331
自分もそう思ったんですが(2)のほうの答えが
解答では24通りになっているんですが・・
346大学への名無しさん:04/10/06 23:49:53 ID:qEj2xx15
>>332-333で関数の和を関数の合成と誤表記しているのが気になった。
347大学への名無しさん:04/10/07 00:16:04 ID:sUwEQfif
>344 その文字はベクトルですか?
348331:04/10/07 00:16:48 ID:/RWFIidg
>>346
確かに。ご指摘ありがとう。

>>345
ミスったわ。(2)は「ちょうど二色」だから、(1)から2を引いた8を3倍して、答えは24通りだ。
349河合生:04/10/07 00:20:51 ID:wod68tv/
338
の解き方をお願いします、できれば計算も。m(__)m河合の東大文系数学の添削問題です。数学の得意な方お願いしますm(__)m
350大学への名無しさん:04/10/07 00:20:54 ID:SAgNi8kZ
>>345
「ちょうど2色」なので
(1)から赤一色の場合と青一色の場合を除いてから3倍する必要がある
よって(10-2)×3より24通り
351河合生:04/10/07 00:23:22 ID:wod68tv/
>338
ちなみにaは正の定数です。お願いします!
352344:04/10/07 00:25:24 ID:SN+UUDjf
>>347
ベクトルじゃありません。よろしくお願いします
353大学への名無しさん:04/10/07 00:28:11 ID:E7asaz5y
>>338>>349
自分で悩んだほうが力がつくのでは?と思いつつ

A(p,p^2),B(p+a,(p+a)^2),C(p+a+1,(p+a+1)^2)とおくと
ABの傾きは2p+a,ACの傾きは2p+a+1

tanα=2p+a,tanβ=2p+a+1とおくと
tan∠BAC=tan(β-α)
=1/(1+(2p+a)(2p+a+1))
これが最大になるのは、分母が最小になるときで
p=-(2a+1)/4のとき(答)
354大学への名無しさん:04/10/07 00:29:25 ID:E7asaz5y
>>344>>352
ベクトルじゃなければ
OC・OD=25
なので当然のごとく一定では?
355344:04/10/07 00:37:15 ID:SN+UUDjf
>>354
すいませんm(_ _)m訂正です
OC=OD=5→OA=OB=5
申しわけないっす
356河合生:04/10/07 00:37:19 ID:wod68tv/
>353
ありがとうございます!Bのx座標をtとおいたうえにCOSで解こうとした計算でルートの微分が出てきたりして止まってました(^o^;
357大学への名無しさん:04/10/07 00:40:07 ID:HYtLkfed
>>344
点Cと点Dが一致するとき値が異なるから一定ではない。
358大学への名無しさん:04/10/07 00:41:28 ID:SN+UUDjf
そうですよね
問題がやっぱり間違ってますね。ありがとうございました
359大学への名無しさん:04/10/07 00:50:26 ID:0YtEY6uq
>>327
ありがとうございます!解けました☆★
360大学への名無しさん:04/10/07 01:04:10 ID:BwFmyHK1
答えが分からなくて困ってます。どなたか質問に答えていただけませんか?
「正の実数r,tに対し,点(t,t)を中心とし1辺が2rtの正方形をSとする。
ただし,Sの各辺はx軸またはy軸に平行であるとする。
このとき,正方形Sと曲線y=log x が交わりをもつようなtが存在するためのrの条件を求めよ。
(log は自然対数とする)」という問題なんですが

この問題で
f(t)=log(1+r)t-(1-r)t (t>0)とおくと
f(t)≧0となるtがあるとき,y=log x とSは共有点をもつ
というところまでは分かったんですが
ここからrの場合分けのやり方が分からないんですが詳しく教えていただけませんか?
361大学への名無しさん:04/10/07 01:15:49 ID:7TRqKdj1
362大学への名無しさん:04/10/07 01:37:05 ID:BwFmyHK1
>>361
そのやり方でやってもr場合分けが必要になるみたいだし
だからr場合分けのやり方を聞いてるんですが(場合分けないと不正解りそうだから
363大学への名無しさん:04/10/07 01:38:04 ID:BwFmyHK1
訂正
不正解りそうだから→不正解になりそうだから
364大学への名無しさん:04/10/07 01:45:53 ID:HYtLkfed
>>362-363
だったらそっちのスレで聞きな。マルチポスト(・A・)イクナイ!!
365大学への名無しさん:04/10/07 01:53:53 ID:BwFmyHK1
>>364
分かりました。そうします

(確かに問題文は同じだけど)この質問はここでしかしてないのにマルチって言われるのか・・・
マルチって全く同じことをいろんなところで聞くことじゃないんですか?
366大学への名無しさん:04/10/07 02:25:59 ID:9smLE1Yw
y=f(x)をxで微分したときなどに使う、
「y'」と「dy/dx」は全く同じ意味の記号として使っていいんでしょうか?

y^2=x
両辺をxで微分すると
2yy'=1

という部分が解説の途中にあるんですが
2yy'=1を2y・dy/dxと書き換えても問題ありませんか?

かなり基本なのかも知れませんが分かりません。教えてください。
367大学への名無しさん:04/10/07 02:27:27 ID:9smLE1Yw
すいません、訂正です。

>2yy'=1を2y・dy/dxと
    ↓
2yy'=1を2y・dy/dx=1と
368大学への名無しさん:04/10/07 13:01:35 ID:ptQDh/ef
2y・dy/dx=1 → 意味は同じだが、2y(dy/dx) =1 と、カッコで括ったほうがいいかも。
369大学への名無しさん:04/10/07 22:48:35 ID:bINzOqak
点(0,2)を中心とする半径r(>0)の円をCとする。
Cの領域y≧x^2に含まれるようなrの最大値を求めよ。

お願いします。
370大学への名無しさん:04/10/07 22:56:32 ID:pIDAoCTA
点A(0,2)とする
放物線y=x^2上の点P(t,t^2)と点Aとの距離の2乗は
AP^2
=t^2+(t^2-2)^2
=t^4-3t^2+4
=( t^2-(3/2) )^2+7/4
よってAPの最小値は(√7)/2で、これがrの最大値
371大学への名無しさん:04/10/07 23:19:25 ID:F9V8Re1F
ある観光船の船客のうち、
27人は男の子供、
15人は日本人の子供、
27人は男の大人、
21人は外国人の男の子供、
42人は日本人、
18人は日本人の男、
21人は外国人の女である。
船客は全部で何人か。

この問題、順に人数をa,b,c,d,e,f,gと置くと
(a+c)+{(e-f)+g}=99
と解けてしまうようなのですが、提示された条件を使い切らずに解くのには何か違和感があります。
どこかに罠が隠れていたりするんでしょうか
372大学への名無しさん:04/10/07 23:29:49 ID:bINzOqak
>>370
ありがd♪

>>369
【訂正】
誤:Cの領域
正:Cが領域
373大学への名無しさん:04/10/07 23:56:26 ID:8WT/h0dO
>>371
使わなくていい条件を探すのも問題のうちでしょ。
374大学への名無しさん:04/10/08 02:05:28 ID:8rqDGvp0
>>371
日本人の子供、日本人の男∈日本人
よって日本人は42人
男の大人+男の子供-日本人の男=外人の男=36
外人の男+外人の女=外人=57
よって船客は99人
375大学への名無しさん:04/10/08 13:15:34 ID:8JhU4iMO
(a+b+c)(ab+bc+ca)≧9abc を証明してください 左辺‐右辺 したんですけどくくれなくて
376大学への名無しさん:04/10/08 14:46:02 ID:+niPzKoy
>>375
相加平均相乗平均の不等式って聞いたことある?
377大学への名無しさん:04/10/08 14:52:02 ID:8JhU4iMO
一応ありますけど あんまりよくわかりません(>_<)
378大学への名無しさん:04/10/08 15:05:02 ID:+niPzKoy
>>377
んじゃまず、それを調べて、それに関する練習問題をいくつかやって
そいでから、もう一度問題を見直してみろ。
379大学への名無しさん:04/10/08 15:08:49 ID:1fnXTunr
というか数と式の問題端からやった方が良いかと。
380大学への名無しさん:04/10/08 16:52:41 ID:US9iGAzT
(tanθ)^2の原始関数はどうやったら求められますか?
一応いろいろ変形させてみたんですが、
どうやっても上手くいかないんです。
381大学への名無しさん:04/10/08 17:07:32 ID:LOOpTGKK
>>381
tanθ - θ
382大学への名無しさん:04/10/08 17:13:21 ID:kgKdUj/A
>>375
a=-2,b=0,c=3
383大学への名無しさん:04/10/08 17:16:30 ID:j0hXQ6d3
tan^2(θ) = sin^2(θ)/cos^2(θ) = {1 - cos^2(θ)}/cos^2(θ) = 1/cos^2(θ) - 1
384380:04/10/08 17:30:36 ID:US9iGAzT
>>383
すいません。できたら
>>1/cos^2(θ)
この先をもう少し詳しくお願いできますか?
385大学への名無しさん:04/10/08 17:38:35 ID:NsRb6aLm
>>384
教科書嫁
386大学への名無しさん:04/10/08 18:11:01 ID:riWD6Q08
組立除法で、割る数が分数の時、求めた商を割る数で割るのは何故ですか?問題集に説明がありません。
387380:04/10/08 18:23:43 ID:R4/jrQB4
>>383
すいません、自己解決しました・・・
わざわざすみませんでした。
388大学への名無しさん:04/10/08 20:44:01 ID:M0irUhQM
>>386
f(x)=(ax-b)Q(x)+R(x)より
f(x)={x-(b/a)}aQ(x)+R(x)
となるから
389大学への名無しさん:04/10/08 22:33:55 ID:GNoh8Jhz
[問題]
曲線y=x^3-3x^2-1の接線で原点を通るものを求めよ。

接線は原点を通るからy=mxとおいてy=x^3-3x^2-1に代入、整理して
x^3-3x^2-mx-1=0が重解となるmを求めればいいまでは考えましたが、
そこからが分かりません。

390大学への名無しさん:04/10/08 22:38:47 ID:NsRb6aLm
>>389
萩野だったらブチ切れな解法だな。
接点のx座標をおいてから計算するんだよぉ!!!って。
391大学への名無しさん:04/10/08 22:41:55 ID:Fou2AqlV
>>389
接線を求めるわけだから微分汁
y=x^3-3x^2-1  @
y'=3x^2-6x    A
接点の座標(x,x^3-3x^2-1)  B
@ABより、接線の方程式
y=(3t^2-6t)(x-t)+t^3-3t^2-1  C
これに原点を通すんだから y=x=0
これをCに代入
0=-3t^3+6t^2+t^3-3t^2-1
0=-2t^3+3t^2-1
これでtを求めて、それをCに代入して整理する
三次関数の因数分解は面倒なので俺はここまでで・・・
392大学への名無しさん:04/10/08 22:44:04 ID:lZQa88d1
y'=3x^2-6x
よって、
(y-t^3+3t^2+1)=(3t^2-6t)(x-t)
x=y=0
-t^3+3t^2+1=-3t^3+6t^2
2t^3-3t^2+1=0
(t-1)(2t^2-t^2+1)=0
よって、t=1
y=-3x
393大学への名無しさん:04/10/08 22:53:21 ID:GNoh8Jhz
>>390
??

>>391>>392
なるほど、ありがとうございます。
接線の問題は分かったけど

「x^3-3x^2-mx-1=0が重解となるmを求めよ。」
こっちの解き方はどうなんだろう?答えは同じはずだけど。

x^3-3x^2-1=mxは重解なのでx^3-3x^2-1=0とy=mxは接する。
よってmはx^3-3x^2-1=0の接線のうち原点を通るものの傾きである(以下>>391に続く)

ってやるわけ?こんなんおもいつかねー。数字変わったら絶対わかんねー。
394大学への名無しさん:04/10/08 23:00:58 ID:lZQa88d1
>ってやるわけ?こんなんおもいつかねー。数字変わったら絶対わかんねー。

微分して条件代入するだけじゃん。普通の解き方をすれば解ける。
てか、この問題3次式だし、解が多そうだから普通重解かどうかではしないよね。
やっぱりmに微分した値を代入して3次式をとくだけだよね。
その中で解が重なる部分が解かな。
395大学への名無しさん:04/10/08 23:02:13 ID:lZQa88d1
そうでもないか。微分した時点で接線の条件になってるから、
後は>>391に帰着だね。
396大学への名無しさん:04/10/08 23:11:02 ID:GNoh8Jhz
x^3-3x^2-1=mxは重解なのでx^3-3x^2-1=0とy=mxは接する。
よってmはx^3-3x^2-1=0の接線のうち原点を通るものの傾きである(以下>>391に続く)

じゃなくて

x^3-3x^2-1=mxは重解なのでy=x^3-3x^2-1とy=mxは接する。
よってmはy=x^3-3x^2-1の接線のうち原点を通るものの傾きである(以下>>391に続く)

の間違いだね。きゃー恥ずかしい。

上の解き方だと、mがxの係数ならOKだけど、mがx^2の係数にくるともうわけわかめに
なる・・・。OTZ
397大学への名無しさん:04/10/08 23:21:01 ID:Ws1knfIR
「x^3-3x^2-1=mxは重解なので」なんて書いてる時点で駄目だな。
398大学への名無しさん:04/10/08 23:23:18 ID:/u5yfvnJ
どぉでもいいが、もう1本接線ひけない?
399大学への名無しさん:04/10/08 23:25:10 ID:8T1dQgnM
http://www.mashiko-tcg.ed.jp/masityu/1/img-box/img20041008232324.jpg

模範回答とは別にlog{3}を底にしてみたのですが答えがあいません。
たぶんめっちゃバカな事でスベってるんだとおもうんですが、ご指摘おねがいします。
400大学への名無しさん:04/10/08 23:30:22 ID:Fou2AqlV
>>399
3行目が間違い
log_{3}(4)/log_{3}(2)
については分子だけをまず考えて
log_{3}(2)^2 ⇒ 2log_{3}(2)
よって
2log_{3}(2)/log_{3}(2)
=2/1=2
401大学への名無しさん:04/10/08 23:37:45 ID:8T1dQgnM
あっりがとうございます!
マジこーゆー事先生に聞くと答えてもらった後気まずい雰囲気なんすよ。
だからでかい問題のついでにもってくしかなくって・・・
どーもでした!
402大学への名無しさん:04/10/08 23:41:59 ID:lZQa88d1
>>392
y'=3x^2-6x
よって、
(y-t^3+3t^2+1)=(3t^2-6t)(x-t)
x=y=0
-t^3+3t^2+1=-3t^3+6t^2
2t^3-3t^2+1=0
(t-1)(2t^2-t-1)=0
(t-1)(2t+1)(t-1)
よって、t=1,1/2
y=-3x , -9/4
403大学への名無しさん:04/10/08 23:46:31 ID:Fou2AqlV
>>402
もしよかったらどうやって3次関数を因数分解してるか教えてもらえないだろうか・・・
いつもそれの分解で時間を食ってしまう
ちなみに私は
2t^3-3t^2+1=0
などの場合まず先頭の項と定数項の公約数あたりを考えて
(2t-1)(t^2 -1)
(t-1)(2t^2 -1)
(2t+1)(t^2 +1)
(t+1)(2t^2 +1)
なんて置いて一個ずつ当てはめていくんだが…orz
404大学への名無しさん:04/10/08 23:51:31 ID:8JhU4iMO
(a+b+c)(ab+bc+ca)≧9abcの証明の仕方をおしえてください(>_<)やっぱりどうしてもわからないんです
405大学への名無しさん:04/10/08 23:52:00 ID:lZQa88d1
1入れて0になったから(t-1)で割ってみた。
なんていうか定数項の約数の正と負を代入したら大抵出る。
それ以外は別の方法を考える。
406大学への名無しさん:04/10/08 23:53:24 ID:lZQa88d1
±定数項の約数/再高次の係数の約数
だね。
407大学への名無しさん:04/10/08 23:55:27 ID:lZQa88d1
>>404
a=-2,b=0,c=3 代入したら成り立たないよ。
全部正とかの条件が抜けてない?
408大学への名無しさん:04/10/08 23:56:59 ID:Fou2AqlV
>>405
なるほど・・・
確かにどっちか出ればだいぶ楽に分解できそう
ありがとう
409大学への名無しさん:04/10/08 23:59:23 ID:tBBZeMAJ
410大学への名無しさん:04/10/09 00:05:26 ID:o4PRrOfc
>>409
代ゼミのサテラインの授業映像と思われる。
内容から教科書基礎レベルぐらいかと思われ。

PC精神ともに無害。
411大学への名無しさん:04/10/09 00:06:52 ID:o4PRrOfc
>>409
>>390への返信だね。
412大学への名無しさん:04/10/09 00:13:29 ID:rqHENJcr
>>393
>「x^3-3x^2-mx-1=0が重解となるmを求めよ。」

F(x)=x^3-3x^2-mx-1
F'(x)=3x^2-6x-m

3次式が重解を持つときのグラフを思い描いてみてください
y=F(x)がx=aのときに極大値(極小値)を持ち、その値が0になればよいので
F(a)=0とF'(a)=0を連立してm=(15/4),-3を得ます
413大学への名無しさん:04/10/09 00:14:40 ID:o4PRrOfc
>>412
良いね。それ。
414大学への名無しさん:04/10/09 00:18:09 ID:dW4F8Mui
404 ごめんなさい(>_<) 全部正です
415389:04/10/09 00:37:59 ID:R9U6wYho
>>412
どうも、ありがとうございます。
>>409
ワラタ。僕は誘惑に負けたのか。

スレへの恩返しのために>>404の問題を解いてみます。

(a+b+c)(ab+bc+ca)-abc
=(b+c)a^2+(b+c)^2a+bc(b+c)←aについて整理
=(b+c){a^2+(b+c)a+bc}
=(b+c)(a+b)(a+c)
≧2√(bc)・2√(ab)・2√(ac)←a,b,cが正より相加相乗平均の関係より
=8abc
よって(a+b+c)(ab+bc+ca)≧9abc
416大学への名無しさん:04/10/09 00:39:20 ID:rqHENJcr
>>415
機械的にやるなら(x-a)(x-b)^2を展開して
(x^3-3x^2-mx-1)と係数比較してもいいでしょう
417大学への名無しさん:04/10/09 00:42:54 ID:T4rPIrqn
>>415

どうせ相加相乗使うのなら、

a+b+c ≧ 3 √(abc)  ※ この √ は3乗根と思ってくだされ
ab+bc+ca ≧ 3 √(abc)^2  ※ この √ も3乗根と思ってくだされ

右辺の3乗根の中は、(abc)^3 になるから3乗根は外れて
右辺は、9abc になるね。


418大学への名無しさん:04/10/09 00:45:18 ID:T4rPIrqn
>>1みたら、累乗根の書き方は

●累乗根:[n] √(a+b)=(a+b)^(1/n)
らしいね。

a+b+c ≧ 3 (abc)^(1/3)
ab+bc+ca ≧ 3 (abc)^(2/3)

(a+b+c)(ab+bc+ca)≧9(abc)^(1/3+2/3)=9abc



419大学への名無しさん:04/10/09 00:52:15 ID:bnKkYLNI
>>414
解1
a+b+c≧3(abc)^(1/3),ab+bc+ca≧3(ab*bc*ca)^(1/3)
(いずれも等号成立はa=b=cのとき)
より
(a+b+c)(ab+bc+ca)
≧3(abc)^(1/3)*3(ab*bc*ca)^(1/3)
=9abc(等号成立はa=b=cのとき)

解2
(a+b+c)(ab+bc+ca)-9abc
=(b+c)a^2+(b^2+c^2)a+bc(b+c)-6abc
≧2(bc)^(1/2)*a^2+2abc+(bc)^(3/2)-6abc(∵相加相乗平均、等号成立はb=Cのとき)
=2√(bc){a^2-2√(bc)+bc}
=2√(bc){a-√(bc)}^2
≧0(等号成立はa=√(bc)のとき)
∴(a+b+c)(ab+bc+ca)≧9abc(等号成立はb=Cかつa=√(bc)、すなわちa=b=cのとき)
420大学への名無しさん:04/10/09 00:55:26 ID:R9U6wYho
>>416
いろいろなやり方があるんですね。

>>415で突然(a+b+c)(ab+bc+ca)-abcを因数分解した動機は説明がいるかも。
前に(a+b+c)(ab+bc+ca)-abcの因数分解をやったことがあって結果も特徴的だったから覚えてたんですね。
(a+b+c)(ab+bc+ca)を見たとたん、(a+b+c)(ab+bc+ca)-abcと関係あるかなと思ってやってみたらたまたまうまく行っただけの話です。
もっと自然な発想の解法があると思います。このスレの神達がきっとレスしてくれるでしょう。

と思ったらさっそく神(>>417)が!

3変数の相加相乗平均ですか。そんなの授業でやらんかったよ。OTZ
興味深いんで証明してみよ。
a+b+c ≧ 3 √(abc)を認めるなら、うちのより筋の良いかつ瞬殺の解答ですね。
421大学への名無しさん:04/10/09 01:07:52 ID:UjMvjw6y
>>420
ていうか、アンタ相加平均相乗平均の不等式を知らないのか?
別に2変数とか3変数だけの物じゃないぞ。
422大学への名無しさん:04/10/09 01:15:17 ID:R9U6wYho
>>421
知ってますって。でも教科書見てもチャート式覗いても
a+b ≧ 2 √abしかないです。3変数のは初耳です。


423大学への名無しさん:04/10/09 01:17:47 ID:o4PRrOfc
424大学への名無しさん:04/10/09 01:19:02 ID:9HQXVeBT
>>422
2変数が一般的でそれ以上は確かに発展的だが、3変数までは知ってたほうがいいよ。
証明もやさしいし。
425大学への名無しさん:04/10/09 01:19:21 ID:UjMvjw6y
正の実数a(1),a(2),…,a(n)に対して
Σ[k=1,n] a(k)/n
を相加平均といい、
(Π[k=1,n] a(k))^(1/n)   ( ←ΠはΣの掛け算バージョン )
を相乗平均という。
んで、相加平均相乗平均っていうのは、相加平均≧相乗平均っていう不等式。
等号成立条件a(1)=a(2)=…=a(n)
だから別に2変数でなくても良い。 証明は簡単だから略。
426大学への名無しさん:04/10/09 01:24:27 ID:R9U6wYho
>>423-425
ほえーn変数までできるのか。こいつはたまげた。
>>423は理解不能だけど、3変数までは証明やって見て納得した。

今日はいろいろと勉強になった。皆さん本当にありがとう。
427RedRum ◆RRlBLdA0dk :04/10/09 03:43:44 ID:M96zUL3t
コーシーシュワルツの不等式より、
{(√a)^2+(√b)^2+(√c)^2}{(√bc)^2+(√ca)^2+(√ab)^2}≧(√abc+√abc+√abc)^2=9abc
等号はa=b=cのとき成立
428大学への名無しさん:04/10/09 10:19:58 ID:+oBSOoi0
複素数α、βで↓これの証明ってどうやればいいんですかね?
|α+β|≦|α|+|β|

図に描いてみるとなんとなくわかるんですけど、式での証明ができません。
429大学への名無しさん:04/10/09 10:56:38 ID:0sLfayNI
三角不等式を知らんのか
430大学への名無しさん:04/10/09 11:34:09 ID:AxWY3wPJ
>>428
一般に複素数 z=x+yi について
|z|=√(x^2+y^2)≧√(x^2)=|x|≧x=Re(z)
(等号成立は y=0 かつ x>0。すなわち z が正の実数)
したがって
|α+β|^2=(α+β)(α~+β~)
=αα~+αβ~+α~β+ββ~
=|α|^2+2Re(αβ~)+|β|^2
≦|α|^2+2|αβ~|+|β|^2
=|α|^2+2|α||β|+|β|^2
=(|α|+|β|)^2
よって
|α+β|≦|α|+|β|
等号成立は αβ~ が正の実数のとき
すなわち、arg(αβ~)=0°
⇔argα-argβ=0°
⇔argα=argβ
431大学への名無しさん:04/10/09 11:40:13 ID:AxWY3wPJ
>>430で微妙に間違えた。
3行目
 x>0 →x≧0
 正の実数→負でない実数
下から4行目
 正の実数→負でない実数
その下3行の後ろに
 または αβ~=0
 ⇔ αβ=0
を追加

要するに αとβのどちらかが0のときも等号成立ってこと。
432大学への名無しさん:04/10/09 12:38:28 ID:JHD1YbWk
>>404
相加相乗平均をつかわないでも
左辺-右辺をやって
(a+b+c)(ab+bc+ca)-9abc
=(b^2+c^2)a+(c^2+a^2)b+(a^2+b^2)c-6abc
=(b^2-2bc+c^2)a+(c^2-2ca+a^2)b+(a^2-2ab+b^2)c
=a(b-c)^2+b(c-a)^2+c(a-b)^2≧0(a,b,cはすべて正なので、等号はa=b=cの時成立)

で証明できるよ。
433大学への名無しさん:04/10/09 13:37:27 ID:ASNzuTjH
行ベクトルと列ベクトルの違いって何ですか?

問題解いてて、行ベクトルを何の断りもなしに、列ベクトルを使って表してかまわないの?
内積とか出す時に。
434大学への名無しさん:04/10/09 14:08:15 ID:AxWY3wPJ
>>433
行ベクトルと列ベクトルの違いは成分を縦に並べて書くか横に並べて書くかの違いです。
それぞれ(1,n)型行列、(n,1)型行列とみなせます。行列同士の加法は型が同じときのみ定義されてます。

何の断りもなく行ベクトルを列ベクトルとして扱うのはダメ。
何の断りもなく座標を列ベクトルで表すのはOK。
座標は行ベクトルで表すことも列ベクトルで表すこともできますが、1つの問題内ではどちらか片方に統一しておきましょう。

内積については、定義を見ればわかるように成分の並べ方には関係ない。縦に並べようが横に並べようが斜めに並べようが問題ない。
ただ、ベクトルを行列とみなして内積を行列の演算を用いて定義しているような場合は別ですが、高校ではそんなことやらんでしょ。
435大学への名無しさん:04/10/09 14:24:23 ID:ASNzuTjH
A↓=(a,b) , B↓=(c,d) とした時、
2つのベクトルの内積を、(a,b)・(c,d) と表してはいけないと高校で言われたんですが、かまわないんですか?
436大学への名無しさん:04/10/09 14:42:01 ID:AxWY3wPJ
>>435
おまいの学校の採点基準は知らん。間違いではないし、かまわないと思う。
ただ見にくい。誤解を招く可能性もないことはない。望ましい表記ではない。
437大学への名無しさん:04/10/09 15:24:03 ID:ngoc/5N5
1+1=田
438大学への名無しさん:04/10/09 16:59:44 ID:eJBTW5l4
>>430thx。すごいです。
439大学への名無しさん:04/10/09 18:11:56 ID:44Xk8cHJ
y=x^2をx^2+(y-a)^2=r^2に代入して
y+(y-a)^2=r^2⇔y^2-(2a-1)y+a^2-r^2=0…★
★の解からy=x^2とx^2+(y-a)^2=r^2の位置関係をどうやって決めるのですか?
参考書にもなんかごまかして書いてあるのでよろしくお願いします。
440大学への名無しさん:04/10/09 18:15:26 ID:WR8JY356
とりあえず
y=x^2かつ
x^2+(y-a)^2=r^2

y^2-(2a-1)y+a^2-r^2=0 かつ
y≧0
な。
441大学への名無しさん:04/10/09 18:15:51 ID:fBxeWPew
>>439
妙に判別式っぽいよね。
442大学への名無しさん:04/10/09 18:17:52 ID:WR8JY356
あとは>>440から古典的解の配置問題に帰着して終了。
443439:04/10/09 18:22:09 ID:44Xk8cHJ
>>442
この判別式ではx=0で接するときのを重解で求められないです
444大学への名無しさん:04/10/09 18:24:33 ID:WR8JY356
判別式じゃなくて、y≧0なんだからy=0でも接するじゃん。
445439:04/10/09 18:25:36 ID:44Xk8cHJ
判別式は使わないんですか?
446大学への名無しさん:04/10/09 18:26:03 ID:WR8JY356
判別式も使うよ。
447439:04/10/09 18:36:32 ID:u202S4Np
じゃあ放物線の内側にある円がx=0のみで接するとき
★の解が0と虚数ではなくて0と0以下の解になるのはなぜですか?
448大学への名無しさん:04/10/09 18:39:59 ID:WR8JY356
y≧0
なのに0以上の解なんて出る?
449大学への名無しさん:04/10/09 18:41:45 ID:WR8JY356
0未満だな。
450439:04/10/09 18:53:09 ID:u202S4Np
a>0とする。y=0を通るので★よりa=r。
この時★はy{y-(2a-1)}=0よりy=0、2a-1
2a-1>0のとき放物線と円は三点の共有点。
2a-1<0のとき放物線と円はy=0のみで接する。
こういう意味。
451大学への名無しさん:04/10/09 18:55:32 ID:WR8JY356
何が疑問なのかがわからない。。。
452大学への名無しさん:04/10/09 18:59:46 ID:WR8JY356
y^2-(2a-1)y+a^2-r^2=0 かつ
y≧0

y=0で接するとき、
a=rよって、
y^2-(2a-1)y=0かつ
y≧0
すなわち
y{y-(2a-1)}=0かつ
y≧0
2a-1≧0の時、
y≧0とは2点で接触し、
y=0 かつ y=2a-1
すなわち、y=0で接し、y=2a-1で共有点を持つ。
2a-1<0のとき、
y≧0とはy=0のみの共有点。
すなわち、
y=0で接するのみ。
453大学への名無しさん:04/10/09 18:59:53 ID:vhFt94p1
>>450
まず、a=rならば、x=0で円と放物線は接する。
接しても、yの方程式については重解ではないんだよ。
y=x^2だから、
y{y-(2a-1)}=0
は、xの方程式にすれば、
x^2{x^2-(2a-1)}=0
となって、きみの納得できる方程式になると思うが。
454大学への名無しさん:04/10/09 19:12:03 ID:WR8JY356
455大学への名無しさん:04/10/09 19:12:51 ID:WR8JY356
右がy軸な。
456大学への名無しさん:04/10/09 21:06:19 ID:XL+rvsUN
x^3(x-2)-(3x-2)(x-2)が(x-2)(x-1)(x^2+x-2)
になるのを教えて下さい。
457大学への名無しさん:04/10/09 21:14:05 ID:vr2RNyyW
チンコが勃起するのは何故ですか?方程式で説明してください。
458大学への名無しさん:04/10/09 21:19:05 ID:KPhtB1A4
http://www.mashiko-tcg.ed.jp/masityu/1/img-box/img20041009211549.jpg
茶のラインのあるところがわかりません。
なぜこうなるんですか?
459大学への名無しさん:04/10/09 21:26:26 ID:NUXSPrEm
>>458
両辺の対数をとっただけ
対数関数は単調増加関数だよ
460大学への名無しさん:04/10/09 21:27:15 ID:RjT1+jYa
log_{a}x=p が成り立つとき a^p=x
…指数関数の基本中の基本だぞ
461大学への名無しさん:04/10/09 21:29:04 ID:RjT1+jYa
すまん、指数関数じゃなくて対数関数だな
462大学への名無しさん:04/10/09 21:34:51 ID:KPhtB1A4
げ・・・どうもでした。>>459-461
463大学への名無しさん:04/10/09 21:35:19 ID:xJPFX8zH
>>459
底によるだろ > 単調増加
464大学への名無しさん:04/10/09 21:36:04 ID:KPhtB1A4
ですよね、0<a<1だとグラフが対称っすよね
465大学への名無しさん:04/10/09 21:39:18 ID:ECW9UVGx
二次方程式 ax^2+bx+c=0 (aは自然数、b,cは整数)
が重解をもつとき、
√(100*a+10*b+c)が整数となるような(a,b,c)の条件を求めよ

これは(1,2,1)だけですか?
466大学への名無しさん:04/10/09 21:39:45 ID:zDISEAqk
>>464
いや・・・・ちがう
467大学への名無しさん:04/10/09 21:40:21 ID:ECW9UVGx
一つ抜けたこれも(1,-2,1)
468大学への名無しさん:04/10/09 21:42:23 ID:zDISEAqk
>>465
(4,8,4)は?
469大学への名無しさん:04/10/10 00:00:05 ID:wuRrfObj
重解を持つとき方程式は
m(x+n)=0
とかける。このとき a=m, b=2mn, c=m(n^2)
よって
√(100*a+10*b+c) = √(100m + 20mn + m(n^2)
=√{m( n^2+20n+100)}
=√{m(n+10)^2}
=(n+10)√m
mが整数のとき√は整数となる。
k,lを整数とすると、求める(a,b,c)の組み合わせは(k^2 , 2(k^2)l , 2(kl)^2)
じゃなかろうか?
470大学への名無しさん:04/10/10 00:01:28 ID:wuRrfObj
書き間違えだらけだった
重解を持つとき方程式は
m(x+n)^2=0
とかける。このとき a=m, b=2mn, c=m(n^2)
よって
√(100*a+10*b+c) = √(100m + 20mn + m(n^2)
=√{m( n^2+20n+100)}
=√{m(n+10)^2}
=(n+10)√m
mが整数の二乗のとき√mは整数となる。
k,lを整数とすると、求める(a,b,c)の組み合わせは(k^2 , 2(k^2)l , 2(kl)^2)
じゃなかろうか?
471大学への名無しさん:04/10/10 00:03:13 ID:aEszWmAl
>>470
great!
472大学への名無しさん:04/10/10 00:14:05 ID:wuRrfObj
k,lを自然数とすると、求める(a,b,c)の組み合わせは(k^2 , 2(k^2)l , 2(kl)^2)
a,b,cが自然数だからこっちでよかったっぽ。
473大学への名無しさん:04/10/10 00:21:31 ID:TWvwdlB4
>>470>>472
解答乙。微妙なとこ補完しとく。
a は自然数で b,c は整数だから k は自然数で l は整数。
ラスト答えの c が違う。c=(kl)^2
あと計算途中ルート外すところで (n-10) ではなくて |n-10|
474大学への名無しさん:04/10/10 00:23:10 ID:TWvwdlB4
補完を間違えてどうすんだ漏れorz
n-10 でなくて n+10 ね。書かなくてもわかると思うけど…
475大学への名無しさん:04/10/10 00:44:07 ID:Y3PJFWck
三角関数についてなんですがsin,cos,tanの180度以上の値を
出すのに時間がかかります。単位円書いてやるかなれるかしかないんでしょうか?
476大学への名無しさん:04/10/10 00:57:47 ID:aEszWmAl
やってたら覚えるよ。
477大学への名無しさん:04/10/10 01:12:17 ID:TWvwdlB4
>>475
360°-θ の公式使って 180°未満に直せばよろし。
さらに 180°-θ の公式使って 90°未満に直せばよろし。
さらに 90°-θ の公式使って 45°未満に直せばよろし。

覚える方が早いが。
478大学への名無しさん:04/10/10 01:32:50 ID:wuRrfObj
>475
慣れてしまえば、頭の中に単位円が浮かんで、そこに三角形描いて、で求まると思う。
教えてる身の感覚だけど、頭のよしあしとは別に数学に不向きな人、図形イメージが
浮かばない構造の脳の人がどうしてもいる。
あなたが三角関数にまだ慣れていないなら、慣れるのを待ちましょう。
長いこと三角関数を使っていてパッと思い浮かばないなら、それはそれ。
単位円描いてもかかる時間はせいぜい一分未満ですから、着実に、あせらずが一番ですよー。
479344 355:04/10/10 02:07:01 ID:2PIUPIKP
問題あってて解けました
菱形ACBDの中点とCの距離をxとおくと
OC・OD=(√(5^2−x^2)−√(3^2−x^2))(√(5^2−x^2)+√(3^2−x^2))
      =25−x^2−9+x^2=16
480大学への名無しさん:04/10/10 03:06:10 ID:R7UHQNWB
>>470>>472>>473
間違い。
481大学への名無しさん:04/10/10 14:01:21 ID:4L+S/YYG
ロピタルの定理って入試の答案で使ってもいいんですか??
482大学への名無しさん:04/10/10 14:17:36 ID:EuXrf8ym
>>481
答え出すだけの問題ならいいけど、記述式の問題で使うのはよろしくない。
483大学への名無しさん:04/10/10 14:50:29 ID:VqzHM3iF
整数問題なんですが
Q.自然数nに対して、4^n + 6n -1が27の倍数となる条件を求めよ

数学的帰納法で、nが自然数のときに上の式が常に9の倍数になることはわかり、
さらに実験的にnが3m,3m-1(mは自然数)であれば成り立つこともわかったのですが、解答が書けません
よろしくおねがいします(´・ω・`)
484大学への名無しさん:04/10/10 15:12:16 ID:EuXrf8ym
>>483
4^n=(1+3)^nと考えて2項展開してごらん。
4^n=27の倍数+(n(n-1)/2)*9+3n+1
となることが分かるはず。後はできるでしょ。
485大学への名無しさん:04/10/10 15:29:50 ID:VqzHM3iF
>>484
そこで二項展開ですか、なるほど。

計算してみれば、(3^3){1+…+3^(n-3)}+(9/2)n(n+1)
となって、nが3m,3m-1(mは自然数)であれば確かに成り立ちますね(連続2整数の積が2の倍数になることは明らかなことですし…)

ご教授ありがとうございました。
486大学への名無しさん:04/10/10 16:07:33 ID:x5XAJd2E
b(n−1)−b(n)=(−1/2)^(n−1)
はどうやってb(n)=の形にするんでしたか?教えて下さい。
487大学への名無しさん:04/10/10 16:24:14 ID:VqzHM3iF
>>486
 b(n-1) - b(n)  = (-1/2)^(n-1)
+ b(n-2) - b(n-1) = (-1/2)^(n-2)
+ b(n-3) - b(n-2) = (-1/2)^(n-3)
        (中略)
+ b(2)  - b(3)  = (-1/2)^2
+ b(1)  - b(2)  = (-1/2)
__________________
 b(1)  - b(n)  = Σ(k= 1 〜 n-1) (-1/2)^k

あとは漸化式の計算だけっす。
488大学への名無しさん:04/10/10 16:35:13 ID:Gf9VtgBJ
b_(n+1)-b_n=-n/2 + 1/2
よって
b_(n+1)-b_1=-n(n+1)/4 + n/2
b_(n+1)-b_1=-n(n-1)/4
b_n=-(n-1)(n-2)/4 + b_1
489大学への名無しさん:04/10/10 16:36:09 ID:Gf9VtgBJ
あぁ、間違えた。まぁ、いいや。
490大学への名無しさん:04/10/10 16:52:12 ID:AyWuEhL4
1-1=日
491大学への名無しさん:04/10/10 16:53:46 ID:4L+S/YYG
>>482
そうか。記述で「ロピタルの定理より」ではだめなのか…。
じゃあ極限の計算は難しいものでも
微分の定義とかからなんとかだすしかないのか…。
492大学への名無しさん:04/10/10 16:54:56 ID:AyWuEhL4
1-1=日
493大学への名無しさん:04/10/10 17:09:02 ID:TWvwdlB4
>>491
lim[x→0](sinx)/x=1 とはさみうちだけ使えれば高校範囲の関数の極限はOK。
微分の定義やロピタルを使う方法は、知ってると得する程度のもの。と思うのだがどうよ?
494大学への名無しさん:04/10/10 17:59:34 ID:4L+S/YYG
>>493
いや例えばさ
lim[x→0](e^x-1)/xは微分の定義を使えば 
f(x)=e^x-1とおくとf'(x)=e^x

(e^x-1)/x=f(x)/x
=f(x)-f(0)/x-0→f'(0)=1 (x→0)
となるけど、微分の定義での解き方知らなきゃ解けないよな。

こうゆう問題は出ないと言ってしまえばそれで終わりだけど
教科に微分の定義はちゃんと書いてあるしさ。
入試範囲内だよなぁ。
それとも出題されるときは誘導がついてたりするんかな。
495大学への名無しさん:04/10/10 18:01:41 ID:V6ycMyNw
OA=3、OB=4である三角形OABの辺OBの中点をM、辺ABを1:3に内分する
点をDとし、線分OD、AMの交点をPとする。また、OA↓=a↓、OB↓=b↓とする。
このとき、
OD↓=3/4a↓+1/4↓である。
次に、直線AM上の点をQとし、OQ↓=sa↓+b↓(s,tは実数)とすると、
sとtの間には関係式 s+2t=1が成り立つ。

と、あるんですが、

sとtの間には関係式 s+2t=1が成り立つ。
というのは、どのように考えればいいのでしょうか?

OQ↓=s・a↓+1/2t・b↓/t+s
と、なるのは分かるんですが・・・。
496495:04/10/10 18:05:00 ID:V6ycMyNw
訂正です。

※誤 Q↓=s・a↓+1/2t・b↓/t+s

※正 Q↓=(s・a↓+1/2t・b↓)/t+s

宜しくお願いします。


497大学への名無しさん:04/10/10 18:52:23 ID:4L+S/YYG
>>495
OQ↓=sa↓+tb↓とすると
a↓=OA↓,b↓=OB↓=2OM↓だから
OQ↓=sOA↓+2tOM↓
今、点Qは直線AM上にあるから
s+2t=1


最後の2行がなんでそうなるか
分からないなんてことはないよな??
498大学への名無しさん:04/10/10 18:53:50 ID:vHLWWJAM
極限だろうがなんだろうが、教科書レベルはマスターしとけよ、最低限。
極限なんてそんなに難しくないんだしさぁ。
499495:04/10/10 19:44:17 ID:V6ycMyNw
>>497
その最後の2行がわかんないんです・・・。
教科書も見てみましたが、
わかんないです・・・・。
500大学への名無しさん:04/10/10 19:57:41 ID:YypPCqy/
>>499
点Qが直線AM上にあれば、AQ↓=tAM↓(tは実数)
となることはわかる?

これがわかってれば、
OQ↓=OA↓+AQ↓=OA↓+tAM↓
=OA↓+t(OM↓-OA↓)
=(1-t)OA↓+tOM↓

となって、各ベクトルの係数の和が1じゃなければならないことがわかるでしょ。これが内分点のベクトルの公式。
501大学への名無しさん:04/10/10 21:09:05 ID:OzAGeIue
係数の和が1って別に流してもいい所だとおもう
502大学への名無しさん:04/10/10 21:20:12 ID:4L+S/YYG
>>501
は??
ベクトル習う上での基本だしめちゃくちゃ大事なことだろ。
503大学への名無しさん:04/10/10 21:24:37 ID:vHLWWJAM
あたりまえのことすぎてそんなところで言い争うような部分ではないと思う。
504495:04/10/10 21:46:03 ID:V6ycMyNw
OQ↓=(1-t)OA↓+tOM↓になるのはわかるんですが、
a↓とb↓に直すと、
OQ↓=(1-t)a↓+1/2(b↓)となって
(1-t)+1/2t=1 となって、
(1-t)=sとおいて
s+1/2t=1 となって
2s+t=2 となるような気がするんですが、
どの部分の考え方が、間違っているんでしょうか?
505大学への名無しさん:04/10/10 21:47:39 ID:vHLWWJAM
>>504
なんで?
506大学への名無しさん:04/10/10 21:52:14 ID:JGeNF71i
平面上に20個の円がある。
それらのどの2個も2点で交わりどの3個も1点で交わらないとする。
これらの20個の円は平面を何個の部分に分けるか?
507495:04/10/10 21:59:28 ID:V6ycMyNw
>>505
OQ↓=(1-t)OA↓+tOM↓
から
s+2t=1
の導き方がわかんないんです・・・。
508大学への名無しさん:04/10/10 22:02:10 ID:YypPCqy/
>>504
>OQ↓=(1-t)a↓+(1/2)t(b↓)となって
ここから、
>(1-t)+1/2t=1 となって、
にいく過程が間違い。

>>497とよく見比べてごらん。
OQ↓=sa↓+tb↓
とおいたときに、sとtの関係がs+2t=1になるんだよ。
OQ↓=(1-u)a↓+(1/2)u(b↓) (紛らわしいから文字を変えただけ)
との対比では、1-u=で、(1/2)u=tとおいている。
uを消してみ。

509大学への名無しさん:04/10/10 22:03:18 ID:YypPCqy/
>>508
最後から2行目、1-u=sね。
510大学への名無しさん:04/10/10 22:06:46 ID:4L+S/YYG
>>504
OQ↓=(1-t)a↓+1/2(tb↓)となって
(1-t)+1/2t=1
が間違ってる。
OQ↓=(1-t)a↓+1/2(tb↓)
=(1-t)OA↓+1/2(tOB↓)
ここで直線AB上にQがあるとは誰も言ってない。
511大学への名無しさん:04/10/10 22:08:29 ID:4L+S/YYG
重なった…。悪い。
512495:04/10/10 22:19:26 ID:V6ycMyNw
>>508
>>510
なるほど〜、連立して解けばよかったんですね。
AQ:QMをt:(1-t)のようにおいていたから、分からなかったっぽいです。
じゃなく、u:(1-u)のようにおいて考えたら、あっさり分かりました。

どうも、ありがとうございました!
513大学への名無しさん:04/10/10 23:14:20 ID:YypPCqy/
>>506
2^20
514大学への名無しさん:04/10/10 23:47:39 ID:lIv3+ZN9
>>506
一見基本くさいけどいやらしい問題だな(w

円が1個のとき分けられた平面は2個
円が2個のとき平面は4個
円が3個のとき平面は8個だから
円が20個のとき平面を2^20個とする輩が多いんだよなあ。
ダメだよぉ。円が4個のとき平面は14個なんだから。ちゃんと書いてみ?

2 4 8 14・・・

階差を取ると、2 4 6・・・だから階差数列は初項2公差2の等差数列と予想(?)できる
円が20個のとき平面は2+2・1+2・2+・・・2・19=382個に分けられる。
でも予想の部分が論理的に説明できないとちゃんとした答案にならんよなあ(w
515大学への名無しさん:04/10/10 23:53:36 ID:HClCra4n
n個の円によって平面が分けられているところに
条件を満たすように(n+1)個目の円を書くと、
(n+1)個目の円は、それまでにあった円と2n個の交点を作る。
よって分けられる個数は2n個増える。

これでよさげじゃない?(ちょっと文章わかりにくいから誰か添削を。。)
516大学への名無しさん:04/10/10 23:54:44 ID:oyvXyCHJ
その予想も5個目以降で覆される思い込みかもよ?>>513みたいにね
517大学への名無しさん:04/10/11 01:19:47 ID:MpD/bVCE
1対1にでてくる複素数の折り返しって何に使うのですか?それを使う問題みたことないのですが私の演習不足でしょうか?高3東北理系です
518大学への名無しさん:04/10/11 04:37:20 ID:Ke1Fglg9
公式の質問です
円と軌跡
円(x-a)^2+(y-a)^2=r^2の周上の点(x0,y0)における接線の方程式

(x0-a)(x-a)+(y0-b)(y-b)=r^2

になるのがわかりません。
519大学への名無しさん:04/10/11 04:38:59 ID:Ke1Fglg9
まちがえた

×(x-a)^2+(y-a)^2=r^2の
○(x-a)^2+(y-b)^2=r^2の

520大学への名無しさん:04/10/11 05:28:18 ID:Axdx9l1i
x≧1のとき、xlogx≧(x-1)log(x+1)を示せ。

という証明問題なんですがアプローチの仕方がわかりません。
とりあえず
f(x)=xlogx-(x-1)log(x+1)
を2回微分してf(x)>0みたいな感じにしてみたんですが曖昧です。
よろしくお願いします。
521大学への名無しさん:04/10/11 05:48:52 ID:fvBzvZbl
>>518
x-a=X
y-b=Yとおく。
Y'=-X/Y
よって、
Y-Y0=-(X0/Y0)*(X-X0)
Y0Y+X0X=X0^2+Y0^2=r^2
よって
(x0-a)(x-a)+(y0-b)(y-b)=r^2
522大学への名無しさん:04/10/11 05:52:51 ID:fvBzvZbl
>>520
f'(x)=log(x)+1-log(x)-(x-1)/(x+1)={(x+1)-(x-1)}/(x+1)=2/(x+1)>0
f(1)=0より、
f(x)≧0
523大学への名無しさん:04/10/11 06:36:19 ID:abby85o8
>>520
 多少テクニカルだけど、
x≧1のとき、xlogx≧(x-1)log(x+1)を示せ。
 の両辺をx(x-1)で割っちゃって、
 f(x)=logx/(x-1) と見れば、左辺=f(x) 右辺=f(x+1) になる。
 んで、このf(x)が単調減少であることを示しちゃえばいいんでね?
524大学への名無しさん:04/10/11 10:08:11 ID:By5AQ72K
>>518
接線上の任意の点P(x,y),接点Q(x0,y0),中心C(a,b)とすると
常にPQ⊥QCですから、その内積=Oとして、内積の成分計算をすればいつかはその式になるんじゃない?
PQの傾き・QCの傾き=-1でもいいけど。
PQがx軸に垂直なときは、別に考察しないとダメなのでうざい。
525大学への名無しさん:04/10/11 10:20:21 ID:Qvyla2JZ
>>524
521のでいいんじゃないか??
526大学への名無しさん:04/10/11 10:23:17 ID:By5AQ72K
>>525
もちろん
でもね、>>521は数3の知識を使っているじゃないですか。
>>518が文系ということを考慮して2Bまでで理解できる方法を提示してまで。
527大学への名無しさん:04/10/11 12:36:03 ID:QJUTJF9A
実数x,yが(x−1)^2+(y−1)^2=1なる条件のもとで変化するとき、
2−2√2≦x+y≦2+2√2であることを証明する問題です。

お願いします><
できれば途中計算など細かく書いて頂けると助かります。
528大学への名無しさん:04/10/11 12:57:18 ID:tryk/vhY
>>527
x+y=k とおく。k の範囲を求めればよい。
y=k-x を条件式に代入すると(文字 k を含んだ) x についての2次方程式ができる。
この x についての2次方程式が実数解を持てばよい。

図を描いて求める方法もある。ここで図は描けないから説明しないが。
529520:04/10/11 13:39:37 ID:bMCOLXq/
>>522>>523
ありがとうございました。
>>523の両辺をx(x-1)で割るという発想に感動しました(´Д⊂
530大学への名無しさん:04/10/11 14:25:25 ID:Qvyla2JZ
>>522
>f'(x)=log(x)+1-log(x)-(x-1)/(x+1)={(x+1)-(x-1)}/(x+1)=2/(x+1)>0
f(1)=0より、  ↑↑↑
f(x)≧0    






f'(x)=log(x)+1-log(x+1)-(x-1)/(x+1)
=log(x)/(x+1)+2/(x+1)
ここでg(x)=(x)/(x+1)とおくと
g'(x)=1/(x+1)^2>0となりg(x)は増加関数
よってlog(g(x))も増加関数だから、f'(x)>0
f(1)=0よりf(x)≧0

じゃないか??


531大学への名無しさん:04/10/11 14:34:13 ID:jUmV0zNq
>>494
(e^x-1)/x
=(e^x-1)/[log{1+(e^x-1)}]
=[log{1+(e^x-1)}/(e^x-1)]^(-1)
=[log{1+(e^x-1)}^{1/(e^x-1)}]^(-1),
lim[x→0](e^x-1)=0(∵e^x:continius at x=0).
532大学への名無しさん:04/10/11 14:42:02 ID:ZAoyO7Rw
かなり初歩的な質問かもしれませんが、
「aの値を求めよ」「〜を示せ」とかいったときに、
どういう場合に、「逆に〜を代入すると成り立つ」という必要十分の書き足しが
必要なのかいまいちわかりません。どういう場合に必要なんでしょうか?
533大学への名無しさん:04/10/11 14:44:34 ID:jUmV0zNq
>>532
論証の各ステップで、同値変形でないところが一箇所でもあったら必要。
534532:04/10/11 14:52:02 ID:ZAoyO7Rw
例えば青チャの例題5でいう、x=0を代入すると、っていうところで必要になるんですか?
535大学への名無しさん:04/10/11 14:52:41 ID:abby85o8
>>534
 問題文全部写そうね。
536大学への名無しさん:04/10/11 14:56:03 ID:tryk/vhY
>>532
原則として必ず必要。書くかどうかの判断は、省略可能かどうかの判断。
明らかなことは省略可能。明らかでないことは省略可能でない。見る人にわかるように書く。
省略をするかしないかに関わらず必要十分性のチェックは必ずする。
迷ったら全部書いておいて問題ない。
537大学への名無しさん:04/10/11 14:57:17 ID:abby85o8
>>536
 初学者には余計に混乱招きそう。
538大学への名無しさん:04/10/11 15:01:29 ID:VkYjF+ZC
>>532
必ずしも必要では無いだろう?全部に書いても間違いじゃないけど。
論証が全て同値変形によっておこなわれていれば必要ない。
539538:04/10/11 15:44:30 ID:VkYjF+ZC
>>538のレスアンカー間違えた。
>>532ではなく>>536でした。
540大学への名無しさん:04/10/11 16:41:33 ID:fI+DByS4
どんな問題かによる
541大学への名無しさん:04/10/11 16:42:49 ID:/Io7CW6I
必要条件が足りないなら必要条件を書く。
十分条件が足りないなら十分条件を書く。
542大学への名無しさん:04/10/11 17:10:45 ID:Qvyla2JZ
>>531
そうか!解けるのか。
でも変形ムズいし、eの定義使ってるし
微分の定義使わなくても難しいな。
531さんすごいです!自力でこの解法思いついたんすか?
もしそうだったら尊敬します。



それより>>530ってあれで合ってますかね??
522は明らかに計算間違いだと思います。
543大学への名無しさん:04/10/11 17:32:59 ID:jUmV0zNq
>>542
>log(g(x))も増加関数だから、f'(x)>0
??
544大学への名無しさん:04/10/11 18:03:33 ID:6J3/+rPt
>>542

大学入試で(e^x -1)/x (x->0)を聞かれたら
「e^xのx=0における微分係数だからe^0=1」で十分
これを正解にしない大学はないよ

下手に回りくどいことを書くと,碌に回答も読まずに
「こいつはこんな基本的なことも判ってないのか・・・それじゃ×」
と思われかねんよ
545大学への名無しさん:04/10/11 18:05:20 ID:WRUyxQXp
>>544
入試なら理由無しで使っていいよ。
微分の定義なんていうから
そんなんいらんってつもりで>>531書いただけだよ。
546大学への名無しさん:04/10/11 18:23:50 ID:0YdgGddo
>>544
例えが悪くないか?

それが必要な場合には
「ただし(e^x -1)/x→1(x→0)は証明不要」
などと問題文に注釈がつくこともあるので
一概にはいえない微妙な問題だろう

教科書もごまかしの証明をするくらいなので
実際にこんな問いはありえないが
もし(sinx)/x (x→0)を聞かれたら
「sinxのx=0における微分係数だからcos0=1」で十分
…ということになってしまう

果たして十分か?
循環論法では?
547大学への名無しさん:04/10/11 18:24:53 ID:/Io7CW6I
sinxのx=0における微分係数だからcos0=1
それでただしいだろ。
548544:04/10/11 18:42:33 ID:6J3/+rPt
>>546
「それが必要」の「それ」がいったい何を指すのか意味不明だが...
要は数学的に正しい解答を簡潔明瞭に記述する限り
不正解にされることはない,ということだ

> 「ただし(e^x -1)/x→1(x→0)は証明不要」
> などと問題文に注釈がつくこともあるので
> 一概にはいえない微妙な問題だろう

その問題においては既知の事実として
証明をつけずに使ってよい,というだけのことだ

証明しろという設問に対しては正しく証明しないといけない

> 実際にこんな問いはありえないが
> もし(sinx)/x (x→0)を聞かれたら
> 「sinxのx=0における微分係数だからcos0=1」で十分
> …ということになってしまう
>
> 果たして十分か?
> 循環論法では?

三角関数を図形的に定義する場合は何らかの
循環論法がつきまとう

上の回答が循環論法的で気持ちわるいというのであれば
整級数を使って三角関数を定義する必要がある.
論理的に正しく書こうとすると微積の教科書の数節を
費やす内容になってしまう

試験時間内にそんなことはやってられんから
上の回答で十分だ
549大学への名無しさん:04/10/11 18:42:44 ID:PirpkSPp
>>547
結果は正しいんだけど、sinxの導関数を求めるときにsinx/x→1 (x→0)を使うから、論理としてはその論法は間違いなんだよ。
550大学への名無しさん:04/10/11 18:54:09 ID:0YdgGddo
>>548
要は本人が数学的に正しいつもりなだけの解答ということ
試験時間内にそんなことはやってられないだろうから注釈がつく場合がある
注釈がなければ>>531のようにすればよい
551大学への名無しさん:04/10/11 18:58:14 ID:6J3/+rPt
>>550
まあこれくらいで解放してやるが

>>544の解答が正しいことくらい大学入学までには理解しておいてくれ
552大学への名無しさん:04/10/11 18:59:21 ID:0YdgGddo
数学的には正しくなくても得点できる事実があるから十分だ、というならさもありなん
553大学への名無しさん:04/10/11 19:03:23 ID:6J3/+rPt
>>552

君,もしかしたら本当に544が正しいことが判らないの?

微分の定義を理解していないとか,exp xの導関数がexp xに
なることを証明できないとか,まさかその程度の理解度かい?
554大学への名無しさん:04/10/11 19:06:13 ID:/Io7CW6I
e^xのx=0における微分係数だからe^0=1

e^xの導関数は教科書の公式だから既知として良い。
555大学への名無しさん:04/10/11 19:10:09 ID:fI+DByS4
d/dx (e^x) = e^x は e^x の定義
lim[x->0] sin(x)/x = 1 は ラジアンの定義
556不死鳥@ささみ食えよ ◆OlSvCzhmFk :04/10/11 19:22:34 ID:updk9d7l
くだらない質問スマソ

(a/6)^2/3 = 2/3a√a

って成り立ちますか?
右が模試の解説と自分の書いた答えが違ってたんで。
557大学への名無しさん:04/10/11 19:25:11 ID:abby85o8
>>556
 2/3a√a って 2/3*a√a?ならない。
558大学への名無しさん:04/10/11 19:26:02 ID:Nw/LKk/6
>>556
両辺の対数とってみれ
559大学への名無しさん:04/10/11 20:14:40 ID:Qvyla2JZ
>>556
(a/6)^2/3=[3]√(a/6)^2

3乗根でルートの中身の6やaが2乗でしかないんだから
ルートの外に出せるはずがない。
560大学への名無しさん:04/10/11 20:17:58 ID:WO5W3YoO
9人を3組に分ける方法は何通りあるか、求めよ。

561大学への名無しさん:04/10/11 20:18:35 ID:GaAltm5U
>>560
青茶嫁。
562大学への名無しさん:04/10/11 20:50:25 ID:Qvyla2JZ
>>560
9C3*6C3*3C3か


釣られた。
すまん。
563高1:04/10/11 20:56:56 ID:16Am846t
9C3*6C3*3C3÷3
じゃないんですか?
564大学への名無しさん:04/10/11 20:58:20 ID:s3y5TZsl
積分しないで面積だけを一発で出す技√D^3/|a^2|

誰かこの詳細を教えてくれ…
565大学への名無しさん:04/10/11 21:09:40 ID:AHsXbjSi
|x^2+1|=|-3x^2+2|

これを解きたいんですが、両辺二乗しないとダメですよね?
566大学への名無しさん:04/10/11 21:14:06 ID:/Io7CW6I
|x^2+1|=|-3x^2+2|
x^2+1>0
-3x^2+2=-3(x-√(2/3))(x+√(2/3))
567大学への名無しさん:04/10/11 21:17:52 ID:AHsXbjSi
>>566
なるほど。ありがとう。
568大学への名無しさん:04/10/11 21:18:07 ID:ZRlsnGsc
円に内接する四角形ABCDがあり、
AB=AD=5、BC=10、cos∠ABC=4/5
をみたしている。
(1)対角線ACの長さを求めよ。
(2)cos∠ADCの値と、辺CDの長さを求めよ。
(3)辺BCの中点をMとするとき、三角形ACMと三角形ACDの面積比を求めよ。

模試の問題なんですが、答えをもらえなかったのでさっぱりでした・・・。
よろしくお願いします。
569大学への名無しさん:04/10/11 21:20:42 ID:/Io7CW6I
何が分からないのか分からない。
570大学への名無しさん:04/10/11 21:20:47 ID:QJUTJF9A
>>528
あいがとうございました!
571大学への名無しさん:04/10/11 21:26:32 ID:PirpkSPp
>>563
3人ずつ分けたきゃそんな感じ。
ちょっと惜しいけど。
572大学への名無しさん:04/10/11 21:31:20 ID:Asmk9H0F
>>568
たぶん基本が分かっていない。
悪いことは言わないから、ここで説明してもらうより、
中学の幾何と三角関数の復習をして理解した方がいい。
573大学への名無しさん:04/10/11 21:38:09 ID:Y9b0VJAo
空間上に6つの点をおく。どの四点も同一平面上にないと仮定する。
このとき、6つの点を適切に3つずつに分け、それぞれ、A,B,CとA',B',C'とすれば、
このとき、△ABCの周と△A'B'C'の内部が一点を共有し、△A'B'C'の周と△ABCの内部が一点を
共有するようにできることを示せ。
----
という問題が分かりません。教えてください。
574大学への名無しさん:04/10/11 21:40:04 ID:/Io7CW6I
図を書けば良いのかな?
575560:04/10/11 21:48:18 ID:WO5W3YoO
3組に分けるとは言ったが、誰も一組が3人だなんて言ってませんよ。>>560
576562:04/10/11 22:12:45 ID:Qvyla2JZ
>>575
そうか。すまん、早とちり。


じゃあ、9人を二つの仕切りで3組に分けると考えれば
8C2=28か??

577560:04/10/11 22:16:19 ID:WO5W3YoO
>568 3√5 -4/5 2 5:2
578560:04/10/11 22:19:35 ID:WO5W3YoO
>>576
それはA組B組C組のように区別した場合の数
579大学への名無しさん:04/10/11 22:22:41 ID:Asmk9H0F
>>573
どの4点も同一平面上にない6点は八面体をなす。
6頂点から面と一致しない三角形をなす3点を選びA,B,Cとする。
残りをA',B',C'とし、AA'は八面体の辺と一致しないものとする。
BCとB'C'はねじれの位置にあるから
1. BCがA'B'C'と共有点を持ち、かつB'C'がA'B'C'と共有点を持つ
2. ともに共有点を持たない
のどちらかになる。2.の場合AB'C'とA'BCについて考えると、
1.のような状況が成立している。
580大学への名無しさん:04/10/11 22:23:40 ID:PirpkSPp
>>578
本気で聞きたいなら、人を区別するのか、組を区別するのか、各組は1人以上属さないといけないのか、とかをはっきりしろ。
581大学への名無しさん:04/10/11 22:24:50 ID:hMxEFr8n
{3^9-3・(2^9-2)-3}/3!
582560:04/10/11 22:29:13 ID:WO5W3YoO
書かなくても分かると思ったからね。
玉などじゃなくて人間 だから、ふつう区別はする。3組に分ける だからふつう組は区別しない。
583大学への名無しさん:04/10/11 22:30:30 ID:/Io7CW6I
>>560
は放置で。
584大学への名無しさん:04/10/11 22:31:38 ID:4NAOQv/W
sappari wakaran orz
585大学への名無しさん:04/10/11 22:32:29 ID:Qvyla2JZ
つーか560は自分が分かってて書いてんだから
質問じゃないだろ。
スレ違い。
586560:04/10/11 22:32:30 ID:WO5W3YoO
>>581
thxあってるよ
587560:04/10/11 22:35:13 ID:WO5W3YoO
いや、試しただけ。どれくらいの人ができない・放棄するか
588560:04/10/11 22:38:13 ID:WO5W3YoO
>>583
放置じゃなくて わ か ら な い の間違いだろ。言葉間違ってるよ。
ちゃんと条件は書いてんだし、出来てる奴もいる。
589大学への名無しさん:04/10/11 22:38:51 ID:/Io7CW6I
>>587
問題もまともに掛けない奴は去ね。

これでも解いてろ。
0≦a≦1
0≦b≦1
x=a-b
y=a^3-b^3
で定まる点(x, y)のとりうる範囲を求めよ。

文系数学だよ。えらそうなこと言ってる前にさっさと勉強しろ。
590大学への名無しさん:04/10/11 22:41:12 ID:/Io7CW6I
簡単すぎたか。まぁいいや。
591560:04/10/11 22:42:00 ID:WO5W3YoO
>>589
どっかで見たぞ。お前それ2chで拾っただろ。
そういうお前みたいな目的でやった訳じゃない
592大学への名無しさん:04/10/11 22:42:23 ID:/Io7CW6I
>>591
オレが出した問題だからな。
593大学への名無しさん:04/10/11 22:42:57 ID:/Io7CW6I
2chで、リュードベリに。
594560:04/10/11 22:45:27 ID:WO5W3YoO
マンドクセ。難しくてもいいからすぐ終わるのにしれ
595大学への名無しさん:04/10/11 22:46:38 ID:/Io7CW6I
>>594
さっさと消えろ。
596大学への名無しさん:04/10/11 22:47:13 ID:cYMZAyQB
いちいち構うなよ。放置しろ
597大学への名無しさん:04/10/11 22:48:19 ID:1MtWQ/82
曲線y=√(x-1)上の点と曲線y=(x^2)+1上の点を結ぶとき、その2点間の
距離の最小値を求めよ。

お願いします...
598560:04/10/11 22:49:09 ID:WO5W3YoO
>>595
確認だけどう上の問題の出典言える?
嘘はよくないよ ウソ は。はったり君
599大学への名無しさん:04/10/11 22:53:14 ID:Y9b0VJAo
>>579
ありがとうございます。でも、
>どの4点も同一平面上にない6点は八面体をなす。
の部分が分かりません。例えば、凸包として四面体を考えた場合もこのようになるのでしょうか?
600大学への名無しさん:04/10/11 23:00:15 ID:/Io7CW6I
>>597
図を描いてみると対象性が見えるだろう。
凸性から直線y=xに最も近くなる点を結んでみてはどうだろうか。
601大学への名無しさん:04/10/11 23:05:19 ID:23r4gXNb
>>589は出典も何も有名問題だろ…
602大学への名無しさん:04/10/11 23:05:54 ID:0cqcB7Lg
>>597
その2曲線はy=xについて対称だから・・・
603560:04/10/11 23:06:04 ID:WO5W3YoO
簡単な計算も出来ない奴が何偉そうにアドバイスしてんだか・・
604大学への名無しさん:04/10/11 23:10:50 ID:/Io7CW6I
>>598
ちなみに東大文系数学の冬期講習テキストから。
605560:04/10/11 23:11:05 ID:WO5W3YoO
>>601
だろ?あたかもこいつID/Io7CW6I が出したみたいなハッタリ言ってたから
606560:04/10/11 23:13:11 ID:WO5W3YoO
>>604
なるほど、そこから自分が一番解けなそうなのを出した訳ね。
納得。
607大学への名無しさん:04/10/11 23:14:19 ID:/Io7CW6I
>>606
君には難しかったからそう感じるのかもね。
608大学への名無しさん:04/10/11 23:15:41 ID:/Io7CW6I
以下放置します
609大学への名無しさん:04/10/11 23:15:49 ID:cYMZAyQB
放置しろって。見たらわかるだろ。最初から嵐目的だこいつは
610560:04/10/11 23:15:57 ID:WO5W3YoO
君の気持ちは十分分かったから。もういいよ
611560:04/10/11 23:19:34 ID:WO5W3YoO
こんなとこに嵐も正統派もねぇよ。勉強せずに2chしてる時点で皆同類項
612大学への名無しさん :04/10/12 00:13:48 ID:KjBBHaSG
pを2以上の整数とする。2以上の整数nに対し、次の条件A、Bをみたす
複素数の組(z1,z2,...,zn)の個数をanとする。
A k=1,2...,nに対し、zk^p=1かつzk≠1
B z1z2...zn=1
このとき、次の問いに答えよ
(1)a3を求めよ。
(2)anを求めよ。

お願いします
613大学への名無しさん:04/10/12 00:31:45 ID:JNqpdEcU
>>599
1.6点が八面体をなす
2.5点が六面体をなし、内部に1点がある
3.4点が四面体をなし、内部に2点がある
に分けるべきでした、失礼。
1.は証明済みで、2.の場合について。
========================================
六面体をなす5点をABCDEとし、AEは六面体の辺でないとする。
6個目の点Fは四面体ABCDの内部とする。
線分AEは△FBC,△FCD,△FDBのいずれかと交わる。
△FBCと交わるとすると、△AEDと△FBCは題意を満たす。
========================================
3.の場合について
========================================
四面体をなす4点をABCDとする。
内部の点E,Fについて考える。
点Fは四ABCE,四BCDE,四CDAE,四DABEいずれかの内部。
四ABCEの内部とする。
線分DFは△ABE,△BCE,△CAEのいずれかと交わる。
△ABEと交わるとする。
△ABEと△CDFを選べば題意を満たす。
========================================
以上です。
614大学への名無しさん:04/10/12 00:46:14 ID:zqdzr7Dr
>>613 ありがとうございます。
615大学への名無しさん:04/10/12 00:46:32 ID:3ql/Zwjh
>>612
問題文正確に書いて。。
何をしたいのかわからん。。
pは任意の数なのか、それともAを満たすようなpが少なくとも一つ存在するのか。
複素数の組の数はnに関係なく1つだと思うが。。
616大学への名無しさん:04/10/12 00:48:06 ID:KjBBHaSG
誰か>>612お願いします。
よく分かりません
617大学への名無しさん:04/10/12 00:52:48 ID:KjBBHaSG
>>615
問題文そっくりそのままです。
見にくいところといったら zk^pのkは添え字です。またBは積です
618大学への名無しさん:04/10/12 01:02:54 ID:NE2padU5
>>612
arg(z_k)=(l/p)*2π (lは1〜p-1までの自然数)
Σ[k=1 to n]arg(z_k)=2mπ (mは整数)
となるような、z_kの組を求めりゃいい。

結局、1〜(p-1)までの自然数いずれかをとるn個の数で、足すとpの倍数になるようなもの、が何通りあるかを求めることになる。
619大学への名無しさん:04/10/12 01:07:32 ID:NE2padU5
a_2=p-1 は明らかで、
a_n=(p-1)^(n-1)-a_(n-1)
って漸化式が成り立つからa_nが求まるな。
620大学への名無しさん:04/10/12 01:15:17 ID:KjBBHaSG
うーん。題意すらまともに分からない俺は逝ってよしか・・
621大学への名無しさん:04/10/12 01:44:23 ID:pAVBwqQc
問:1から100までの整数について、3で割ると1余る数の和

これって「1」も含まれるんですか?
1÷3=0.3・・・余り0.1じゃないんですか?
622大学への名無しさん:04/10/12 01:54:25 ID:UjcddynG
>>1÷3=0.3・・・余り0.1じゃないんですか?
これがわけわかめだが、1も含まれるよ。 1 = 3*0 + 1
623大学への名無しさん:04/10/12 02:00:24 ID:gNuDbD8p
△ABCにおいて、点Aから対辺BCの中点Mに中線をひく。
∠AMBの二等分線をひき、その直線と辺ABとの交点をDとおく。
同様に∠AMCの二等分線をひき、その直線とACとの交点をEとおく。
このとき、DE<EC+DBを示せ。(中2内容です。教えてください。。。)
624大学への名無しさん:04/10/12 02:19:49 ID:JD0nPgcr
>>623

いろんな解答がありそう。

-----------------------------------------------------------------------
Cを通りBDに平行な直線を引き、【DB=CF…@】になるように点Fを取ると、
四角形BDCFは平行四辺形になるので、【DM=MF…A】

∠EME=1/2∠BMC=∠R とAより、
△EDFは、【DE=FE…B】の二等辺三角形

△EFCにおいて三角不等式より EF<EC+CF が成立するが、
@Bより DE=EF<EC+CF=EC+DB
-----------------------------------------------------------------------

後、細かい記述は面倒なので省くが
----------------------------------------------
MB=MB'になる点PをAM上に取ると、
△BMD≡△PMD → BP⊥MD → BD=PD が順に言える。
同様に、CE=PE も言える。
そして△PDEにおいて、DE<PE+PD=EC+DB
----------------------------------------------
みたいな流れもある。
625大学への名無しさん:04/10/12 02:29:30 ID:XzYrCsxP
>>621
それしちゃうと7÷3=2.3あまり0.1ってなって不適になっちゃうぞ
626大学への名無しさん:04/10/12 02:37:45 ID:JD0nPgcr
------------------------------------------------------
AM:BM=AD:BD, AM:CM=AE:CE だが
ここでは BM=CM なので、AD:BD=AE:CE よって DE//BC
DE=BC*a/(a+b), EC=AC*a/(a+b), DB=AB*a/(a+b)

△ABCにおける三角不等式より BC<AC+AB が成立するので
両辺 a/(a+b) 倍した DE<EC+DB も成立する。
------------------------------------------------------
627大学への名無しさん:04/10/12 02:54:24 ID:pAVBwqQc
>>622>>625
なるほど。そうですねw
オイラが馬鹿ですた。
628 ◆OlSvCzhmFk :04/10/12 09:24:25 ID:o4nQQb0l
確率についての質問です。
というか自分の解答の不備の指摘きぼんぬ。
【問題】
赤玉5個、白玉4個、青玉3個の入った袋から無作為に四個取り出す。
4個の中に赤玉、白玉、青玉が全て含まれる確率を求めよ。

模試の解説には 「赤2個 白1個 青1個」 「赤1個 白2個 青1個」 「赤1個 白1個 青2個」
のいずれかだから…として計算して答え 6/11 を導いています。
この解答は理解できたのですが、
自分の解答のどこがいけなかったのかが分からないので不備な点を指摘して欲しいのです。

自分は
(全事象)−(全部赤)−(全部白)ー(赤と白のみ。ただし全部赤と白は除く)
      ー(白と青のみ。ただし全部白は除く)ー(青と赤のみ。ただし全部赤は除く)
として495-5-1-120-34-37=298
よって答えは298/495 して必死に頑張ったのですが爆死しました。

あとできればこれに似たやり方での別解もよろしくお願いします。
629大学への名無しさん:04/10/12 09:27:29 ID:a1cfn8C0
「nの倍数」に0は含まれますか?
この前の模試の解答では含まれていたんで、いくつかの辞書や参考書で調べてみたんですが、定義はまちまちです。
「約数」の対義語だと考えると0は含まれないと思うんですが、問題解く上では異なったりするんでしょうか。
630大学への名無しさん:04/10/12 09:35:41 ID:KtsqUWS5
>>628
「青赤のみ」は、8C4-5=65じゃね?
そしたら合ってね?
631大学への名無しさん:04/10/12 09:51:49 ID:52fANuCx
>>629
倍数っていったら0も負も含むし、約数っていったら負の約数も含むんじゃね?
でも問題によっては、正の整数を全体集合としてる場合もあるんじゃね?
632 ◆OlSvCzhmFk :04/10/12 10:04:48 ID:o4nQQb0l
>>630
あ〜亜wせdrftgyふじこlp;@
部分点ゲッツ!
マジ感謝です。自分では気づきませんでした。
確率に対して絶望的になってたんですが少し救われました。
でも基本的には模試の解説の方針を先に思いつくようじゃないとダメなんだろうけど。
重ね重ねサンクス
633大学への名無しさん:04/10/12 11:39:48 ID:KjBBHaSG
正の数x,y,zが a/x+b/y+c/z=1 をみたしながら動くとき、x^2+y^2+z^2の最小値を求めよ。

お願いします
634大学への名無しさん:04/10/12 11:41:36 ID:KjBBHaSG
a,b,cを正の定数とします>>633
635大学への名無しさん:04/10/12 11:56:17 ID:KjBBHaSG
整数p,qは0<p<q, pとqは互いに素 をみたすものとする。
数列{a_n}を a_n=pn/q-[pn/q] (n=1,2,3,・・) で定める。
ただし、[x]はxをこえない最大の整数を表す。また、 S_n=納k=1 to n] とする。
このとき、lim S_n/n を求めよ。

これもよろしくお願いします
636大学への名無しさん:04/10/12 11:58:16 ID:KjBBHaSG
↑訂正 S_n=納k=1 to n]a_k です。
637高2:04/10/12 12:25:35 ID:6wk1/yb+
期待値の問題を復習していたのですが、わからないところがでてきましたあ〜・。.・(⊃д`).・。.・教えて先輩方!!
問)2つのサイコロをふり、出た目のうち、小さくない方をXとするとき、Xの期待値を求めよ。

なんで小さくないなんて変な言い回ししてるんだあ( ´Д`)
638大学への名無しさん:04/10/12 12:43:10 ID:3gJC13g3
>>637
同じものが出たときの事を考慮してのこと。
AとBの組み合わせは
11 22 33 44 55 66
12 23 34 45 56
13 24 35 46
14 25 36
15 26
16
とその逆なんだから、21*2/6^2=7/6となるわけだ。
639大学への名無しさん:04/10/12 12:49:06 ID:3gJC13g3
あ、ごめん、間違えた。てか何してんだ。
ちゃんと書かなきゃな。
11 12 13 14 15 16
21 22 23 24 25 26
31 32 33 34 35 36
41 42 43 44 45 46
51 52 53 54 55 56
61 62 63 64 65 66
のうち、一番左上が1
左上2×2から1になるものを除くと、3つが2
……(2n-1個がnになることが分かる)……
1*1+2*3+3*5+4*7+5*9+6*11=161
よって、
161/36が期待値。
640大学への名無しさん:04/10/12 13:31:52 ID:KjBBHaSG
T a,b,cを正の定数とする。
正の数x,y,zが a/x+b/y+c/z=1 をみたしながら動くとき、x^2+y^2+z^2の最小値を求めよ。

U 整数p,qは0<p<q, pとqは互いに素 をみたすものとする。
数列{a_n}を a_n=pn/q-[pn/q] (n=1,2,3,・・) で定める。
ただし、[x]はxをこえない最大の整数を表す。また、 S_n=納k=1 to n]a_k とする。
このとき、lim[n→∞] S_n/n を求めよ。

誰かお願いします
641高2:04/10/12 13:41:19 ID:6wk1/yb+
>>638さん
ありがとうございます(>_<) 小さくない方とは、同じ目の場合も考慮する事なのですかー。
学びました☆
642大学への名無しさん:04/10/12 15:54:20 ID:pFdw8zOg
aベクトル= ( L,-2,3 )
bベクトル= ( 3,M,1 )
cベクトル= ( 2,5,N )
a,bベクトルは始点が同じであり、M>0である

上記の3つのベクトルが三角形を作るときL,M,Nを求めよ
643大学への名無しさん:04/10/12 16:16:57 ID:I23g5f23
(純虚数)≠0

ですか?
644大学への名無しさん:04/10/12 16:20:15 ID:tgZrGNAG
>>643
0は実数だろがあ〜!
645大学への名無しさん:04/10/12 16:28:54 ID:Ye4lTmwc
>>644
「何聞きたいのか分からん」って答えてやるほうが>>643のためのような気もするけどね。
646大学への名無しさん:04/10/12 16:39:36 ID:GjZPKWrS
>>643
複素数とは a+bi (a,b は実数) という形で表される数のことを言います。a を実部、b を虚部と言います。
この複素数の中で特に a+0i という形をしたものを、実数 a と同一視して実数とみなします。
この同一視を通して考えることで、複素数は実数を含むといえるわけです。
そして、実数でない複素数のことを単に虚数と言います。すなわち虚部 b≠0 であるような複素数 a+bi が虚数です。
この虚数の中で、実部 a=0 となるもののことを純虚数と呼びます。
したがって純虚数とは a=0 かつ b≠0 であるような複素数 a+bi のことです。
といったことは教科書に書いてあるはずですので、自分できちんと調べてから聞くようにしましょう。
647大学への名無しさん:04/10/12 17:54:24 ID:cr7CilPK
同一視の説明が少しおかしいね。
648大学への名無しさん:04/10/12 18:41:20 ID:cJcWO0I0
>>640
I
めんどくさそうだな…。大学以上になればこんなのはラグランジュの未定乗数法っていうので解くんだけど…。
(a^(2/3)+b^(2/3)+c^(2/3))^3になると思う。

U
a_nは、pk+1≦n≦pk+(p-1)の範囲で、1/q〜(p-1)/qを1回ずつ取る。
まず、これを示すこと。後は求まるだろ。

649大学への名無しさん:04/10/12 18:43:08 ID:d6RRueR0
nが2よりも大きな自然数だったら
X^n+Y^n=Z^n
を満たすような自然数X、Y、Zは存在しない

という問題はどうやれば証明できますか?
650大学への名無しさん:04/10/12 18:44:42 ID:cJcWO0I0
>>648
U a_nは、qk+1≦n≦qk+(q-1)の範囲で、1/q〜(q-1)/qを1回ずつ取る。
に訂正。スマソ

651大学への名無しさん:04/10/12 19:57:34 ID:21F2bxl4
>>649
複素数を使った面白い証明法を思いついたが、紙が足りないのでここに書くことは出来ない。
652大学への名無しさん:04/10/12 20:02:30 ID:o+LjW1iR
>>649
俺も思いついたが、ここに書くには余白が少ない
653大学への名無しさん:04/10/12 20:42:40 ID:UjcddynG
>>649
一生を数学のためだけに捧げれば証明できるかもしれません。
654大学への名無しさん:04/10/12 20:51:27 ID:t6YRsMEx
>>649
もう少しで思いつきそうだったのに・・・
655長助:04/10/12 21:37:03 ID:KNzVZEFR
>>649
高校の範囲からは少し外れるのですが、つぎを読んでみると良いと思います。

A. Wiles : Modular elliptic curves and Fermat's Last Theorem, Ann.Math.142(1995),443-551
http://math.stanford.edu/~lekheng/flt/wiles.pdf
656大学への名無しさん:04/10/12 21:49:50 ID:y7F7B/B8
>>642
三角形をつくるので、
a-b=cまたはb-a=cが成立
Mの条件より後者のみ採用
657大学への名無しさん:04/10/12 22:33:13 ID:/Mfx282E
2円x^2+y^2+lx+my+n=0,x^2+y^2+px+qy+r=0が交わるとき
その二つの交点を通る円は
x^2+y^2+lx+my+n+k(x^2+y^2+px+qy+r)=0 (k≠-1)

がわかりません。このkってなんですか?二つたすのはなぜ?
また、この公式を使わないでとく方法ありませんか?
658大学への名無しさん:04/10/12 22:35:56 ID:YHpBmLEm
>>657
別にその公式使わなくても、2円の交点の座標を求めれば解けるよ。
それを一度やってみれば、その公式のありがたみがわかるであろう。
659大学への名無しさん:04/10/12 22:44:44 ID:21F2bxl4
加減法の基礎定理より、

x^2+y^2+lx+my+n=0かつ
x^2+y^2+px+qy+r=0

x^2+y^2+lx+my+n+k(x^2+y^2+px+qy+r)=0 (k≠-1)
かつ
x^2+y^2+px+qy+r=0

かな。
660大学への名無しさん:04/10/13 04:04:26 ID:jyeX/VAI
(A-1)a+Ab=0
(A-1)b+Ac=0
(A-1)c+Aa=0

Aを求めたいのですが、どのようにすれば求められますか?
661大学への名無しさん:04/10/13 04:27:46 ID:Zts97Q+I
>>660
左辺全部足して=0とすると
(2A-1)(a+b+c)=0
∴a+b+c≠0のときA=1/2
 a+b+c=0のときAは一つに定まらない.
662大学への名無しさん:04/10/13 09:51:06 ID:ikm11bRP
三角比の問題なんですが

三角形ABCの頂点A、B、Cから対辺へ下ろした3つの垂線の長さ
がそれぞれ15,12,10である。いま、3辺の長さを
BC=a、CA=b、AB=c
とするとき、
a:b:c=4:5:6である。
この時cosAの値とa、b、cの値を求めろ

【解答】
三角形ABCに余弦定理を適用して
cosA=5^2+6^2−4^2/2・5・6=3/4

a、b、cの値はcosAを利用してSinA=√1−cosA^2=√7/4・・・・・・・@
ところで、
∠AEB=90゜なので、 BE/AB=SinA・・・・・・・・・・・・A
∴c=AB=BE/SinA=48/√7・・・・・・・・・・・B
a=4/6c、b=5/6cより
a=32/√7、b=40/√7

で、分からない所が@、A、Bなんですが
@はcosをSinに変換する理由がわかりません。
Aはなぜ、BE/ABがSinになるのか全く分かりません∠AEBが90゜ならcosになると
思うのですが。
Bはc=ABまでは分かるのですが、正弦定理の
BE/SinAとイコールになる理由が分かりません。
よろしくお願いします。
663大学への名無しさん:04/10/13 11:08:25 ID:wRqXinjA
具体的な問題の質問じゃないんですが
自分、一浪なんですが最近気づきました。
計算力が欠けてると思うんです。
たまにココでも質問させてもらうのですが
方針はあってるのに四則計算で間違えるんです。
和田秀樹の本で計算力テストやったのですがボーダーラインで
「中学の参考書をやりなおすといい」みたいな感じなんです。
今、中学の頃やりまくってた参考書やってるんですが
計算力アップにはどうしたらいいですか?
このまま中学の計算をやってるのでいいのでしょうか。
時間的にも本来ならばこんなことやってる時期じゃないんですが
664大学への名無しさん:04/10/13 11:44:11 ID:zsekKUoC
2浪汁
665大学への名無しさん:04/10/13 11:52:27 ID:zGhRmM0Z
0≦χ≦π、0≦у≦π、k>1のとき、
-k≦cosχ≦k
は、どうして
0<y<πになるんですか?だれか教えてください。
666大学への名無しさん:04/10/13 12:40:53 ID:wRqXinjA
663ですが
664さんの言葉は最高のやる気になりました
二浪はゆるされません
667大学への名無しさん:04/10/13 13:20:32 ID:yWfgqcqh
>>663
まずは和田の方法論の間違いに気付こうな。
668大学への名無しさん:04/10/13 15:12:00 ID:wx6KimiQ
Y=e^-xの微分ってどうなるんですか?教科書にかいてないんです
669大学への名無しさん:04/10/13 15:25:51 ID:F7mx2IyK
>>668
合成関数の微分法は教科書に載ってるよな?
670大学への名無しさん:04/10/13 15:41:46 ID:wx6KimiQ
>>669
載ってますが、eについては書いてないんです。同じように-1をかければいいんでしょうか?なにぶん独学なもので。
671大学への名無しさん:04/10/13 15:54:11 ID:2YZCZHu4
f(x)=e^x,g(x)=-xとすると
f(g(x))=e^(-x)
まったく同じ問題が載ってないとできないってのはちとあれかと思われ
672大学への名無しさん:04/10/13 15:58:56 ID:sgvDDtkz
ま、独学だと最初はツライと思うよ

>>662
sinは、正弦定理の方じゃなくて、斜辺/対辺っていう定義の方を使ってるんだよ。
忘れたころに出てくる解法だから、覚えておくといいかも

(2)でcosじゃないのは、cosだと、垂線によって角が割れてしまい、うまく使えないから。
673662:04/10/13 16:52:49 ID:9iUL2gFy
>>672
レスありがとうございます。
>sinは、正弦定理の方じゃなくて、斜辺/対辺っていう定義の方を使ってるんだよ。
その定義は知ってます。cosが斜辺/底辺ですよね?
斜辺以外の底辺、対辺は直角の位置によって変わるという解釈でいいですか?
あとこの長さを求めるにはcosに変換しなければいけないのでしょうか?
お願いしますm(..)m

今年受験で一通り全部公式も覚えて問題集をしてて
数学はセンターのみなんですがこの程度の問題は解けないとヤバイでしょうか・・?
674662:04/10/13 16:55:18 ID:9iUL2gFy
あ、あと
>垂線によって角が割れてしまい
っていうのはどういう事でしょうか?お願いします
675大学への名無しさん:04/10/13 17:33:12 ID:sgvDDtkz
お絵かき掲示板が使えない・・・

>>673
>斜辺以外の底辺、対辺は直角の位置によって変わるという解釈でいいですか?

うん。そんな感じ。三角比は、あくまでも角が主体なんだよね。
直角三角形があったときに、ある一つの角(直角で無い角)に注目。このとき、この角の
正面(この角が接してない角)にある辺が対辺で、斜辺と違う側にある接している辺が底辺。

今回は、解答から考えるに、
     B


A    E   C

みたいな位置関係だよね。ここで、直角三角形ABEに注目。
角Aのcosの値が分かっているから、角Aに注目してみる。
角AはABとAEに接しているから、△ABE(辺はAB,AE,BE)において、
角Aの対辺(Aと接してない辺)はBE
斜辺はABだから、底辺は残ったAE。

このときcosA=AE/AB
cosAは求められているけど、AEの長さは分かってない。ABも分かってないから
未知数はAEとABの二つ。けれども式は一つだからコレは解けない。
だからcosは使えない。

次にsinに注目してみよう
sinA=BE/AB
問題文中にBEの長さは与えられている。ここまでで、未知数はsinAとABの2つ。
しかし、sin^2(A)+cos^2(A)=1を使えば、式2つになるから解ける。
676大学への名無しさん:04/10/13 17:40:04 ID:sgvDDtkz
>今年受験で一通り全部公式も覚えて問題集をしてて
数学はセンターのみなんですがこの程度の問題は解けないとヤバイでしょうか・・?

ちょうど、センターがこの程度の問題レベルのはずだよ。
今ヤバイかどうかより、今からどうやって過ごすかの方がはるかに大事。
別に今すごくよく出来てたとしても、例えば1〜2ヶ月全く勉強しなければ、力なんて簡単に落ちるんだからさ
(大学一年生の人に遭ってみると、わかるかも)

現役なら、
・問題になれること
・解法を整理すること
・弱点を認識して対策をとること
あたりが冬までにしておくことだと思う。
677大学への名無しさん:04/10/13 17:55:04 ID:syQ258Ls
678大学への名無しさん:04/10/13 18:46:13 ID:avauGVKq
AB=3,BC=5,CA=4の三角形ABCの重心をGとする。AB,AC上に2点P,RをそれぞれベクトルAP=αベクトルAB、ベクトルAR=βベクトルAC(0≦α1、0≦β≦1)にとる。さらに点Qを三角形PQRの重心がGに一致するようにとる。
679大学への名無しさん:04/10/13 18:50:07 ID:avauGVKq
>678
続きです。
(1)ベクトルAQをベクトルAB,AC,α,βを用いて表せ
(2)QがBC上にあるとき、三角形PQRの面積の最小値を求めよ

数学苦手でまったくわかりません。どなたか解説をお願いできますでしょうかm(__)m
680大学への名無しさん:04/10/13 18:52:30 ID:9tV46Dy1
>>679
αとかβとはベクトルの記号にしてはキモイ記号だな

そこのところはtとsと相場が決まってるんだぜ
681大学への名無しさん:04/10/13 19:07:43 ID:9tV46Dy1
>>679
まぁ △ABCの重心=△PQRの重心
の式を立てればQが出るし

面積は苦手だと全然わからんレヴェルなのでおれもわからんよW
682大学への名無しさん:04/10/13 19:08:50 ID:avauGVKq
すみませんm(__)m無知なもので…しかし、問題をそのまま書き写しました。
683大学への名無しさん:04/10/13 19:11:26 ID:sgvDDtkz
>>678
ベクトルの基本
1.起点となる点を決める(問題中にベクトルが与えられていれば必要なし)
2.起点から始まるベクトルを2本(空間なら3本)決める(大概は問題文中で与えられるが)。
これを基本ベクトルとでも呼ぶとする
3.問題文に出てくる点を、2.で決めた基本ベクトルで表していく(未知数を使っていってOK)
4.辺の長さが与えられていれば、内積の値を出しておく
5.問題文中の条件(「垂直」とあれば内積ゼロ。「線分〜〜上」とあれば係数足して1 など)
を使い、4.で設定した未知数を消していく


684大学への名無しさん:04/10/13 19:11:46 ID:sgvDDtkz
>AB=3,BC=5,CA=4の三角形ABCの重心をGとする
ふんふん と読んで行く

>AB,AC上に2点P,RをそれぞれベクトルAP=αベクトルAB、ベクトルAR=βベクトルAC(0≦α1、0≦β≦1)
ベクトルAB(以下bと書く)とベクトルAC(以下cと書く)を基本ベクトルにしよう

>AB=3,BC=5,CA=4の三角形ABCの重心をGとする
出てきている点はA,B,C,G。これらを基本ベクトルで書いていく。
Aは起点なので無視。Bはb。Cはc。
Gは重心なので(「重心」を見たときに1/3公式を想いだす)ベクトルAG≡g=(b+c)/3

>AB,AC上に2点P,RをそれぞれベクトルAP=αベクトルAB、ベクトルAR=βベクトルAC
そのまま、p=αb,r=βcと書いちゃう

>さらに点Qを三角形PQRの重心
三角形PQRの重心は(p+q+r)/3

>三角形PQRの重心がGに一致する
(p+q+r)/3=g=(b+c)/3
p=αb,r=βcだから
αb+q+βc=b+c
だから、q=(1-α)b+(1-β)c
685大学への名無しさん:04/10/13 19:13:52 ID:Ogou+2BU
>>679
1)Aを原点AB方向をy軸AC方向をx軸とした座標を取って、重心の公式を用いて重心の位置が一致するという条件を数式で表せ。
2) 1)からQの位置が分かるので、QにBC上である条件を代入し、PとRの位置を考え、面積を求めよ。
686大学への名無しさん:04/10/13 19:31:00 ID:sgvDDtkz
>(2)QがBC上にあるとき
「AQ=□AB+△ACのとき、□+△=1」を思い出して(線分上にあるとき、係数足して1)、
(1)より(1-α)+(1-β)=1
⇔α+β=1

問題中に辺の長さが与えられているから内積を出しておく
角A=90度より
b・c=0

以下、何通りか解き方があるけど、とりあえず、力技の方で。
三角形PQRの面積=1/2*√(QP^2*QR^2-ベクトルQP・ベクトルQR)・・・(I)
ベクトルQP=ベクトルAP−ベクトルAQ=(2α-1)b-(1-β)c
ベクトルQR=ベクトルAR−ベクトルAQ=(2β-1)c-(1-α)b

QP^2=((2α-1)b-(1-β)c )^2=9(2α-1)^2+16(1-β)^2
QR^2=((2β-1)c-(1-α)b)^2=16(2β-1)^2+9(1-α)^2
ベクトルQP・ベクトルQR=9(2α-1)(1-α)-16(2β-1)(1-β)

上3つを(I)に代入して、α+β=1でβを消去してαだけの一変数関数にした後、
あとは0≦α≦1のもとでの最小値を出しちゃってください
687大学への名無しさん:04/10/13 19:43:53 ID:sgvDDtkz
ベクトルを使わないなら、α+β=1の後、
AP:PB=CR:RA=BQ:QC=α:1-α
を使って(q=(1-α)b+αcとなるから、QはBCをα:1−αに内分する点)
面積を出してもOK
688大学への名無しさん:04/10/13 20:40:22 ID:avauGVKq
みなさん解説ありがとうございましたm(__)m尊敬します
689国文・浪人:04/10/13 23:03:17 ID:sDr23hGt
現在センター数学対策中の浪人です。

河合パーフェクト問題集数学2Bの
空間ベクトル【89】で出てきたところなんですが、
途中計算の所なので問題は省いています。

http://castaff2620.hp.infoseek.co.jp/cgi-bin/img/17.jpg

因数分解か平方完成するんだと思うんですけど係数がでかくてよくわからないのです。
過程を詳しく教えてください、おねがいします。
690大学への名無しさん:04/10/13 23:06:06 ID:6KLB52T5
そんな事だから浪人するんだよ。
もう1年浪人しろ。
691大学への名無しさん:04/10/13 23:23:43 ID:XCMtBqj7
>>689
お前絶対自分で解こうとしてないだろ?
読んでるだけじゃねえの。手を動かせ。
ちゃんと手を動かせ。
ただの平方完成だよ。
係数が大きくてわからないなんてことありえない。
わからないじゃなくて出来ないならまだ少しはわかるが。
692689:04/10/13 23:27:50 ID:sDr23hGt
>>690
>>691
すみません。690でガーンときて、もう一度自分でやってみたらできました。
因数分解にこだわっていてできなかったみたいです。平方完成したらできました。
どうもすみませんでした。
693大学への名無しさん:04/10/14 05:26:12 ID:794JBc2c
導関数についての質問なんですけど
ある関数f(x)について、たとえば f'(3)=2は x=3におけるf(x)の傾きは2ってことを表しますよね?
じゃあ f''(3)=2 の2って何を表すんでしょうか?
694大学への名無しさん:04/10/14 05:52:37 ID:boxBvGCz
3角形ABCの内接円をOとします。円Oと3辺BC,CA,ABとの接点を
P,Q,Rとします。線分AQ,AR,円Oに同時に接する円と円Oとの接点をL,
線分BR,BP,円Oに同時に接する円と円Oとの接点をM,線分CP,CQ,円O
に同時に接する円と円Oとの接点をNとします。このときPL,QM,RNは一点で
交わることを証明して下さい。


695大学への名無しさん:04/10/14 06:47:09 ID:Mh1dJB/N
>>693
f'(x) の x=3 での傾き.または f(x) は x=3 の十分近くで f(3) + f'(3) x + f''(3)/2 x^2 に近似出来るから,
f''(3) = 2 は f(x) を x=3 の十分近くで二次式に近似すると x^2 の係数が 1 になることを表してる.
696大学への名無しさん:04/10/14 11:10:12 ID:s5/FEDBL
チンコが赤くなるのは何故ですか?
697大学への名無しさん:04/10/14 13:59:28 ID:brsx1EQd
法政の社会学部志望なんですが、今青チャが途中まで終わってるんですけど文系は黄チャの方がいいと言われました。黄チャに乗り換えた方がいいですか?
698大学への名無しさん:04/10/14 14:07:33 ID:brsx1EQd
すみません。スレ違いでした。
699662:04/10/14 15:20:40 ID:SPU6tF0E
>>675>>676
有難うございます。お陰でよく分かりました
つまり
     B


A(θ)    E   C

で考えればいいってことですね?
700大学への名無しさん:04/10/14 15:32:10 ID:xWknzFQn
>>697
青ちゃが難しいと感じているなら黄ちゃ
じゃなきゃそのまま池
701大学への名無しさん:04/10/14 15:50:14 ID:794JBc2c
695
ということは、2という数字自体は編曲点とは関係ないんでしょうか?
702大学への名無しさん:04/10/14 16:06:05 ID:zDYv1sgO
>>701
横レスだけど、変曲点に関係あるのはf''(x)=0だけでしょ。
他の点でf''(x)求めて何がうれしいかって言われたら、
下に凸か上に凸かが重要な情報になる問題では役立つ。
703大学への名無しさん:04/10/14 16:21:51 ID:Vr+xJzCT
お願いします

半径が1の3つの円柱があり、軸は原点Oにおいて互いに直行している。
このとき、3つの円柱の共通部分の体積Vを求めよ。
704大学への名無しさん:04/10/14 16:37:25 ID:794JBc2c
>695 >702
そうかぁ、やっとわかりました。わざわざ答えてくれてありがとうございました。
705大学への名無しさん:04/10/14 16:46:23 ID:5PFw1DBU
立方体を5色の色を使って隣り合う面が同じ色にならないような塗り方は
なん通りありますか?
706大学への名無しさん:04/10/14 16:55:12 ID:zDYv1sgO
>>703
高校の知識で出せるのか謎。
とりあえず図は描けた?
>>705
3色---1通りずつ
4色---1色を3回使うか、2色を2回ずつ使うか
5色---2回使う色はどれか、あとは数珠順列
707大学への名無しさん:04/10/14 16:56:21 ID:Vr+xJzCT
130
708大学への名無しさん:04/10/14 16:59:22 ID:5PFw1DBU
>>705
すべての色を使います。
答えもよろしくお願いします
709大学への名無しさん:04/10/14 17:01:57 ID:zDYv1sgO
>>708
5×3!=30かな?
答え聞いちゃったら面白くないと思うけど。
710大学への名無しさん:04/10/14 17:02:00 ID:n2TX2gl7
>>703
http://www.fukkan.com/vote.php3?no=3629
の「立体の捉え方」に、図、発想法、計算、答えともに書いてる
711709:04/10/14 17:03:30 ID:zDYv1sgO
ごめん、30÷2=15だわ、多分。
712大学への名無しさん:04/10/14 17:08:00 ID:Vr+xJzCT
答えは自分なりに出しました。16-8√2になった
713大学への名無しさん:04/10/14 17:09:26 ID:F3if4JYp
>>712
正解
714大学への名無しさん:04/10/14 17:11:24 ID:Vr+xJzCT
>>706の言ってるような立体の概形を考えるんじゃなくて
断面積しか考えてないんですが・・
715大学への名無しさん:04/10/14 17:15:47 ID:zDYv1sgO
>>714
よく出たねえ。立派なもんです。
716大学への名無しさん:04/10/14 17:20:15 ID:nX8VVJuE
>>705の問題の答えは15ですか?
717大学への名無しさん:04/10/14 17:33:20 ID:c/UhEf9M
x≧0、y≧0において
C(x+y)≧2√xy…Aとするとき(Cは正の数)
(1)C≧1のときAが成り立つことを示せ
(2)Aが成り立つときC≧1であることを示せ
(3)√x+√y≦K√x+yを満たす最小のKを求めよ

わかりません(´д`)
だれか教えてくださいm(_)m
718大学への名無しさん:04/10/14 17:41:59 ID:+wELKe6S
>>716
そうだよ。

>>717
√xy=√(xy)
√x+y=√(x+y)
だと思うが、分かるように書いたほうがよい。

719717:04/10/14 17:48:00 ID:c/UhEf9M
>>718
スイマセン(>_<)
720大学への名無しさん:04/10/14 17:55:24 ID:yjC9rFg0
>>703
題意がようわからん。
円柱の中心軸と座標軸が一致してるってことか?

それなら、
座標をx,y,zとおいて
x^2+y^2≦1
y^2+z^2≦1
x^2+z^2≦1
x-y,y-z,x-z平面のどれで切っても同じ切り口になりそうだから
例えば、x=t,y=t,z=tのどれかにおける切り口の面積を求めりゃいいだろうね。
まぁz=tで切るなら
x^2≦1-k^2
y^2≦1-k^2
x^2+y^2≦1か。
この3つの不等式で囲まれる領域の面積を求めればいい。
正方形の大きさを考えて場合わけするべし。

こういう問題が出てきたら、まずは図形の不等式を作って
求めやすい平面で切ることを考えよう。
721大学への名無しさん:04/10/14 18:24:05 ID:c/UhEf9M
誰か>>717お願いします泣
722大学への名無しさん:04/10/14 18:34:36 ID:n2TX2gl7
√2かな
違うかも
723大学への名無しさん:04/10/14 18:35:22 ID:PKPzZ1Ym
>>717
(1)
(左辺)−(右辺)
=C(x+y)-2√(xy)
={(x+y)-2√(xy)}+(C-1)(x+y)
(x+y)-2√(xy)≧0(∵相加相乗)、(C-1)(x+y)≧0(∵C≧1、x+y≧0)より
(左辺)−(右辺)≧0
よってAが成り立つ。

(2)
x≧0、y≧0を満たす任意のx,yでAが成り立つ。
C(x+y)≧2√xy≧0より、両辺2乗すると、(左辺)^2−(右辺)^2≧0が成り立つ。
(左辺)^2−(右辺)^2を平方完成すると、Cについての条件式が出てくるので
それを解くとC≧1が示される。

(3)
√x+√y≦K√(x+y)・・・Bとする。
x≧0、y≧0を満たす任意のx,yでBが成り立つとよい。
まずK√(x+y)≧√x+√y≧0より明らかにK≧0・・・(*)
Bの両辺を2乗して(右辺)^2−(左辺)^2を計算すると、(K^2-1)(x+y)≧2√xy
ここで(1)(2)より、(K^2-1)(x+y)≧2√xyがx≧0、y≧0を満たす任意のx,yで成り立つには、
K^2-1≧1であることが必用十分条件なので、(*)とからK≧√2
よってKの最小値は√2
724大学への名無しさん:04/10/14 18:41:14 ID:F3if4JYp
>>723
おつかれさん

(2)はx=y=1を代入するほうが早いよ
725717:04/10/14 18:59:15 ID:c/UhEf9M
>>723
わかりました!!
ありがとうございます涙
726大学への名無しさん:04/10/14 20:45:42 ID:cfIEX+fg
(a^2/8)-3a+10 が -(1/8)*(a-12)^2 -8 に
する方法がわかりません。
(a^2/8)-3a+10 = a^2 -24a+80 = ここから・・・わからん。
727大学への名無しさん:04/10/14 20:56:45 ID:zDYv1sgO
>>726
ax^2+bx+c=a(x-b/2a)^2-(b^2-4ac)/4a
解の公式ってこうやって導くの知らん?
728大学への名無しさん:04/10/14 21:03:26 ID:S6X9ze7k
x、y、zに関する同次連立一次方程式
x+ycos(ν)+zcos(β)=0
xcos(ν)+y+zcos(a)=0
xcos(β)+ycos(a)+z=0
はa+β+ν=0のとき、自明でない解を持つことを示せ。

よくわからないので、宜しくお願いします。
729大学への名無しさん:04/10/14 21:25:39 ID:cfIEX+fg
>>727
a =a^2/8 、b=-3a 、c=10 を a(x-b/2a)^2-(b^2-4ac)/4a にあてはめると
(a^2/8)*(x-(12/a))-16になりました。

わかりません。すいません細かくお願いします。

730大学への名無しさん:04/10/14 21:28:52 ID:zDYv1sgO
>>728
自明な解ってのは(x,y,z)=(0,0,0)ってのはOK?
つまり(0,0,0)じゃない解を持つようにせよってこと。
角度はa,b,cにした。
zを消去して
x{1-cos^2(b)}=y{cos(a)cos(b)-cos(c)}
y{1-cos^2(a)}=x{cos(a)cos(b)-cos(c)}
x,yを消去して
{1-cos^2(a)}{1-cos^2(b)}={cos(a)cos(b)-cos(c)}^2
1-cos^2(a)-cos^2(b)=cos^2(c)-2cos(a)cos(b)cos(c)---(*)
これを満たすa,b,cの条件はa+b+c=360*nになるんじゃない?
ちなみに(*)は三次の行列式。知らなくてもいい。
731大学への名無しさん:04/10/14 21:34:06 ID:zDYv1sgO
>>731
なんで右辺のようになるのかは
「展開するとそうなるから」としか言えん。
ただ言っとくと俺も覚えてるわけではない。
頭ん中で考えて係数合うように変形すんの。
右辺の形にできたら=0の方程式を解いてみ。
a(x-b/2a)^2-(b^2-4ac)/4a=0
a(x-b/2a)^2=(b^2-4ac)/4a
(x-b/2a)^2=(b^2-4ac)/4a^2
x-b/2a=±√(b^2-4ac)/2a
x=b/2a±√(b^2-4ac)/2a
732大学への名無しさん:04/10/14 21:35:15 ID:zDYv1sgO
間違った。731って俺じゃん(笑)
731は>>729ね。
733大学への名無しさん:04/10/14 21:50:21 ID:S6X9ze7k
>>730
あぁ、そういうやり方もあるんですね。
ただ、描き忘れたんですが>728は行列(の階数)を用いて
解けという問題だったので、行列のほうの解法を教えてくれませんか?
質問が不完全で申し訳なかったです。
734大学への名無しさん:04/10/14 21:52:46 ID:QWzuSKcX
2次関数y=f(x)とy=cosxはx=0で共通の接線をもち、
かつ点(Π/2、0)で交わる。
このとき、2曲線y=f(x)とy=cosxの0<x<Π/2
における上下関係を調べよ。


という問題なんですけど、
なぜ「X=0で共通接線をもつ」と言う条件からf(x)=ax^2+1と
おけるんでしょうか?
735大学への名無しさん:04/10/14 21:54:59 ID:zDYv1sgO
>>733
はなから行列かよ(笑)
だったら行列式=0でいいじゃん。
上のとまったく同じ結果がでるよ。
上三角に変形してrank=2の条件を求めたって
行列式=0を導くだけじゃない?
736大学への名無しさん:04/10/14 21:57:43 ID:zDYv1sgO
>>734
cos(x)の接線はy=1
つまり(x,y)=(0,1)を通り傾き0
二次関数で接線の傾き0となるのは頂点
すなわち二次関数の頂点は(0,1)
(0,1)を頂点とする二次関数の一般形はそれ。
737大学への名無しさん:04/10/14 22:06:57 ID:S6X9ze7k
>>735
よくわかりました。
どうも有難うございました。
738大学への名無しさん:04/10/14 22:07:40 ID:mjS95YSU
ロル、ロピタル、チェバ・・・などの様々な定理は会得したほうがいいでしょうか?
739大学への名無しさん:04/10/14 22:12:06 ID:zDYv1sgO
>>738
チェバは中学で会得してるはず。
ロルは平均値の定理と同じっしょ?
ロピタルは大学入ってからで十分。
受験で使うと激しく粗捜しされる。
740大学への名無しさん:04/10/14 22:22:44 ID:QWzuSKcX
>>736
ありがとございます
741大学への名無しさん:04/10/14 22:35:15 ID:cfIEX+fg
>>731
ありがとうございます。
742大学への名無しさん:04/10/14 23:48:12 ID:oKtFH1Di
なぜ、関数を積分すると
x軸とその関数で囲まれる部分の面積になるんですか??
743大学への名無しさん:04/10/14 23:50:06 ID:zDYv1sgO
>>742
数Vの教科書に証明が載ってるでしょ?
744大学への名無しさん:04/10/14 23:52:40 ID:tu+M8PIN
>>742
逆だよ、逆。
積分によって面積を定義するのが本当。
高校の教科書は誤魔化し。
面積という概念が既に存在しているとして話を進めている。
745大学への名無しさん:04/10/14 23:57:44 ID:zDYv1sgO
>>744
面積で積分定義しますか?
不定積分の定義は微分の逆演算のはず。
746大学への名無しさん:04/10/14 23:59:07 ID:zDYv1sgO
>>744
ああゴメン。読み間違えた。
でも面積って積分で定義するんだ、へ〜。
747大学への名無しさん:04/10/14 23:59:26 ID:oKtFH1Di
>>744
つまり、本当は
積分したらたまたまそれが面積だったってことですか??
748大学への名無しさん:04/10/15 00:01:59 ID:yk2XZPmn
微分の逆演算で不定積分が定義されるとは限らない。
解析概論でも読め。

749大学への名無しさん:04/10/15 00:05:00 ID:yk2XZPmn
よーく考えてみなよ。
放物線と直線で囲まれた部分の面積ってなんだ?
広さを厳密に定義しようと思ったら、小さい長方形とかで埋めつくしていって極限とるしかないだろ?
750大学への名無しさん:04/10/15 00:09:15 ID:XyDIWh6Q
■初項A 公差d の等差数列{An}の和Snは、n=8のとき最大値136となる。
Aとdを求めなさい。

という問題なんですが、等差数列の和の公式から
2A+7d=34 と出ました。A9以降が負になることは分かるんですがここから
進みません。誰か教えてくれませんか?

751747:04/10/15 00:15:07 ID:F5hS2JNv
>>749
教科書にある区分求積法っていうやつですか??


lim(n→∞)(k=0)(n-1)f(a+(b-a)k/n)(b-a)/n=∫a→b f(x)dx
という公式が書いてあるけど、説明読んでなんとなくわかった気がします。
752大学への名無しさん:04/10/15 00:17:43 ID:yi2udUU0
>>750
A,dは整数とかない?
753大学への名無しさん:04/10/15 00:19:43 ID:hiJoCzmn
>>751
高校レベルだとそのイメージで良い。
754大学への名無しさん:04/10/15 00:20:44 ID:agEVI9GA
>>750
n=8で最大となることをどのように解釈する?
755750:04/10/15 00:25:03 ID:XyDIWh6Q
>>752
ないです。
>>754
A9以降は負になる・・・ことしか思いつきません。
756大学への名無しさん:04/10/15 00:28:44 ID:hiJoCzmn
大学レベルでは積分の定義はRiemann和(区分求積法のようなものとでも思ってくれ)によって定義される。
そして面積は重積分というもので定義される。

が、高校レベルのイメージとしては、区分求積法ぐらいで良いだろう。

多少のごまかしはしかたない。
これは理解をするためのものだ。
中学生にH+は水の中に存在すると言われているが、実際は存在しないといったり、
高校生に、原子の周りをある一定の軌道(orbit)を描いて飛んでるイメージはうそだといったりしては
ただ混乱を招くのと同じ。
757大学への名無しさん:04/10/15 00:31:28 ID:yi2udUU0
>>755
A8>0,A9<0を不等式で表して何とかならない?
758747:04/10/15 00:35:06 ID:F5hS2JNv
>>756
レスありがとうございます。

とりあえず区分求積法で
なるほどと思えたので良かったです。

それにしても微分と積分を考えた昔の人って
どんな頭してたんでしょうか。
「なぜこんなことを思いつくんだ?」ってかんじです。
759大学への名無しさん:04/10/15 00:41:29 ID:hiJoCzmn
>>758
もともとは微分と積分は別々のものとして発達していた。
微分は接線の傾きを求めるためのもので、
オイラーががんばっていたように思う。
レムニスケート曲線やアルキメデスの螺旋に接線を引いていたように思う。
ラテン語で説明がなされていた。

積分は。。。あまり記憶が無いが、面積を求めるためのものだったように思う。

そして、ニュートンやライプニッツが、ニュートンは物理に生かすために、
ほぼ同時期に、違う場所で、
微分と積分の形を整えたのだったと思う。

そしてライプニッツ型の変数分離の微積と
高校などでやる、ニュートン型の微積が生まれた。

ちなみに世界で最も早く微積を整えたのは日本人で、
ほぼ一人で作り上げた天才が(驚くべきことに)ほぼ同時期にいたわけだが、
これはヨーロッパで発表をしていないので、
現在残っているのは、上のふたつの流派を組んだ微積分だけである。

というのを大学の教養学年のときにやった。
760750:04/10/15 00:45:52 ID:XyDIWh6Q
>>757
その不等式と2A+7d=34から、適当に当てはめてA=31 d=-4が出たんですが
なんか上手い解法はないのかなぁと思いました。
761大学への名無しさん:04/10/15 00:49:57 ID:yi2udUU0
>>760
それしかないんじゃない?
それ以外情報ないし。
762大学への名無しさん:04/10/15 00:50:49 ID:kk9PUHXt
>>760
なぜそれが成り立つ?A,dは整数とは限らないんでしょ?
763大学への名無しさん:04/10/15 00:54:19 ID:agEVI9GA
>>760
整数条件を探す方がよいのかも。
設問の最初の方に書いてない?
764大学への名無しさん:04/10/15 01:09:11 ID:FQyf3gtA
高校一年生を教えているんですが、三角比・三角関数で、
大学受験の範囲において、単位円を使うメリットは何なのか教えてもらえませんか。

>>475>>478にあるようなsin、cos、tanを求めるために使ってみる、
というのぐらいしか知らないんですが。あとは還元公式のために使うぐらい?
私はsin、cos、tanを覚えるときに目安程度に使っていたぐらいで、
加法定理さえ覚えていればいいや、って感じで
単位円にそれほどのメリットを感じずに過ごしたもので。
765大学への名無しさん:04/10/15 01:14:34 ID:hiJoCzmn
>>764
メリットを感じなかったのなら、
君にとって、使う必要性はなかったのだろう。

具体的に言うことは難しいが、
おれは結構多用する。
766大学への名無しさん:04/10/15 01:16:35 ID:lm5k5b1n
-(b-c)(a-b)(a-c)
=(a-b)(b-c)(c-a)
なんでですか?
やっぱいちいち展開するんですか?
青チャートIAの例題12より
767大学への名無しさん:04/10/15 01:18:24 ID:EttPdXOO
3 sinθ + 4 cosθ ( 30°≦ θ ≦ 240°)
みたいな式の最大最小を求めるときに、

a↑ = ( 3 , 4 )
b↑ = ( sinθ , cosθ )

とすれば、xy座標上で視覚的に処理出来たりしません?


もっと単純に、
sin(π/2±θ) = cosθ
みたいなものも、視覚的に処理が出来る。

加法定理だって、何年か前に東大で出題されたように、
単位円を使って証明するわけだし。
768大学への名無しさん:04/10/15 01:22:51 ID:EttPdXOO
> b↑ = ( sinθ , cosθ )

すまん。θを書きたくないのでコピーしたら
sinとcosの順番間違えた。


769大学への名無しさん:04/10/15 01:22:59 ID:hiJoCzmn
>>766
-(b-c)(a-b)(a-c)
=(b-c)(a-b)(-a+c)
=(a-b)(b-c)(c-a)
なにかぼけてるのか?
770大学への名無しさん:04/10/15 01:28:06 ID:lm5k5b1n
766です
>>769さん優しく答えてくれてありがとうございます
なんかわかんないけど俺って
ほんっとひょんなとこでつっかえるんです。
計算力が低いのかと思っていまでも毎日一時間中学の数学やるくらいです
771大学への名無しさん:04/10/15 01:31:31 ID:hiJoCzmn
>>770
中学の勉強しても計算力はそんなにつかんよ。

問題解くときに集中して、間違えないように解く事にいみがあるんだ。
772大学への名無しさん:04/10/15 01:41:01 ID:lm5k5b1n
たしかに、ちょっとヌけてるかも
ちゃんと集中してやってみます。アドバイスどうもでした。
773大学への名無しさん:04/10/15 10:11:32 ID:Vz7URcqb
0<1−x^2≦2x

ってどういうふうになおせばいいんですか?教えてください
774エッセンス布教者:04/10/15 10:27:43 ID:1Qf3vckS
>>773
0<1−x^2≦2x は1-x^2が0よりは大きいけど2x以下だということだ。
つまり・・・
775大学への名無しさん:04/10/15 10:42:54 ID:acTVD7fJ
>>767
意味不明
776大学への名無しさん:04/10/15 11:13:12 ID:mxF9KLLG
>>775
>>764の言う単位円を使う、という意味からは少しずれているかもしれないが、意味は分かるだろ。
777大学への名無しさん:04/10/15 11:34:31 ID:GAuvJuVT
加法定理の語呂合わせ教えてください
778高1:04/10/15 11:39:58 ID:Vo+Htkbm
順列のとこで質問ですが記述模試の時
例えば円順列の場合いきなり円順列なので
(6−1)!=5
と書いていったら
×にされますか?
参考書には
6人が円卓に着席する
方法は、
異なる6個の円順列
であるから
と詳しく書いてありますが
ちょっと省略したら
駄目かなと思いまして。
779高1:04/10/15 11:46:45 ID:Vo+Htkbm
あと10人の生徒のなかから、
兼任を認めないで、
部長、副学長、会長を
一人ずつ選出する方法は全部で何通りあるか。
という問題ですが
これは模試の時
求める場合の数は
10個の異なるものの
なかから3個とった
順列に等しいので
10P3=10×9×8=720通りと答案に書いたら駄目でしょうか?
参考書にはかなり詳しく書いてありますが
模試の時そんなに
詳しく書いてたら
時間足りないような感じがするので
上のようにシンプルな感じで
書いたら駄目ですか?
アドバイスよろしく
お願いします
780大学への名無しさん:04/10/15 12:07:38 ID:acTVD7fJ
まぁこれくらいの問題なら問題ないんじゃないかな
781大学への名無しさん:04/10/15 13:34:48 ID:Ls/X1HX+
参考書は何もわかってない人が1から見てもわかるように、
少し冗長になってることがあるからね。
782大学への名無しさん:04/10/15 16:05:46 ID:2LrqUGJa
>>778-779
確率や場合の数は答えだけ書いてもあってれば満点もらえるよ。
けど計算間違いしてたら部分点すらもらえないから、計算式と、それでもまだ不安なら計算式の説明を書き添えればいい。
書きすぎで減点されることはないから。
783大学への名無しさん:04/10/15 16:37:09 ID:4HdTTWrq
数列の特性方程式が分かないのですが、どうしてa(n)と、a(n+1)を同じxと置いて良いのでしょうか?
784大学への名無しさん:04/10/15 17:18:17 ID:w/kbb/jY
a_(n+1)=pa_n+q、p≠1の時
a_(n+1)-α=β(a_n-α)と変形できるとすると
a_(n+1)=βa_n+α(1-β)となり
係数を比較して
α(1-β)=q
β=p
∴α=q/(1-p)
これは最初の漸化式でa_(n+1)=a_n=xとおいた1次方程式の解と同じ
785大学への名無しさん:04/10/15 17:29:12 ID:4HdTTWrq
>>784 分かりました!どうもありがとうございましたm(__)m
786大学への名無しさん:04/10/15 18:10:36 ID:wTc6sQn4
a_(n+1)=pa_n+qに対し、
α=pα+qを満たすαを考えれば、
上の式引く下の式をすることで
{a_(n+1)-α}=p{a_n -α}となり、
等比数列に
帰着させられる。

ということに過ぎない。
この特性方程式の考え方は複素数の1次変換等の問題でも使えたりして、
意外と数学全般で使ってたりする考え方なのだ。
787大学への名無しさん:04/10/15 18:20:22 ID:E7vjmgxm
得意な人やって

1個の玉を打つとアタリかハズレに同様に確からしく入り
はずれると0個、当ると2個玉が出てくるパチンコマシンがある。
今1個の玉を持った人が破産するまでに玉を打つ回数の期待値を求めよ。
788大学への名無しさん:04/10/15 19:14:07 ID:3YbK3I1s
合成って何で合成できるんですか?
問題は解けるんだけどどういう理由
で合成できるかが分からない。誰か
おせーて。
789大学への名無しさん:04/10/15 19:16:56 ID:wTc6sQn4
なんの合成だよ。
790大学への名無しさん:04/10/15 19:19:46 ID:/Gsy4r7Z
相加平均≧相乗平均 という関係において、

(A+B+C+D)/4 ≧ (ABCD)^(1/4)

↑が成り立つと思うんですが、
この場合、等号はどの場合に成り立ちますか?
A=B=C=D の時のみですか?
791大学への名無しさん:04/10/15 19:24:15 ID:wTc6sQn4
うん
だいにゅうしてみなよ。
792大学への名無しさん:04/10/15 19:33:31 ID:AMF88vMd
>>790
A=B=1, C=D=-1を入れてみたら成立しなかったよ。


ついでだ、暇だから相加平均・相乗平均の不等式の証明を書いてやるよ、
a(1),a(2),…a(n)>0として、一般に
( Σ[k=1,n] a(k) )/n ≧ ( Π[k=1,n] a(k) )^(1/n)
が成立する。

[ 証明 ]
両辺を( Π[k=1,n] a(k) )^(1/n)で割り、
b(i) = a(i)/ ( Π[k=1,n] a(k) )^(1/n)
と置き直せば、 Π[k=1,n] b(k) =1 の条件の下 Σ[k=1,n] b(k)≧nを示す問題に置き換えることができる。
n=2の時、不等式の成立は明らかなので、帰納法によりn≧3の時、成立することを示す。

n=k(≧2)の時、成立したとして、帰納法を用いる。
明らかにb(i)≦1≦b(j)となるようなi,jが存在する。bを並べ替え、b(1)≦1≦b(2)としても問題ない。
( b(1)-1 )(b(2)-1)≦0が成立し、これを展開すれば
b(1)b(2) + 1 ≦ b(1)+b(2) が成立する。
以上より、

Σ[i=1, k+1] b(i)
= b(1)+b(2) + Σ[i=3, k+1] b(i)
≧ 1+ b(1)b(2) + Σ[i=3, k+1] b(i)
が成立する。 明らかにb(1)b(2) + Σ[i=3, k+1] b(i)に対して帰納法の仮定を用いればb(1)b(2) + Σ[i=3, k+1] b(i)≧k
が成立する。

ここまで書けば、わかるっしょ。
んで、>>790はA,B,C,Dが正っていう条件が抜けてる。
793大学への名無しさん:04/10/15 19:45:13 ID:wTc6sQn4
x_i≧0 (i∈N)
Σ[i=1 to n]x_i/n=A
(x_1*x_2*……*x_n)^(1/n)=Gとおく。
x>-1に対してe^x≧1+x≧0が成り立つので、
Σ[i=1 to n](x_i/A-1)=0
1=e^0=e^{Σ[i=1 to n](x_i/A-1)=0}=e^(x_1/A-1)*e^(x_2/A-1)*……*e^(x_n/A-1)
≧(x_1 /A)*(x_2 /A)*……*(x_n /A)=G^n/A^n
よって、
A≧G
794大学への名無しさん:04/10/15 19:52:21 ID:bEHsAjyX
a≠0のときf(x)=sin(ax)の周期が2π/|a|であることを証明せよ。

周期の定義はすべてのxについてf(x+p)=f(x)をみたすpのうち正の最小値である。
f(x+2π/|a|)=sin(ax+(a/|a|)2π)=sin(ax±2π)=sin(ax)=f(x)なので
p=2π/|a|のときすべてのxについてf(x+p)=f(x)をみたすのは分かりますが
p=2π/|a|が正の最小値であることの説明が分かりません。
795大学への名無しさん:04/10/15 20:09:22 ID:AMF88vMd
>>794
sin(ax)=0となるxの値は
x=0、±kπ/|a|  kは自然数 つーか、kπ/aでいいんだけどな k∈Zとして……

ま、んなことはいいとして、

上のことより、明らかsin(ax)の周期はπ/|a|の倍数となり、今>>794の定理により
2π/|a| は周期の条件を最小性以外満たしている。2π/|a|が周期であるためには
π/|a|が周期でないことを示せばよい。
明らかに、
sin( π/2|a| ) ≠ sin( π/2|a|+π/|a| )
が成立するため。 あ〜〜めんどせ
796大学への名無しさん:04/10/15 22:33:56 ID:yk2XZPmn
>>794
sin(a(x+p))=sin(ax) ⇔ 2cos(ax+ap/2)sin(ap/2)=0
これが任意のxについて成り立つには?
797大学への名無しさん:04/10/15 22:47:08 ID:yIOauUIk
a,bを異なる実数とする時、xに関する方程式
(xー2a)(x-2b)-(2x-a-3b)=0は異なる2つの実数解を持つ事を
証明せよ
展開して判別式使う所まではできたのですがそのあとが分かりません
教えて下さい。
798大学への名無しさん:04/10/15 22:47:45 ID:bEHsAjyX
>>796
sin(ap/2)=0、nを整数として
ap/2=nπつまりp=2nπ/aが成り立つことです。
・・・!
p=2nπ/aのうち正の最小値となるのは
a>0のときはp=2π/a、a<0のときはp=-2π/a
合わせるとp=2π/|a|!!

ようやく分かりました。ありがとうございました。
>>795のは正直よう分からんですけど、レス感謝です。
799大学への名無しさん:04/10/15 22:56:18 ID:wTc6sQn4
(xー2a)(x-2b)-(2x-a-3b)=0
これは
y=(xー2a)(x-2b)
y=(2x-a-3b)=2{x-(a+3b/2)}
が異なる2点で交わることに同値。


図を描けば終わり。
800大学への名無しさん:04/10/15 22:56:25 ID:AMF88vMd
>>798
何がわからんのだ?
801大学への名無しさん:04/10/15 22:56:56 ID:7vHwr4Aw
>>797
判別式D
=a^2+b^2-2ab+a-b+1
=(a-b)^2+(a-b)+1
=(a-b+1/2)^2+3/4
>0
よって異なる2実数解をもつ。
802大学への名無しさん:04/10/15 23:02:32 ID:XEANwVz0
>>797>>798>>794はOKだけど、
>>794の性質はたいてい和積公式の前に習うよな。
>>794を習ったときどういう説明をされたんだろう。思い出せん。
>>795でもなかったし。まあどうでもいいか。

803大学への名無しさん:04/10/15 23:07:22 ID:yk2XZPmn
>>797
>>799と本質的には同じだが...

f(x)=(x-2a)(x-2b)-(2x-a-3b) はx^2の係数が正で
f((a+3b)/2)<0 より題意は成り立つ.

この手の問題は展開しないで解くのが近道.
因みに(a+3b)/2 は 2a と 2b の内分点.
804大学への名無しさん:04/10/15 23:15:21 ID:yIOauUIk
>>799
ありがとうございます。判別式使うことばかり考えてました。
そういう解き方もあるんですね
>>801
ありがとうございます
>>803
すいませんが内分点ってなんですか?
805大学への名無しさん:04/10/15 23:40:26 ID:E8KYKLVk
>>804
たとえばOAを1:2で内分する点P、と言えば
OP:PA=1:2 になるような点のこと
806大学への名無しさん:04/10/16 09:26:35 ID:ERcJpzOj
>>804
たとえばOAを1:2に内分する点P、と言えば
OP:PA=1:2 になるような線分OA上の点Pのこと
807大学への名無しさん:04/10/16 10:11:37 ID:kK9BZbjQ
>>789
sinX+cosX=√2sin(X+α)とか。
どうか親切な方教えてください。
808大学への名無しさん:04/10/16 10:16:22 ID:DGuYh5I4
>>807
グラフかけ
809大学への名無しさん:04/10/16 11:04:54 ID:70LAZyvn
数列の和から一般項を求める問題で、
n=1をS(n)に代入して求めたa(1)と、
求めた一般項a(n)にn=1を代入して求めたa(1)の値
が同じになったり違う値になったりするのはどうしてなのでしょうか?
810大学への名無しさん:04/10/16 11:13:24 ID:N8fBJdZ7
>>809
多分、S(n) (n≧1)が与えられていて、a(n)=S(n)-S(n-1)で求めるような場合を言ってるんだと思うけど、これから求めたa(1)は、
a(1)=S(1)-S(0)
だ。S(n)はn=0のときは定義できないから、この式には本来意味が無いんだけど、たまたまS(0)=0であれば、
a(1)=S(1)
となる。
811大学への名無しさん:04/10/16 12:20:00 ID:Zrpz6LZj
S(0)=0だからS(1)−S(0)=a(1)。
812大学への名無しさん:04/10/16 13:20:30 ID:tQ/Q3KO7
>>807
sinx+cosx
=√2{(1/√2)sinx+(1/√2)cosx}
=√2{sinxcosα+cosxsinα}(←この場合αはcosα=sinα=1/√2をみたす角、すなわち45°)
=√2sin(x+α)
というふうに、加法定理から導く。
813大学への名無しさん:04/10/16 13:20:55 ID:qS1UlU98
>>810>>811
レスどうもです。

S(0)が0じゃない場合があるなんて、考えませんでした。a(1)の値が同じになったり、違う値になったりするのは理解出来ました。
因みにS(0)が0じゃないってどういう状況なのでしょうか?数列の和として定義したSがS(0)の時値を持つということに対して、モヤモヤ感が残るので。
814大学への名無しさん:04/10/16 13:29:24 ID:ICiYw0en
>>813
たとえば、a(1)=2, a(n)=1 (n≧2)とか。
このとき、S(n)=n+1。
815大学への名無しさん:04/10/16 14:03:43 ID:ERcJpzOj
>>813
S(n)を数列の和として定義したのであれば、S(0)は0項の和なので定義されていない。値は持たない。
数列の和を S(n) (n≧1) で定義したのであれば、S(0)の値は数列に対して何の影響も与えないのでどんな値であってもかまわない。
816大学への名無しさん:04/10/16 14:12:07 ID:eWto2SIT
半径5cmの円の中心をOとします。中心Oから3cmはなれた点をPとします。
Pを通り直交する円の2つの弦 ABとCD を引きます。
このとき 四角形ACBDの面積の最大値と最小値を求めてください。


PはOからの距離が3のところにあるのだから半径3の円上にあるとしてよく、また対称性より第1象限にあるとしてよいのでP(3cosθ,3sinθ).(0≦θ≦π/2)
さてACBDの順に並ぶとしてAB⊥CDかつAPB,DPCは一直線上にある。
よってABがx軸に平行、CDがy軸に平行であるとしてよく
A,C,B,D順に(-√(25-9sin^2θ),3sinθ),(3cosθ,-√(25-9cos^2θ)),(√(25-9sin^2θ),3sinθ),(3cosθ,√(25-9cos^2θ))
□ACBD=AB*CD=2√(1600+81sin^2(2θ)))
だから最大値は82最小値は80
どこかおかしいでしょうか?答えがないのでお願いします
817大学への名無しさん:04/10/16 14:18:54 ID:eWto2SIT
あっ!!痛恨のミス2で割るの忘れてました。最大値は41最小値は40です
818大学への名無しさん:04/10/16 14:54:59 ID:Yo+PLHel
相加相乗平均ってどういう時につかうのでしょうか?
819大学への名無しさん:04/10/16 15:11:33 ID:KM9exDX5
使いたいときに。
820大学への名無しさん:04/10/16 16:13:08 ID:gSOyrF5k
>>818
ちょっとした空き時間とか。
821大学への名無しさん:04/10/16 16:25:15 ID:GFUIg9AC
>>818
a>0のとき、8a+(2/a)+1 の最小値を求めよ。

相加相乗で即答できる一例
822大学への名無しさん:04/10/16 16:41:51 ID:Yo+PLHel
>>821
どういう風に見極めるんですか?
まだよくわかりません。。。
823大学への名無しさん:04/10/16 16:48:01 ID:GFUIg9AC
>>822
とりあえず正で(和)≧(積)の証明ならつかえるかもね、くらいでいいのでは
あとは a+(1/a) みたいに積が定数になりそうな時とか
824大学への名無しさん:04/10/16 17:55:18 ID:DJQmrXHq
>>814>>815
レスさんくす。
これでスッキリしました。どうもでした。
825807:04/10/16 18:01:30 ID:RTKvbTRk
>>812
レスありがとうございます!
とても分かりやすかったです。
826大学への名無しさん:04/10/16 18:57:19 ID:si0O+1iB
nは自然数、1≦x≦eにおいて
(logx)^(n+1)≧0になるのは何故ですか?
827大学への名無しさん:04/10/16 19:06:38 ID:fe80cO1d
>>解答する気のある人たちすべて
>>826がもし釣りでないなら、答えてやらないほうが>>826のためだとおもう。
828大学への名無しさん:04/10/16 19:08:30 ID:si0O+1iB
いやマジで教えてください。
829大学への名無しさん:04/10/16 19:11:20 ID:daGKatky
回答するも何も...
830大学への名無しさん:04/10/16 19:12:25 ID:fe80cO1d
>>828
何が分からんの?
あるいは何が分からんから分からんと思う?
831大学への名無しさん:04/10/16 19:14:52 ID:si0O+1iB
ほんとに教えてください。
832大学への名無しさん:04/10/16 19:17:56 ID:KM9exDX5
>>826
何が分からないのか分からない。
833大学への名無しさん:04/10/16 19:18:13 ID:N50ETItI
何がわからないか書いてくれ
834大学への名無しさん:04/10/16 19:18:22 ID:LY3jide8
頂点(1,3)で(-1,1)を通る放物線の方程式を求めよ。

って問題なんですけど、
y=(x-1)+3
にするまではわかるんですが、
この後がわかりません。
どなたか簡単に教えていただけませんか?
宜しく御願いします。
うんこな質問ですみません。
835エッセンス布教者:04/10/16 19:18:30 ID:XyUVt+y/
まぁy=logxのグラフでも書いてみれや。
基本的な関数のグラフを覚えるのは必須だよ。
836大学への名無しさん:04/10/16 19:19:39 ID:si0O+1iB
logx≦0になりませんか?
837エッセンス布教者:04/10/16 19:21:43 ID:XyUVt+y/
>>834
頂点(1,3)な"2次曲線"は
y=a(x-1)^2+3 (a≠0)
であらわせる。
未知数一個だから一つ式が作れればとける。
あとはわかるでしょう。
838エッセンス布教者:04/10/16 19:22:06 ID:XyUVt+y/
>>836
だからグラフかけって。
839大学への名無しさん:04/10/16 19:24:49 ID:fe80cO1d
>>836
log_{e}aってどういう数だと思ってるの?
840大学への名無しさん:04/10/16 19:25:13 ID:KM9exDX5
y=(x-1)^2+3
じゃないだろ。
y=ax^2+bx+cなんだから、3つの条件が無いと一つに定まらない。
すなわち、上の式は2つの式で1つの変数が残ってないといけないのに、
もうすでに変数が残ってないから間違い。

頂点が(1, 3)である条件は、
(y-3)=a(x-1)^2
これは原点が頂点である放物線y=ax^2を動かしたものだ。
そしてこれが(-1, 1)を通るのだから、
(1-3)=a(-1-1)^2が成立するaの値を求めれば良い。

つまり、a=-2/4=-1/2
841大学への名無しさん:04/10/16 19:26:35 ID:KM9exDX5
>すなわち、上の式は2つの式で1つの変数が残ってないといけないのに、
>もうすでに変数が残ってないから間違い。

訂正。頂点が(1, 3)という条件で2つの条件を使ってまだ一つ残っているはずなのに、
もうすでに変数が残ってないから間違い。
842大学への名無しさん:04/10/16 19:27:48 ID:KM9exDX5
>>836
底がeのlogの関数y=log(x)をグラフに描いてみてくれ。
843大学への名無しさん:04/10/16 19:33:13 ID:si0O+1iB
あー分かりました!!
すいませんあほすぎました。
すいませんすいません。
これでも偏差値60はあるんです。ごめなさい
844大学への名無しさん:04/10/16 19:33:45 ID:KM9exDX5
>>843
がんばれ。超がんばれ。
845エッセンス布教者:04/10/16 19:34:42 ID:XyUVt+y/
>>843
まぁたまにはそんなことも・・・ねえよw
846大学への名無しさん:04/10/16 19:38:21 ID:si0O+1iB
まじすいません。
847大学への名無しさん:04/10/16 19:42:21 ID:lj7exLkD
OA↑=a↑,OB↑=b↑,|a↑|=|b↑|=1,a↑・b↑=k
のとき、なぜOH↑=(cosθ)a↑=ka↑になるんですか?




848エッセンス布教者:04/10/16 19:47:26 ID:XyUVt+y/
>>847
Hって何よ。まぁなんとなくわかるけど・・・
俺はめんどくさいから説明しないけど、正射影ベクトルでググってみてはどうか。
849大学への名無しさん:04/10/16 19:52:06 ID:lj7exLkD
BからOAへの垂線をBHとし∠AOB=θ
としたときです…
850大学への名無しさん:04/10/16 19:56:06 ID:KM9exDX5
a, bをベクトルとする。ベクトルOHをhとする。
内積<a, b>=||a||*||b||*k=k
よって、||h||=k(三角形を描いて確かめよ)
よって、||h||*a/||a||=ka
851大学への名無しさん:04/10/16 20:02:04 ID:si0O+1iB
ってことでもう一問出させてください。

f(x)=|x^2−2x|に対して、
数列{a(n)}が漸化式a(n+1),1<a(1)<2を満たす時
グラフを利用して極限値lim a(n)(n→∞)を求めよ。

スゴイ特殊な解き方してあるんで良く分かりません。
852大学への名無しさん:04/10/16 20:03:39 ID:lj7exLkD
あ〜分かりました!!ありがとうございます!
あと△ABCの重心をG、外接円の中心をEとする時
EA↑EB↑EC↑=3EG↑
になるのはどうしてですか?
853大学への名無しさん:04/10/16 20:04:57 ID:KM9exDX5
漸化式はa_(n+1)=f(a_n)でいいかな?
854大学への名無しさん:04/10/16 20:05:48 ID:KM9exDX5
>>852
>EA↑EB↑EC↑

なにこれ。ちゃんと書こうね。
855エッセンス布教者:04/10/16 20:06:13 ID:XyUVt+y/
>>851
すごくよくある解法ですよ。
自分でもうすこし考えてみる価値がある解法だと思うよ。
てか文字で説明しにくいしね。
856大学への名無しさん:04/10/16 20:08:23 ID:si0O+1iB

f(x)=|x^2−2x|に対して、
数列{a(n)}が漸化式f(a(n))=a(n+1),1<a(1)<2を満たす時
グラフを利用して極限値lim a(n)(n→∞)を求めよ。です。
857大学への名無しさん:04/10/16 20:09:05 ID:lj7exLkD
EA↑+EB↑+EC↑=3EG↑です!
ごめんなさい
858大学への名無しさん:04/10/16 20:23:45 ID:KM9exDX5
>>851
0<a_n<1 (n>1)を示す。
0<a_2=3/4<1
a_kで成り立つとすると、
0<a_(k+1)=-a_k^2+2a_k<1

さらに、a_(k+1)-a_k=-a_k ^2 +a_k=-(1/2 -a_k)^2+1/4>0
よって、a_(k+1)>a_k

ここで、(極限は1だと分かるので)
1-a_(n+1)=1+a_n ^2-2a_n=(a_2-1)^2>(1-a_2)(1-a_k )
∴{1-a_(n+1)}=(1-a_2)(1-a_k )=(1-a_2)^(n-1)→0
よって、a_n→1
859大学への名無しさん:04/10/16 20:24:45 ID:KM9exDX5
>>857
成分表示して確かめろ。
860大学への名無しさん:04/10/16 20:30:54 ID:KM9exDX5
>>858
訂正
0<1-a_(n+1)=1+a_n ^2-2a_n=(a_n-1)^2<(1-a_2)(1-a_n ) (なぜならばa_n>a_2∴0<1-a_n<(1-a_2) )
∴0<{1-a_(n+1)}<(1-a_2)(1-a_k )<(1-a_2)^(n-1)→0

挟み撃ちにより、a_n→1
861大学への名無しさん:04/10/16 20:43:59 ID:KM9exDX5
>>857
ベクトルe=(e_1, e_2)
ベクトルa=(a_1, a_2)
ベクトルb=(b_1, b_2)
ベクトルc=(c_1, c_2)
とおく。
e-a+e-b+e-c=3e-(a+b+c)=3e-3g=3EG↑
862大学への名無しさん:04/10/16 20:50:37 ID:LY3jide8
2回も申し訳ないです。

軸の方程式がx=3で、かつ2点、(2,-2),(5,4)を通る放物線の方程式は?

これで最後にします。
どうか教えていただけないでしょうか。
863エッセンス布教者:04/10/16 20:55:13 ID:XyUVt+y/
>>862
頂点の座標を(3、b)とおけるからあとは上にあったのとおなじ。
未知数が二つだから二つ式を立てればいい。
864大学への名無しさん:04/10/16 21:00:27 ID:KM9exDX5
>>862
y=a(x-b)+c
軸がx=3よって、
b=3
さらに2点を通るので、
-2=a+c
4=4a+c
865大学への名無しさん:04/10/16 23:24:58 ID:Disdz6yi
マセマの実践ゼミp118の演習問題39の(2)です。
∫[0,1]x*e^-x^2dx
なんですが、僕は今までだと、1/2*xを微分していましたが、この参考書ではe^-x^2を微分して、やっているみたいです。
しかし、なぜそこから導けるかが分かりません。
どなたか教えて下さい。
非常に簡単な問題なのに申し訳ありません。
お願いします。
866大学への名無しさん:04/10/16 23:33:02 ID:deCyO3O/
>>865
x*e^(-x^2)の原始関数は(-1/2)e^(-x^2)じゃない?
f(g(x))の微分はg'(x)f'(g(x))でしょ?
f(t)=(-1/2)e^t,g(x)=-x^2とおけば
f(g(x))=(-1/2)e^(-x^2)で微分は(-2x)*(-1/2)e^(-x^2)=x*e^(-x^2)じゃん。
867大学への名無しさん:04/10/17 02:40:43 ID:68HGkKg0
>>809-815
階差数列の時の場合はどうなの?
868大学への名無しさん:04/10/17 02:42:31 ID:dbhszTcB
>>815
ていうか数学偏差値73だがS(1)とa_1とが一致しないということが分からない。

誰か例題でも持ってきてくれないか?
869大学への名無しさん:04/10/17 02:43:53 ID:bLoZPwrt
>>868
a(0)があったら一致しないんじゃない?
870大学への名無しさん:04/10/17 02:51:35 ID:07ZbbWZR
>>868 心配しなくてもS(n)=Σ[k=1,n] a(k)
と定義している限りは、間違いなくS(1)=a(1)だと思われ。
他の定義をしないと駄目だろ
871大学への名無しさん:04/10/17 02:53:47 ID:dbhszTcB
>>870
だよね。オレにはあの議論が理解できない。
というか、一致しない事例にぶつかったことも無いし、
それで問題もいくつも解いてきた。
872大学への名無しさん:04/10/17 02:57:19 ID:07ZbbWZR
ただ、問題となるのはS(n)=Σ[k=1,n] a(k)が成立するかじゃないの?
あらかじめ、問題文にS,aが与えられている場合は、成立するとは限らないわけで。
873大学への名無しさん:04/10/17 03:01:25 ID:dbhszTcB
>>872
どんな問題?
874大学への名無しさん:04/10/17 03:03:29 ID:07ZbbWZR
問題もクソも、 普通に
S(n)=n^2 a(n)=3n とかだったら……っていう意味なんだけど、わかりにくくてゴメソ
875大学への名無しさん:04/10/17 03:04:40 ID:dbhszTcB
だってa_nとS_nの関係の定義って、
S_n=a_1+a_2+……+a_(n)
a_n=S_n-S_(n-1) (n≧2) or S_1 (n=1)
だぜ。
a_n=S_1 (n=1)
という定義がある以上、一致しないのは、
a_nとS_nを構成する数列が違う数列だからと言うほか無い。
876大学への名無しさん:04/10/17 03:05:39 ID:07ZbbWZR
追記、 常識で考えればSは和Sumを現すわけだけど、
問題を作る人によって、んなものはいくらでも変わるって言う意味。

でなければ、S(1)=a(1)が必ず成立しちゃうよぉ
使っちゃいかんっていうのはa(n)=S(n)-S(n-1)だろ、上を見ると。
っていうことは、この式S:N→Rなんだから、n-1≧1なわけで、n=1の時に使ったら
そら、いかんぜよ。
877大学への名無しさん:04/10/17 03:05:53 ID:dbhszTcB
S(n)=n^2 a(n)=3n 
だってこれはS_nを構成する数列じゃないじゃん。
878大学への名無しさん:04/10/17 03:07:37 ID:07ZbbWZR
ま、ID:dbhszTcBタンの考えは全く問題ないと思われ。
879大学への名無しさん:04/10/17 03:08:50 ID:07ZbbWZR
そもそも、上で議論しているのは
a(n)=S(n)-S(n-1) が恒に成立するかじゃないの?
だったら、自分で書いてるとおりだろ。
880大学への名無しさん:04/10/17 03:09:20 ID:8x+Pr+je
何もめてんだかわからんがS1=a1は常に成り立つにきまってんだろクソハゲども
881大学への名無しさん:04/10/17 03:10:25 ID:dbhszTcB
>>878
そうか。。。ありがと。

>>879
a(n)=S(n)-S(n-1)がn=1でも成り立つかどうかってことね。。。
ごめんログの概略しか見てなかったかもしれない。。。
882大学への名無しさん:04/10/17 03:11:06 ID:dbhszTcB
>>880
S_n=a_1+a_2+……+a_(n)
a_n=S_n-S_(n-1) (n≧2) or S_1 (n=1)
だから常に成り立つね。
883大学への名無しさん:04/10/17 03:22:35 ID:bLoZPwrt
>>882
S(n)がa(0)からの和かa(1)からの和かは問題によるって。
884大学への名無しさん:04/10/17 03:24:59 ID:dbhszTcB
>>883
n'=n+1
とおけば問題ない。
885大学への名無しさん:04/10/17 03:28:43 ID:bLoZPwrt
>>884
その通り。逆に言えばS(100)=a(100)ともS(-20)=a(-20)とも言える。
定義次第ではS(100)=a(-20)にだって出来る。
ゆえに常にS(1)=a(1)だというのはナンセンス。問題文から判断する。
886大学への名無しさん:04/10/17 03:31:53 ID:8x+Pr+je
バカか?nは自然数で定義されんだよフツー。何負の数値いれてんだ禿
887大学への名無しさん:04/10/17 03:33:18 ID:8x+Pr+je
885はキティ害としか思えない
888大学への名無しさん:04/10/17 03:33:33 ID:2EstjM6O
あのすみません。質問はもうちょっとあとのほうがよろしいでしょうか?
889大学への名無しさん:04/10/17 03:34:01 ID:07ZbbWZR
場合によっては-∞〜+∞っていう可能性もあるよ。
ところによってはS(n)=Σ[k=-∞,n] a(n)  ( ← 極限を取って考えてね。収束する場合のみ )
とかって書いてあるのもあるしな。自然数とは限らない罠。
890大学への名無しさん:04/10/17 03:34:28 ID:07ZbbWZR
>>888
今でも問題なし
891大学への名無しさん:04/10/17 03:35:23 ID:2EstjM6O
では書いてきます。一応出典はオークションで買った京大数学理系です。
892大学への名無しさん:04/10/17 03:37:49 ID:8x+Pr+je
どこの問題だかもってこい。大体ある規則にしたがって並ぶのが数列。第一項以前を考えようとするのがナンセンス。だいたい何自分で定義してんだ?神になったつもりか?
893大学への名無しさん:04/10/17 03:39:18 ID:2EstjM6O
xの多項式f(x)があり、任意の実数aに対して,f(x)-f(a)がつねにx^3-a^3で割り切れるとする。このとき、ある多項式g(x)によって、f(x)=g(x^3)と表せることを示せ


セクションは複素数と書いてありますが、どう展開すればいいのやらorz
894大学への名無しさん:04/10/17 03:40:08 ID:8x+Pr+je
しかもらりった和分式まで持ちだして。ありえない
895大学への名無しさん:04/10/17 03:41:28 ID:8x+Pr+je
因数定理臭いな。
896大学への名無しさん:04/10/17 03:54:52 ID:2EstjM6O
下から進まないんです
f(x)-f(a)=(x^3-a^3)Q(x)=(x-a)(x^2+ax+a^2)Q(x)=(x-a)(x-α)(x-β)Q(x)
ただしα=(-1-√3i)a/2,β=(-1+√3i)a/2
またα=a{cos(4π/3)+sin(4π/3)},β=a{cos(2π/3)+sin(2π/3)}とかける。
複素数ωを使って表せばω^3=a^3の解・・・?


897大学への名無しさん:04/10/17 04:05:02 ID:07ZbbWZR
>>893
補題
h(x)をxについての多項式とする。xh(x^3) - ah(a^3) が任意の実数aについてx^3-a^3で割り切れるなら、hは恒等的に0である。

x^3=1の1でない解の一つをωとして、xh(x^3) - ah(a^3)がx-a、x-ωa、x-ω^2aで割り切れると仮定する。
x-aで割り切れることは明らかなので、x-ωaで割り切れるかどうかを考える。
xh(x^3) - ah(a^3) にx=ωaを代入する。
ωah(a^3) - ah(a^3) = (ω-1)ah(a)
これが全ての実数aに対して0になるためにはhが恒に0でなければならない。

この補題と同様にh(x)をxの多項式としたときx^2h(x^3) - a^2h(a^3)がx^3-a^3で割り切れるときはhが恒等的に0になる。


今、明らかに
f(x) = g(x^3) + xh(x^3) + x^2p(x)  、g,h,pはxの多項式、
と表記できるので、上の補題より条件を満たす。
898大学への名無しさん:04/10/17 04:07:49 ID:2EstjM6O
(a)=f(α)=f(β)=0よりf(ω)=0
f(x^3)=f(ω^3)=0なのでf(x^3)=g(x^3)とすることにより表せる。

これはなんか気持ち悪いんですけど
必要条件がいえて十分条件は逆のことを言えばいい?のかなorz
899大学への名無しさん:04/10/17 04:09:07 ID:2EstjM6O
となんか入れ違いになってるし、、
ありがとうございます。拝見させていただきます
900大学への名無しさん:04/10/17 04:24:34 ID:2EstjM6O
>>898はおかしいのでスルーで結構です。
901大学への名無しさん:04/10/17 04:36:46 ID:07ZbbWZR
>>900 んなことは分かってる。俺の書いたのはあってるか?
あってて、理解できたら、もう寝るから、なんか言え。
902大学への名無しさん:04/10/17 04:44:25 ID:dbhszTcB
もっと簡単に。
3回以上微分すると定数項にQの微分した回数が3回差ずつのものしかこないことが分かるから、
それを言えば、Qには3の倍数乗項のものしかこないことがわかる。
そうすればf(x)=g(x^3)だと言える。
903大学への名無しさん:04/10/17 04:45:24 ID:dbhszTcB
これなら複素数も使わないし、簡単な微分だけで片付く。

暇なんで別解考えてました。
904大学への名無しさん:04/10/17 04:49:10 ID:dbhszTcB
定数項っていうか、
f(0)のときの値ね。
905大学への名無しさん:04/10/17 04:50:48 ID:2EstjM6O
>>897
文をにらめっこしてましたが流れがよくわからないです
>>902
拝見させていただきます
906大学への名無しさん:04/10/17 04:53:48 ID:2EstjM6O
>>902
あれ?多項式って微分可能とかって普通に使えましたっけ?
907大学への名無しさん:04/10/17 04:54:45 ID:07ZbbWZR
>>905
あっそ、んじゃ、もう寝るわ。ぱっと見たけど、俺が書いたのには
大きな間違いがないみたいだし、特に大恥かくことも無いだろうと言うことで、
今日はお休み。
908大学への名無しさん:04/10/17 04:58:19 ID:2EstjM6O
なんかあるはずなんだけどな。複素数だからもっと簡単にできる方法が。
909大学への名無しさん:04/10/17 05:01:51 ID:dbhszTcB
あああああああああああ
みっすってるううううううううううううううううううううううううううううううう。
910大学への名無しさん:04/10/17 05:02:24 ID:dbhszTcB
もういいや、寝るノシ
911大学への名無しさん:04/10/17 06:28:54 ID:s14UlsIV
>>892
フーリエ級数の部分和とか.S(n) = Σ[k=-n,n] a(n), a(n) = exp(i n x)
こんなんいくらでもあるよ.
912大学への名無しさん:04/10/17 09:07:28 ID:8x+Pr+je
ごめん、俺が間違ってた。a0が初項の問題早速みつけてきた。Snがらみじゃなかったけど。まあ高校数学の範囲ではnが負の問題は見たことないんでフーリエなんていわれたら利根川にとびこんで寒中水泳してくるしかない。土下座m(_ _)m
913341:04/10/17 09:22:04 ID:3UUvEqEy
数学ピンチ・・・
914大学への名無しさん:04/10/17 10:54:38 ID:IoeRJ1Co
蒸し返すようで悪いんですが、はじていの数列の問題
S(n)=3^n+1 (n=1.2.3...)
の一般項を求めよという問題で
n=1の時、a(1)=S(1)=9
n≧2の時求めた一般項a(n)に1を代入したa(1)=6
となりa(1)が食い違うことをはじていレベルの僕でも分かるような説明お願いします。

それとはじていには階差数列の所でもa(1)が一致しないこともあるような感じで書いてあるんですが、それはどういうことでしょうか?
915大学への名無しさん:04/10/17 11:08:43 ID:xZDOFhS6
>>914
S(n)-S(n-1)=a(n) (n≧2)
=3^(n+1)-3^(n)=2*3^(n)
a(1)=S(1)=9
∴a(n)=2*3^(n) (n≧2)or 9 (n=1)
…何か問題でも?
916大学への名無しさん:04/10/17 11:10:22 ID:GH6FlNd4
1 と 3 は互いに素?
917大学への名無しさん:04/10/17 11:12:45 ID:xZDOFhS6
>>916
http://www.asahi-net.or.jp/~tt9h-hskw/sugaku/gcm-lcm/
最大公約数が1となる2つの数は、「互いに素である」と言います。

これを参考に考えろ。
918大学への名無しさん:04/10/17 11:17:00 ID:i4/TlkTP
a(n)=∫{(1−x)^(n-1)/(n-1)!}×e^x・dx[0,1]

のとき0<a(n)<(e/n!)を示せという問題ですけど
教えて下さい
919大学への名無しさん:04/10/17 11:23:31 ID:xZDOFhS6
>>918
部分積分をすると
a_n-a_(n-1)=-1/(n-1)!になるから、
そこからもうちょっと考えてみて。
920大学への名無しさん:04/10/17 11:27:48 ID:i4/TlkTP
解答では
a(n)< ∫{(1−x)^(n-1)/(n-1)!}×e^1・dx[0,1]

ってなってるんですけど何でですか?
921大学への名無しさん:04/10/17 11:30:00 ID:mB+rYHIJ
e^x≦e (0≦x≦1の時)だから、
922大学への名無しさん:04/10/17 11:37:02 ID:i4/TlkTP
(e/n!)ってかなり0に近い数なんですよね?
923大学への名無しさん:04/10/17 11:37:47 ID:xZDOFhS6
>>922
nが大きければな。
924大学への名無しさん:04/10/17 11:39:47 ID:i4/TlkTP
ありがとうございます
925大学への名無しさん:04/10/17 12:00:26 ID:GH6FlNd4
>>917
わざわざthx!
926大学への名無しさん:04/10/17 12:06:38 ID:IcqHFh40
893についておねがいします
927大学への名無しさん:04/10/17 12:12:35 ID:ycmIT/5c
>>926
>>897>>902はスルーか?
928大学への名無しさん:04/10/17 12:17:13 ID:xZDOFhS6
>>926
x^(3n+1)-a^(3n+1)を(x^3-a^3)でわると、xa^(3n)+a^(3n+1)あまり、
x^(3n+2)-a^(3n+2)を(x^3-a^3)でわると、x^2*a^(3n)+a^(3n+2)あまり、
x^(3n)-a^(3n)を(x^3-a^3)でわると、わりきれることから、
多項式をf(x)=Σ[k=0 to n] b_k x^kとおけば、
f(x)-f(a)=Σ[k=0 to n]b_k (x^k-a^k)を(x^3-a^3)で割ったあまりは
Σ[n=0 to n] b_(3n+1){xa^(3n)+a^(3n+1)}+b_(3n+2){x^2*a^(3n)+a^(3n+2)}
とかけ、これらは互いに次元が違い、素なので、任意のx, aに対して成り立つには、
∀n∈N b_(3n+1), b_(3n+2)=0
でなければならない。
929大学への名無しさん:04/10/17 12:20:16 ID:IcqHFh40
902についてはコメントしました。もうひとつのはあとから読んでわかりましたが、別解をね。
930大学への名無しさん:04/10/17 12:24:38 ID:xZDOFhS6
>>926, >>929
別解が欲しいだけなら普通に別解が欲しいといえば角も立たないのではないかと思うが。
931大学への名無しさん:04/10/17 12:47:58 ID:I97Q84y8
>>928
xa^(3n)+a^(3n+1)
−では?
932大学への名無しさん:04/10/17 14:21:15 ID:8x+Pr+je
これから言葉遣いには気をつけようと思う。ごめんなさい
933大学への名無しさん:04/10/17 14:30:01 ID:mB+rYHIJ
>>932
言葉遣いを改めると同時についでだから、ローラン展開でも調べてろ
934大学への名無しさん:04/10/17 14:34:56 ID:R/Vk3wva
x+7<5x+3

   4≧a^2−ax+2x
誰かこの不等式といてよぅ先生に怒られちゃうYO つД`) タスケレ !!
935大学への名無しさん:04/10/17 15:03:32 ID:bLoZPwrt
>>934
上は普通に解けるべ?
下はa≦2とa≧2で場合わけ。
936大学への名無しさん:04/10/17 15:10:30 ID:IUKEEYpz
>>934
先生に怒られることが問題なのでなく、自分で解けないことが問題であることに気付くことが重要である。

a>2 , a=2 , a<2 のときで場合わけ
a<2 のときはさらにもう1回場合わけが必要そう
937 ◆OlSvCzhmFk :04/10/17 15:49:45 ID:MpXIxCei
解説についての疑問点の質問です。
参考までに出典は87年の京都大学です。

y=(-4/27)a^3

y=-a-1

という三次関数と直線の交点が一瞬で(-3/2,1/2)と求まっているのですが、
(正確には求めるプロセスが書いてありません)
これは一体どうやって求めたものなのでしょうか。
連立してもどう解けばいいのかさっぱりです。
領域の図示問題なので交点の求め方とかは軽く扱われてるっぽいのですw
938大学への名無しさん:04/10/17 16:13:18 ID:IxN6MU+r
連立して因数定理
939大学への名無しさん:04/10/17 16:15:03 ID:mB+rYHIJ
>>937
因数定理使って普通に求めればいいじゃん。
4a^3 - 27(a+1) = 0 でaにめぼしい値を入れていくんだろ?

そでなければ、カルダノの解法
x^3 + ax +b =0
a=-3pq  b=p^3+q^3 として、
x^3 + ax +b
=(x+p+q)(x^2+p^2+q^2+xp+pq+qx)=0

p^3とq^3は二次方程式から求まるとして……

っていうやり方を使うか。
940大学への名無しさん
>>937
軽く出すだけなら、適当にうまくいきそうな値を入れてみればよい。
この場合一本目の式でうまく約分できるように a の分母に2、分子に3あたりはすぐ思いつくから
まず a=±3,±3/2 あたりは真っ先に試してみる値だと思う。

この方法でたまたまうまくいけばよし。というかうまくいくように問題が作られているんだろ?